Этого треда уже нет.
Это копия, сохраненная 25 декабря 2022 года.

Скачать тред: только с превью, с превью и прикрепленными файлами.
Второй вариант может долго скачиваться. Файлы будут только в живых или недавно утонувших тредах. Подробнее

Если вам полезен архив М.Двача, пожертвуйте на оплату сервера.
mandarin01.jpg146 Кб, 1200x900
МАТЕМАТИКА ДЛЯ НАЧИНАЮЩИХ, ТРЕД 25: Новогодний 6879 В конец треда | Веб
В этом треде мы изучаем математику, ну или начинаем это делать. Если ты школьник или студент, и тебя есть задача, то в здесь тебе помогут её решить или хотя бы скажут, в каком направлении двигаться для её решения.
Также приветствуется обсуждения самого процесса изучения и учебников/задачников

Архивы тредов
http://pastebin.com/kiRZGVHW
ВНИМАНИЕ! ВНИМАНИЕ!! ВНИМАНИЕ!!!

On-line LaTex. Формулы пишем в нём, а после прикрепляем картинками к посту
http://www.codecogs.com/latex/eqneditor.php
Львовский Набор и вёрстка в системе latex
http://www.mccme.ru/free-books/llang/newllang.pdf

Если ты только зашел в тред и хочешь спросить, какую книжку прочитать, то ответ, скорее всего, будет в этих списках, анон.
Список от ОП-а, бывшего тут до меня. Был составлен на протяжении 13 тредов, к ознакомлению обязателен.
http://pastebin.com/4iMjfWAf
Список от анона с dxdy. Довольно внушителен, тоже рекомендуется к прочтению. Является дополнением к списку старго ОП-а.
http://pastebin.com/YP1uaUyd
Goodbook.txt список книг с dxdy, рекомендованный тамошними обитателями.
http://pastebin.com/4FngRj6n
Литература - НМУ
https://docs.google.com/spreadsheets/d/1UWwIIAFwSwOQLK3m--LOaMOvHUivFDEz-JAnLa87i7Q/edit#gid=0
ОП-список 2. Составляйте список в реальном времени! Предлагайте в тред книги, критикуйте уже имеющиеся!
http://pastebin.com/szzZfkCM
Форчановский список, книги на английском.
http://4chan-science.wikia.com/wiki/Mathematics
Список с видеолекциями(в разработке):
http://pastebin.com/S3d7Jj6J
Качать книги тут:
http://libgen.io
А статьи тут(в разработке):
http://pastebin.com/3BfHPskz
Мемасы(в разработке):
http://pastebin.com/e38Yuj5V

СПИСКИ В РАЗРАБОТКЕ, НУЖНА ПОМОЩЬ АНОНА, ПОЖЕЛАНИЯ ОСТАВЛЯЕМ В ТРЭДЕ
2 6882
>>6879 (OP)
Реквестируется логика/ключевые моменты и контекст, перспективы изучения математики в одном посте в двух предложениях лол - ничего лишнего, ни воды, ни неба, ни самой математики. Только суть.
3 6883
>>6879 (OP)
Джон Дербишир
Простая одержимость.
Риман

Кому понравилась?
4 6884
>>6883
Мне понравилось. Хорошо сделали.
5 6885
Извините, что врываюсь со своим нытьём и мешаю талантливым анонам изучать азы математики, но я просто в отчаянии.
Недавно составляли мою психологическую характеристику, и там отметили, что у меня низкий уровень абстрактного мышления и плохая математическая интуиция. Там надо было продолжить числовые ряды и решить несколько "стандартных" задач. На всё отводилось определённое время. Я очень долго думал над задачами, и поэтому прошёл тест хуёво. Я всегда знал, что сосу в математике, хотя она мне и нравится (в школе вроде легко справлялся со всеми задачами), но этот тест окончательно добил меня.

Это значит, что у меня хуёвые способности и это не лечится вообще, или практика всё таки поможет?

Пиздец, я из-за этой хуйни уже две недели в глубокой депрессии, постоянно рыдаю, как шлюшка, даже к врачу обратился. Он сказал, что я дебил, и выписал мне антидепрессанты.
6 6886
>>6885
Что за тест? Что за задачи?
7 6887
>>6886
Вот этот тест http://psytest24.ru/amthauer/
Задачи пикрилейтед, чуть видоизменённые. Все остальные субтесты хорошо прошёл, а вот здесь тормозил люто.
14814251192550.png37 Кб, 618x388
8 6888
>>6887
А, этот тест. Я его в психаче проходил. Так как плохо считаю, то ели-ели справился с немногими задачами из первого листка. Я там ещё невнимательно задания прочитал в одном месте и сильно затупил на словах. Даже тебе пикчу скину. Там и видно, на каких тестах ступил.
Эти задания, особенно первый листок, имеют мало общего с математикой. Они скорее инженерные-физические. Вторые, правда, более абстрактные, но всё равно. Бурбаки бы этот тест не прошли бы.
Так, что можно сделать вывод, о том, что численными методами тебе не стоит занимаеться. Ка и мне, лол. Можешь топологиями обмазаться.
9 6889
>>6888
Спасибо, антоша. Буду продолжать копротивляться, может в конце концов окажется, что я не такой уж хуй.Хотя из-за этого теста я всё равно ещё недели две истерить буду, потому что у меня что-то вроде психотравмы по этому поводу, лол.
10 6897
Спрошу здесь, так как задача скорее математическая.
Даны матрица A размером 30к*10 и вектор b, элементы матрицы и вектора целые положительные числа. x -- некоторое решение уравнения Ax = b в целых неотрицательных числах. Естественно, вектор x будет состоять в основном из нулей.
Дан функционал F(x) = номер последнего ненулевого элемента x.
Нужно минимизировать {F(x) | Ax = b}. Не обязательно находить точное решение, сойдет и какой-нибудь относительно маленький (понятие "относительно маленький" сложно определить) локальный минимум.
В голову приходит лишь найти какое-нибудь хорошее приближение F(x) функционалом типа c^{t}x, получившуюся задачу линейного программирования скормить комплуктеру (есть пакеты, умеющие решать ЗЛП в целых числах). Но пока получается не очень годно, проблемы в том, как бы получше найти вектор c.
У кого-нибудь есть идеи, как решать данную задачу?
Методом тыка нашел решение min ~ 10к. Но это много, хтелось бы поменьше.
mMfi0p47Chw.jpg148 Кб, 810x1080
11 6898
Двач, есть такая штука,как примитивно рекурсивные функции.
Так вот,мне необходимо УГАДЫВАТЬ формулу последовательности членов.
Но вот возникает вопрос,можно ли использовать что-то кроме сложения/умножения?
препод ничего не сказал, просто необходимо прорешать 30 задач такого типа.
1470444585234.jpg69 Кб, 500x352
12 6910
Спрашиваю, наверное, платину, но тем не менее я начинающий ПОГРОМИСТ, изучаю коддинг чтобы суметь сделать код работать с векторной графикой - генерировать паттерны по определенным алгоритмам применяя как построение с нуля так и уже имеющиеся отдельно элементы. Начал изучать Питон, мне сказали, что мне обязательно понадобится матан для моих кириллистический затей, так что хочу потихоньку начать изучать сразу и матан. Собственно, зашел в раздел чтобы узнать, в каком конкретно направлении двигаться, что мне из математики наверняка пригодится а что может совсем не пригодиться для того, что я описал выше?
13 6911
>>6910
Линейная алгебра, комбинаторика, может какие-нибудь интегральные преобразования, но большинство тем матана тебе вряд ли при пригодятся.
cake.png87 Кб, 651x436
14 6912
>>6911
Сэнькью
15 6919
9.0000009e+93
Поясните дебилу-дошкольнику, что такое e+.

И делится ли число 111...1 (100 единиц) на число 1111111 без остатка?
16 6920
>>6879 (OP)
Добрый день, посоветуйте годные книги по мат.статистике, Теории вероятностей и че-нить про сплайны и т.д
17 6921
>>6919
e+ означает *10^. Мантисса и порядок.
18 6924
>>6919
111..1 = 1111111 + 1111111x10^7 + 1111111x10^14 + ... + 11x10^91 + 11x98^100
Математика и программирование 19 6942
Какие бывают кодерские задачи, где нужен анализ/линал? Хочу совместить приятное с полезным.
20 6943
И действительно ли хаскелль поможет математизировать байтослесарский тип мышления?
21 6944
>>6942
Машинное обучение/игры.
22 6946
>>6942
project euler
23 6949
>>6943
Да, как и любой expression-based язык. Типы и чистота тоже весьма математичны. И в коммунити есть немного интересующихся математикой людей.
Screenshot2017-01-03-04-41-50.png332 Кб, 720x1280
24 6956
Объясните как это вообще решение произошло. Почему не прокатило взять половину от прямоугольного сечения? Хоть и в действительности взята половина, но по цифрам половина не получается. Почему для нахождения площади брали гипотенузу треугольника? Заранее спасибо.
25 6961
Тонкая линза находится между двумя точечными источниками света на расстоянии 1 = 36 см от
одного из них. Источники расположены на главной оптической оси линзы. Расстояние между ними
составляет = 48 см. Каково фокусное расстояние линзы, если изображения источников находятся
в одной точке?

Можете объяснить как решать? Подгоном получено верное числовое значение, но не уверен что в формульном виде правильно решил.
26 6962
>>6956
В условии дана диагональ BD1 т.е. гипотенуза, и одна из сторон прямоугольного треугольника. Для нахождения S параллелограмма нужны 2 его стороны, соответственно по теореме Пифагора находят вторую его сторону
>>6961

>на расстоянии d1 = 36 cm


>Расстояние между ними составляет L = 48 cm

IMG2017-01-04 13:54:43.jpg101 Кб, 960x1280
Сап матемач , ряды , тфкп ... 27 7004
через 2 дня экзамен , плоховато знаю. помогите решить , с меня как обычно
Теормех ебущий 28 7005
Знаю, что доска про математику, но хз куда еще запостить. Решаю это говно, в этом плохо понимаю. Задача простая, вы люди умные, в уме решите. Накидал прогу на питоне, симулирующую движение этого мешка дерьма в канализации, там ответ 3.89491 вышел (скорость в точке В). Дальше не решал, сначала я не учел силу трения и ответ сходился с моей прогой, а после учета все похерилось и я уже второй день не понимаю.
29 7006
>>7005
После подсчета в вольфрам|альфа получается хуйня полная
30 7007
>>7005
Да и на ресунке забыл показать силу трения в начале, но, думаю, и так понятно
31 7008
>>7007
*рисунке
32 7010
На каком уровне знания математики надо быть, чтобы изучать теорию топосов?
33 7016
>>7010
Производящие функции, сочетания, эллептические интегралы и сферические функции. Также полезными будут знания аналитической геометрии.
Иди ботай диффуры, интегралы и комбинаторику.
34 7020
>>7004
1. Признак Д'Аламбера.
2. Радикальный признак. Во всех трёх точках сходится абсолютно.
3. Условие Коши-Римана.
35 7021
Есть ли какая-то сборка инфографиков по математике?
36 7025
>>7005
Пиздос блять, в рот ебал, оказывается в задании на АВ силой трения пренебрегаем, а без нее я уже все решил 20 часов назад.
37 7026
>>7025
У тебя ещё на третьем пике во втором уравнении перед Rz плюс стоит, хотя сила против движения направлена.
38 7027
>>6879 (OP)
сука бл, хули нет ни одной книги по теории вероятностей в шапке?
39 7028
>>7026
Ну это только там опечатка, далее я это учитывал, там потом сумма сил и перед ними минус, но почему с прогой не совпадает все равно, хоть можно и без этого.
40 7033
Неужели нет красивого сайта с интересными картинками, где есть хорошие уроки математики?
41 7034
>>7033
Mathprofi, khan Academy
42 7038
>>7005

>ебущий по лезвию теормеха


ебучий наверное, а не ебущий?

>>7027
это просто невероятно! какова вообще вероятность того, что в шапке маттреда не будет книги по вероятностям??
20170105131331.jpg1,7 Мб, 2560x1920
43 7054
Сап мат. На связи залетный, пишу прямиком из школы. Помоги математический двачер решить мне простую школьную задачу, пожалуйста.
13143567678.gif2 Кб, 298x67
44 7061
45 7062
Анон, подскажи годноты по математической статистике.
46 7073
>>7062
Чем уже закидывался?
47 7076
Что делать, если я плохо обучаемый?
48 7079
>>7073
По матстату ничем ещё.
49 7080
>>7076
Сначала учить наизусть, потом пытаться понять до тех пор, пока не дойдет. И решать много-много задач по теме.
50 7082
>>7076
Не думать про обучаемость. Человек не способен сам оценить свою обучаемость, он способить только недооценить (не понимаю что вообще надо делать) или переоценить (читаю знакомые слова значит знаю). Хороший рецепт это работать и оценивать реальный показатель, именно время потраченное на что-то. Если человек 100 часов читает материал по теме, все более простой и простой и до него не дойдет, вот это реальная необучаемость.
IMG20170105215813.jpg681 Кб, 4160x1156
Аааэээ она меня сожрет 51 7085
Крч взял повторять профильную базу по матике (задачи Сканави, а учебник Ткачук для поступающих во ВТУЗы). Вот сижу я над уроком с пометкой [2] (тип на лицейскую четверку), тема: сложные системы уравнений и вот у меня вопрос.
Как блять догадаться каким способом решать эту хуиту??? Там же дохуя методов, Антон, как найти нужный? Прикрепляю задание для примера (10)
52 7087
>>7085
типовой способ это замена переменных для упрощения алгебраической ебли. например x+y=a, xy=b. Но прежде чем это делать, надо преобразовать выражении, чтобы оно зависело только от этих переменных. Для этого надо прежде всего "избавиться" от степеней.
в примере 10 моя первая идея была выделить (x^2-y^2)^2 из первого уравнения, потом разложить это на (x+y)^2(x-y)^2, а потом представить как (x+y)^2(x^2-2xy+y^2), и далее как (x+y)^2((x^2+2xy+y^2)-4xy), а после уже сделать замену на a и b.
53 7089
Как-то раз, на очередной вопрос сына о том, что такое геометрия, Этьен кратко ответил, что это способ чертить правильные фигуры и находить между ними пропорции, однако запретил ему всякие исследования в этой области. Однако Блез, оставаясь один, принялся углём чертить на полу различные фигуры и изучать их. Не зная геометрических терминов, он называл линию «палочкой», а окружность «колечком». Когда отец случайно застал Блеза за одним из таких самостоятельных уроков, он был потрясён: мальчик, не знавший даже названий фигур, самостоятельно доказал 32-ю теорему Евклида о сумме углов треугольника. По совету своего друга Ле Пайера Этьен Паскаль отказался от своего первоначального плана обучения и разрешил читать сыну математические книги.

Можно ли дойти до похожего? С геометрии я все забыл.
IMG20170105230844.jpg943 Кб, 4160x3120
54 7090
>>7087
А вот со вторым случаем что делать?
55 7092
Как вы вообще из символов и сухих формулировок конвертируете в математические концепции и прочую для размышлений? Я просто смотрю на формулу или сухой текст, и мне сверхтяжело, я или схватываю лишь детали, или какое-то слишком общее. Что делать? Есть ли вообще материалы что-то вроде "Мышление и установки математиков"?
56 7093
>>7092
Вообще что делать с комплексами и страхами перед математикой?
tmp7422-CodeCogsEqn-1303497374.gif4 Кб, 395x153
57 7097
>>7090
Да просто разложил ты как-то по-пидарски
мимопроплывал
58 7130
>>7093
Отбросить, как ненужное.
59 7135
>>7079
Marco Taboga, Lectures on Probability Theory
and Mathematical Statistics
Если надо вероятность освежить, посмотри что там всякие мехматы выкладывают, вроде
http://www.nsu.ru/mmf/tvims/lotov/MMF.pdf
60 7136
>>7089
Можно, но скорее будешь как Рыбников, а не Паскаль.
61 7137
Почему никто не выпустит книгу о математике с йобой и мемасиками?
62 7144
>>7137
есть манга книги по всем темам, целая серия
здесь периодически мелькают
http://nnmclub.to/forum/viewtopic.php?t=993281

Единственный плюс таких учебников - можно подрачивать прямо в процессе обучения
63 7162
Объясните мне про векторные пространства.
Вот наапример возьмем плоскость. Рисуем на ней координатную сетку, задаем векторное уравнение, допустим икс это тэ, игрек это тэ квадрат. Рисуем линии, все замечательно.
Но вот только вектор это же, грубо говоря, стрелочка. В какую сторону она должна быть направленна в моем случа, если я захочу изобразить все направленными стрелочками для наглядности??
64 7164
>>7162

>векторные пространства.


>Рисуем на ней координатную сетку


Ты мальца перепутал. Векторные пространства без координатных сеток, с сетками аффинные.

>задаем векторное уравнение, допустим икс это тэ, игрек это тэ квадрат


>Но вот только вектор это же, грубо говоря, стрелочка. В какую сторону она должна быть направленна в моем случа, если я захочу изобразить все направленными стрелочками для наглядности??


Можешь выбрать положительное направление на кривой, при t -> бесконечности.
65 7185
Как называется линия вокруг шара?
66 7187
>>7185
Какая линия вокруг какого шара?
67 7191
>>7187
Ну как орбита у планеты, тока шобы по земле проходила
68 7197
>>7191
Большая окружность?
69 7227
Объясните, пожалуйста, как решать задачи с sign из обоих билетов.
s143.gif2 Кб, 246x154
70 7237
>>7227
1.Раскладываем в циклы
2.Возводить надо в отрицательную степень,поэтому ищем обратную перестановку
3.Любые циклы в степени своей длины - единичные перестановки, так что достаточно взять степени по модулю длины
4.Вычисляем знак
В другом задании аналогично,только надо ещё проделать ту же муть для h и перемножить.
71 7242
Посоветуйте, пожалуйста, какой-нибудь основательный учебник по геометрии университетского уровня.

В идеале, хотелось бы что-нибудь типа аналога фихтенгольца или кострикина, то есть несколько томиков с максимальным уровнем разжевывания, покрывающих большую часть университетского курса геометрии, и так, чтобы я, для начала, мог как минимум пролистнуть всю эту хуйню по диагонали и просто понять, из каких разделов она блядь вообще состоит.
72 7243
>>7242
Александров Лекции по аналитической геометрии.
73 7245
>>7242
>>7243
Кирпич Александрова хоть и староват маленько, но хорош.
Однако, здесь речь шла немного о другом, и просто невозможно не посоветовать 6-томник Постникова.
74 7247
Подойдет ли книга Ткачука "Математика абитуриенту" для подготовки к егэ?
75 7249
>>7247
Да, вполне.
76 7250
>>7245
Шеститомник? Я только у него помню учебник по аналу с исользованием групп.
77 7313
>>7243
>>7245
Спасибо, котики.
78 7328
>>7237
Спасибо
79 7353
Модер, добавь в шапку.

Школьная математика: http://pastebin.com/Yn8DSzga

Копипаста: heller.ru/blog/2010/12/math-topics-list/, heller. ru/blog/2013/05/math-plan/
Книги: drive.google.com/folderview?id=0B47MUz8iO65LfnkzWUxGdXFqZXByUDVOb0xhNmp1Z1h5aWttb25CaEVJS3hrblpZdFN3N1E&usp=drive_web

Копипаста: imperium.lenin.ru/~verbit/MATH/programma.html, ium.mccme. ru/f04/experimental.html
Книги: drive.google.com/folderview?id=0B47MUz8iO65LfkhMdGt0RTlxUnZkOU9SUjdGUjNIWXRxdmhRWjloWlE2bi01ZGxYNkVxbXc&usp=drive_web

http://lj.rossia.org/users/dmitri_pavlov/
Книги: drive.google.com/folderview?id=0B47MUz8iO65LflZSWGZxcjlvdnRnZzhQczNPYzFtZnFZZENlemlmNklDclg5RmdJNVNsTjA&usp=drive_web

http://verbit.ru/Job/HSE/Curriculum/all.txt
Есть всё кроме:
Цфасман, Влэдуц, Ногин - Алгеброгеометрические коды. Основные понятия
Я. Элиашберг, Л. Трейнор - Лекции по симплектической геометрии и топологии
M. Gromov - Carnot-Caratheodory spaces seen from within
Книги:https://yadi.sk/d/XA0kQ40L37bPdy

Список НМУ
Книги: http://pastebin.com/At2vbyFN

Копипаста: http://pastebin.com/cH7HDf4U
Список: http://pastebin.com/PQ6nsAQj
Книги: https://yadi.sk/d/cMT8z8UBvyEyN

Ссылки:
https://sites.google.com/site/scienceandmathguide/subjects/mathematics
http://freebookcentre.net/SpecialCat/Free-Mathematics-Books-Download.html
79 7353
Модер, добавь в шапку.

Школьная математика: http://pastebin.com/Yn8DSzga

Копипаста: heller.ru/blog/2010/12/math-topics-list/, heller. ru/blog/2013/05/math-plan/
Книги: drive.google.com/folderview?id=0B47MUz8iO65LfnkzWUxGdXFqZXByUDVOb0xhNmp1Z1h5aWttb25CaEVJS3hrblpZdFN3N1E&usp=drive_web

Копипаста: imperium.lenin.ru/~verbit/MATH/programma.html, ium.mccme. ru/f04/experimental.html
Книги: drive.google.com/folderview?id=0B47MUz8iO65LfkhMdGt0RTlxUnZkOU9SUjdGUjNIWXRxdmhRWjloWlE2bi01ZGxYNkVxbXc&usp=drive_web

http://lj.rossia.org/users/dmitri_pavlov/
Книги: drive.google.com/folderview?id=0B47MUz8iO65LflZSWGZxcjlvdnRnZzhQczNPYzFtZnFZZENlemlmNklDclg5RmdJNVNsTjA&usp=drive_web

http://verbit.ru/Job/HSE/Curriculum/all.txt
Есть всё кроме:
Цфасман, Влэдуц, Ногин - Алгеброгеометрические коды. Основные понятия
Я. Элиашберг, Л. Трейнор - Лекции по симплектической геометрии и топологии
M. Gromov - Carnot-Caratheodory spaces seen from within
Книги:https://yadi.sk/d/XA0kQ40L37bPdy

Список НМУ
Книги: http://pastebin.com/At2vbyFN

Копипаста: http://pastebin.com/cH7HDf4U
Список: http://pastebin.com/PQ6nsAQj
Книги: https://yadi.sk/d/cMT8z8UBvyEyN

Ссылки:
https://sites.google.com/site/scienceandmathguide/subjects/mathematics
http://freebookcentre.net/SpecialCat/Free-Mathematics-Books-Download.html
80 7355
>>7353
Шапку составляет оп, а не модер.
81 7358
>>7353
Откуда все это?
Твоя работа?
Тi кто будешь?

>Алгеброгеометрические коды


Охуеть! С 2003 так никто и не отсканировал?!
Ну тогда я могу исправить эту несправедливость, в ближайшее время ждите от меня pdf-ку
82 7359
>>7358
Большое спасибо, анон
83 7360
>>7353

>Carnot-Caratheodory spaces seen from within


Ну вот же:
http://gen.lib.rus.ec/book/index.php?md5=4CEF95CF8A5F2DA286E9008771D26C65
84 7371
>>7367
Матрицы - легко. Производные - легко.
Пределы и интегралы сам нихуя не понимаю.
85 7374
>>7367
Матпрофи.
CompScicompressed1920x1280.jpg171 Кб, 1220x800
86 7380
+++Производная, Дифференциал и Интеграл+++

Помогите, пожалуйста, разобраться в этих страшных зверях полному чайнику.
Что они из себя представляют на самом деле простым языком, желательно с картинками, но можно и на пальцах.
Можно объяснять как ребёнку.

Мне бы самое главное понять какие именно элементарные математические операции выполняются при их помощи? Это самый главный вопрос.

поправил, сорьки.
87 7382
>>7380
mathprofi
88 7384
объясните долбоебу (мне) что такое синус косинус и тангес и с чем эту хуету едят
с меня как всегда
89 7386
>>7360
Ага, я уже позднее нашёл и добавил.

>>7358
Просто хобби, собирать интересные книги по спискам

>>7367
В свое время при сдаче очень помогла книга Фихтенгольца (1 том), учитывая архаичность образования в рахе.

В матрицах до сих не могу запомнить определений, если бы всё было так просто, как у Арнольда: "Определитель матрицы – это (ориентированный) объём параллелепипеда, рёбра которого – её столбцы. Если сообщить студентам эту тайну (тщательно скрываемую в выхолощенном алгебраическом преподавании), то вся теория детерминантов становится понятной главой теории полилинейных форм. Если же определять детерминанты иначе, то у каждого разумного человека на всю жизнь останется отвращение и к определителям, и к якобианам, и к теореме о неявной функции."
90 7394
>>7164
Тобишь общепринятого способа обозначать направление нету?
А то я в книжке смотрю на нарисованные стрелочками векторные картинки и понять не могу почему стрелочка туда или сюданаправленна.
91 7395
>>7394
Скрин картинок?
92 7396
С каких книг арнольда нужно начинать закатыватся в вышмат?
инб4 гамалогея и тапалогея для жоаполазов
93 7397
>>7396
Арнольда очень сложно читать на самом деле, потому что он предполагает, что ты уже многое знаешь.
Закатыватся с него не стоит.
tmp14915-Trigonometricfunctionsen-1303497374.png213 Кб, 2468x1215
94 7398
>>7384
Длины (со знаком) отрезков вокруг угла и описанной единичной окружности.
95 7400
>>7398
Смотри, точки окружности - (cos(t);sin(t)). Положительное направление движения на оркужности - это изменение t от 0 до 2пи. Если будет прибавлять параметр t, то точка будет двигатся против часовой стрелки. Ясно?
Screenshot2017-01-07-21-26-33.png278 Кб, 1024x600
96 7401
>>7395
Не могу найти картинку попроще, поэтому возьму вот эту.
Почекму в тридэ варианте по оси зэд горбы?
97 7402
>>7400
Да, но мне-то зачем это объяснять?
98 7403
99 7404
>>7401
А горбы для удобства, они показывают, как прямые в плоскости изгибаются.
100 7405
>>7400
>>7398
аноны мне дико неприятно мне признаваться, но я школоло - долбоеб который доучился до девятого класса и нихуя не учил. прост я думал что синус косинус и тангенс это отношение чего то к чему то
101 7406
>>7405
В прямоугольном треугольнике прилежащего катета к гипотенузе - косинус, противолежащего к гипотенузе - синус, противолежащего к прилежащему - тангенс. Такой треугольник на картинке с окружностью можно углядеть и там в силу единичности гипотенузы они приобретают удобное гееометрическое представление.
102 7411
может кто-нибудь объяснить, в чем противоречие диагонального метода кантора при попытке сосчитать множество действительных чисел? вот можете объяснить пожалуйста по человечески, почему [0, 1) не счетно. я пытался читать док-во и не въезжаю
103 7412
Начал как-то вкатываться в теорию групп, возникли нубские вопросы, Z/6Z - это группа вращения треугольника? Является ли она абелевой? Ну и еще как находить обратимые элементы?
104 7414
>>7411
Потому что числа можно делать бесконечно.
Ни одна религия не запрещает любое число поделить на десять.
105 7416
>>7411
Диагональный метод лишь для счётных множеств, а как ты собрался перечислять трансцендентные числа?
106 7419
>>7416
да я просто не пойму. в чем там противоречие, например вот мы можем записать множество рациональных чисел и выписать в ряд идя по диагоналям. и у этого ряда установить биекцию с множеством натуральных все ок. но почему когда мы вещественные выписываем это не подходит? какое там противоречие, может кто объяснить? я просто студент первак я вот реально не могу понять
107 7420
>>7419
Почитай Пратусевича 10 класс. Начала анализа.
108 7424
>>7420
Хорошая книга?
109 7425
>>7424
Да.
110 7442
http://mathprofi.ru/opredelenie_proizvodnoi_smysl_proizvodnoi.html

Я не очень понял это место.

Как угол альфа вдруг превратился в фи, а длина LN в длину MN (рис. 3), а точнее ниже...

Дельта x не поменялся, а вот дельта y превратился в маленькую dy, а у него в формулах "=" стоит:
Производная функции равна тангенсу фи (4 рис.), хотя до этого сказано, что производная функции равна отношению катетов, которые дают тангенс альфа (2 рис.).
Но достаточно взглянуть на рис. 1 и становится понятно, что углы альфа и фи не равны и LM не равен MN.
111 7443
>>7442

>Дельта x не поменялся, а вот дельта y превратился в маленькую dy, а у него в формулах "=" стоит:



Вот это вот самое мерзкое, паскудное, подлое, самое гадкое и отвратительное, что случалось со мной в моей сознательной жизни, в которой я имел несчастье учиться в третьесортном "математическом" универе. До слёз, до боли в груди, до дрожи в пальцах я ненавижу всю эту гнусность, этих мудил, которые давали мне это на лекциях. Которые спрашивали это на экзаменах. Я ненавижу школу, где мне объясняли "геометрический смысл производной", и это единственное, что я мог что-то понять (как мне казалось) в понятии предело. Они гады, уроды, садисты, они сломали мне жизнь, они сделали из меня овощ, они посадили меня в комнату 101, к крысам. Я не забуду, что я чувствовал, когда на 5м курсе уже я почему-то решил, что не хочу быть ебаным кодером, а хочу заниматься наукой, и сел себя насиловать опять, открыл старые лекции и стал пытаться понять, что такое этот ебаный dx, в который вдруг превращается дельта, потому что никогда не понимал. О, мерзость! Как же я ненавижу себя за это. Я хочу умереть, пусть меня переедед поезд, обольёт кислотой, оторвёт мне руку и ногу, да так, чтобы я жил ещё 1,5 часа, корчась от боли и перебирая свои кишки

Я читаю теперь всех этих долбаных вербитов, его последователей, всех этих матшкольников и студентов вышки, где они обсуждают, насколько это вредно, рашисткое мат. образование. Я читаю, как они жалуются на что-то плохое в нму или в вышке, я читаю то публичное письмо студентов матфака, где они призывают что-то поменять и предлагают свою программу. Я рыдаю. Слёзы обжигают мне щёки. Они, успешные, понаписавшие статей, понаехавшие в универы и на конференции, постигающие высокую науку, так, на досуге, немного возмущаются, что какие-то старпёры занимаются хуйнёй и убивают студентов. Меня убили.

Дорогой анон, пожалуйста, выбрось эти картинки и не старайся их понять. Возьми J. Lee - Introduction to smooth manifolds. Возьми что-нибудь ещё по многообразиям. Я не знаю, что там нужно по калкульсу, возьми Рудина, блядь. Открой math.stackexhange, там всё это тысячу раз и спрошено и разжёвано. Пойми, что производная - это такая функция, которая 1) линейная (т.е. в одномерном случае это умножение на число), 2) отличается от исходной функции на о малое первого порядка. И это всё, что нужно об определении. Разберись, что такое касательное и кокасательные расслоения. Пойми, что кокасательное расслоение над прямой есть одномерное векторное простарнство (над точкой), поэтому все дифф. формы отличаются умножением на число, и именно она и есть производная (в точке). Выучи, что дифференцирование это гомоморфизм модуля в себя, который удовлетворяет правилу Лейбница. И что, кроме производных, в алгебре гладких функцих на гладком многообразии других дифференцирований нет. Анон, учи нормальную математику, выкини это говно. Ради меня. Ради всех нас, кого уничтожили эти ебаные террористы
111 7443
>>7442

>Дельта x не поменялся, а вот дельта y превратился в маленькую dy, а у него в формулах "=" стоит:



Вот это вот самое мерзкое, паскудное, подлое, самое гадкое и отвратительное, что случалось со мной в моей сознательной жизни, в которой я имел несчастье учиться в третьесортном "математическом" универе. До слёз, до боли в груди, до дрожи в пальцах я ненавижу всю эту гнусность, этих мудил, которые давали мне это на лекциях. Которые спрашивали это на экзаменах. Я ненавижу школу, где мне объясняли "геометрический смысл производной", и это единственное, что я мог что-то понять (как мне казалось) в понятии предело. Они гады, уроды, садисты, они сломали мне жизнь, они сделали из меня овощ, они посадили меня в комнату 101, к крысам. Я не забуду, что я чувствовал, когда на 5м курсе уже я почему-то решил, что не хочу быть ебаным кодером, а хочу заниматься наукой, и сел себя насиловать опять, открыл старые лекции и стал пытаться понять, что такое этот ебаный dx, в который вдруг превращается дельта, потому что никогда не понимал. О, мерзость! Как же я ненавижу себя за это. Я хочу умереть, пусть меня переедед поезд, обольёт кислотой, оторвёт мне руку и ногу, да так, чтобы я жил ещё 1,5 часа, корчась от боли и перебирая свои кишки

Я читаю теперь всех этих долбаных вербитов, его последователей, всех этих матшкольников и студентов вышки, где они обсуждают, насколько это вредно, рашисткое мат. образование. Я читаю, как они жалуются на что-то плохое в нму или в вышке, я читаю то публичное письмо студентов матфака, где они призывают что-то поменять и предлагают свою программу. Я рыдаю. Слёзы обжигают мне щёки. Они, успешные, понаписавшие статей, понаехавшие в универы и на конференции, постигающие высокую науку, так, на досуге, немного возмущаются, что какие-то старпёры занимаются хуйнёй и убивают студентов. Меня убили.

Дорогой анон, пожалуйста, выбрось эти картинки и не старайся их понять. Возьми J. Lee - Introduction to smooth manifolds. Возьми что-нибудь ещё по многообразиям. Я не знаю, что там нужно по калкульсу, возьми Рудина, блядь. Открой math.stackexhange, там всё это тысячу раз и спрошено и разжёвано. Пойми, что производная - это такая функция, которая 1) линейная (т.е. в одномерном случае это умножение на число), 2) отличается от исходной функции на о малое первого порядка. И это всё, что нужно об определении. Разберись, что такое касательное и кокасательные расслоения. Пойми, что кокасательное расслоение над прямой есть одномерное векторное простарнство (над точкой), поэтому все дифф. формы отличаются умножением на число, и именно она и есть производная (в точке). Выучи, что дифференцирование это гомоморфизм модуля в себя, который удовлетворяет правилу Лейбница. И что, кроме производных, в алгебре гладких функцих на гладком многообразии других дифференцирований нет. Анон, учи нормальную математику, выкини это говно. Ради меня. Ради всех нас, кого уничтожили эти ебаные террористы
112 7444
>>7442
ок, разобрался
113 7445
>>7442
пиздец сообщение само отправилось

ок, разобрался https://www.youtube.com/watch?v=wYzc4F9GPGQ
Что насчёт матпрофи, он не умеет ни учить, не объяснять, пытается как проще вроде, но при этом приплетает сразу же кучу ненужной сложной хуйни с кучей вложенных скобок, вот тако вот типа правильней, да мне по хуй как првильней бля, мне понять надо тему, а не дерьмовые поучительные идеи выслушивать по улучшению мира, вместо дельта эф везде сука расписывает динное выражение и всё больше и больше нагромождает, проблема с логикой изложения, не делает акценты из-за чего я начал думать, что тангенгенс равен не пределу, а самому выражению, куча лишнего мусора и тупых словечек, улыбки дебильные итд, признак слабоумия.
А в пределах у него куча вычислительных ошибок, он делает несколько действий в голове, хуяк, а дробь забывает перевернуть или ещё подобные вещи.
Ищем другой сайт для изучения математики.
114 7446
Ещё вопрос по производной
https://www.youtube.com/watch?v=wYzc4F9GPGQ

Если чисто логически прикинуть.
Если дельта x получает значение не достигая нуля, пусть даже и очень маленькое, но не равное нулю.
Тогда дельта икс уже не стремится к нулю, а угол альфа не стремится к углу бета и тангенс угла альфа не стремится к тангенсу угла бета, и производная равная тангенсу бета больше не равна пределу (дельта f)/(дельта x).

Но разница может быть настолько мала, что мы её даже не заметим.
115 7447
>>7443
Что нужно на русском читать, чтобы учить математику правильно?
116 7448
>>7447
В ОП-посте же есть списки.
117 7451
>>7443

> Вот это вот самое мерзкое, паскудное, подлое, самое гадкое и отвратительное


Так ведь там не определение dy, а его "геометрический смысл".
118 7453
>>7446
Подумай, что формально значит "очень маленькое" и "стремится к нулю".
119 7454
>>7404
Ты не понял.
Я имел ввиду по оси зед почему вообще что то есть, если её нету в уравнении?
120 7455
>>7454
Значения функции по этой оси откладываются, куда их ещё-то пихать, если x и y - аргументы?
121 7456
>>7446
Если чисто логически прикинуть то нужно читать несколько разных учебников.

А по вопросу, если оставаться в поле вещественных чисел то разница будет числена, но ей можно будет пренебречь.
А если перейти в грань бесконечно малых, то разница так же будет бесконечно малой, но малой высшего порядка, тобишь быстрее будет становится малой.
122 7457
>>7455
Но ведь это параметричекое уравнение, грубо говоря.
123 7459
>>7457
С хуёв оно параметрическое-то? Там же координаты напрямую участвуют. Взяли, скажем, х и y по корню из 2, получили z=f(x,y)=-2e^-2
124 7460
>>7459
Но ведь каждый отдельный член в уравнении не зависит от другого.
Вот взять уравнение окружности икс квадрат плюс игрек квадрат равняется эр квадрат. Тут все связанно, уменьшился икс увеличился игрек, результат тот же.
В параметрической форме это все будет в виде икс эр косинус тэ, игрек эр синус тэ. ВРоде зависимости нету, а она есть
125 7462
>>7460
В уравнении окружости переменных две - x и y.
В уравнении на картинке их 3 - x,y и f(x,y)
126 7463
>>7462
Так, и f(x,y) откладывается по оси зэд, так!?
127 7464
>>7463
Да
128 7465
>>7451
Там говорится о том, как дельта превращается в d. Вот это самое ужасное во всей этой школьной конструкции. Это ложь, не превращается оно, d не имеет ничего общего с приращениями! это обозначение для ковекторов, а не "приращения в пределе" или как там объясняется

Нету у производной никакого геометрического смысла, кроме того смысла, что производная - это линейное приближение функции. То, что производная совпадает с наклоном касательной, - так это отражение (интуитивного ясного) того факта, что касательная есть линейное приближение прямой. Вся эта мерзость с предельными секущими - это ложь, пытки и изнасилования, и массовые убийства.
129 7466
>>7465

>есть линейное приближение прямой



*линейное приближение кривой
130 7470
>>7465

>мерзость с предельными секущими


Так, падажжи ебана.
Вот у нас есть точка a.
Вот у нас производная f в a существует и равна f'(a).
Это значит, что f(a+h) - f(a) = f'(a)h + o(h).
Это значит, что lim (f(a+h) - f(a))/h = f'(a), h -> 0.
Вот у нас есть определение предела по Гейне.
Берем последовательность h1, h2, h3, ... , сходящуюся к нулю.
Строим последовательность f(a+h1)/h1, f(a+h2)/h2, f(a+h3)/h3, ...
Она сходится к f'(a).
Что тут не так?
131 7473
>>7470
Да он не про это. Он бомбалирует, что дифференциал пределом приращения называют, что, в общем-то, пиздёж и есть.
132 7476
как доказать что если последовательность ограничена, то существует супремум или инфинум?
133 7477
>>7476
Ограниченное множество вещественных чисел имеет супремум и инфимум по аксиоме.
134 7481
Что делать, если я ученый дурак? К слову, в школе может и делали из меня калькулятор, так как постоянно опускали доказательства. Да и сам я не очень слушал. Зато могу вести типа умную беседу с неявным резонерством.

Можно ли привести себя в порядок?
135 7482
>>7481
Какие доказательства тебе нужны?

Что сумма интегралов двух одинаковых функций от а до с и от с до б равны сумме интегралов? Очевидно из самого определения интеграла.

Что парабола это линия у которой разница расстояний до выбранной прямой и точке равно константе? Очевидно.

Что неэвклидовые пространства могут содержать треугольники с суммой углов больше 180 градусов? Очевидно из определения неэвклидового пространства.

Ты просто слушал, но не слышал. Чуть менее чем все доказательства в математике это доказательства уровня "2+2-4. А теперь докажем это на примере тапалагических прасранств"
136 7484
>>7473
Имеет место банальная терминологическая путаница. Тот кун называет дифференциалом линейную функцию D такую, что f(a+h)-f(h) = D(h) + o(h). Это современное понимание.

В классическом анализе (до революции Коши) дифференциалами называются прообразы и образы функции D. То есть если D - функция из пространства X в пространство Y, то вектор x из X обозначается как dx, вектор y из Y обозначается как dy - то есть просто пишется буковка d перед символом. Можно рисовать черточку со стрелочкой, можно писать d.
В одномерном случае функция D просто умножает свой аргумент на некоторое число, а именно на производную.
Поэтому, окончательно:
Пусть f дифференцируема в точке x, D - её дифференциал, f'(x) - её производная.
Запись dy = f'(x)dx означает всего-навсего, что y = D(x).
Сама функция D при классическом подходе не называется никак.

В учебниках матана сначала следуют современному подходу и дифференциалом называют функцию D, а потом, в объяснении геометрического смысла, внезапно начинают следовать классическому подходу и дифференциалами называют прообразы и образы D.

Когда говорят, что дифференциал - главная линейная часть приращения функции, имеют в виду классический подход.
137 7488
>>7484

>f(a+h)-f(h) = D(h)+ o(h)


>Запись dy = f'(x)dx означает всего-навсего, что y = D(x).


Может, f(a+h)-f(a) = D(h)+ o(h) и dy=D(x)?
138 7490
>>7488
У тебя на пике определение пустого множества, а не единицы. "Тот объект, которому не не не принадлежит тот объект, который ему не не принадлежит". Несколько "не" подряд появляются из-за кванторов. Если их аннигилировать, то у тебя на картинке написано всего-навсего, что пустое множество - это "тот объект, которому не принадлежит тот объект, который ему принадлежит".
139 7504
>>7502
Если хочешь пиздеть про нестандартный анализ, то потрудись пролистать хотя бы одну книгу про него. Ты охуеешь. Тени чисел, каждое множество содержит нестандартный элемент, регулярные по Слейтеру программы - вот что такое этот анализ. Нестандартный анализ - наркомания похлеще той, что несёт местный любитель Мартин-Лёфа. О нестандартном анализе нельзя думать как об анализе, в котором есть актуальные бесконечно-малые, поскольку а) на самом деле их там нет и б) это не анализ, это беспощадная теоретико-множественная наркомания.
140 7509
>>7453
Стремление к нулю - не есть ноль так же, как и стремление к супербольшому числу не есть супербольшоечисло, стремление к нулю - это супермаленькое число, но оно чисто мнимое, как и супербольшое.
Бесконечность и бесконечно малое - это направление стремления, а не конкретное число.
Это мой ИМХО. Докажешь обратное?

Тем более если там абсолютно точно получается не ноль, а число большее нуля, тогда вообще о чём разговор?.
141 7510
>>6879 (OP)
Всем привет. Подскажите где посмотреть, как решаются подобные задачи.

Построить граф и график соответствия 8 между множествами X и Y. Указать упорядоченные пары соответствия.

X = {1,10,11,15} Y ={2,8,10,16},S X<=Y
142 7511
>>7456
>>7509
А вот я лично считаю, что 0,00000000000000...00000000000000001 - это не 0, ты же утверждаешь, что это ноль, как сам думаешь кто из нас прав?
Почти равно, т.е. примерно равно или равно - это абсолютно разные вещи.
Разумеется, если дельта икс не равна нулю, тогда производная - чисто примерная штука и чем больше дельта икс, тем больше погрешность. И настанет время, когда из-за этого кто-то сильно пострадает, а может это будет глобальная катастрафа и только после этого людишки начнут разбираться в чём проблема, но они 100% её не найдут и свалят всё на первого попавшегося. А не найдут они её, потому что они попросту идиоты.

Я тебя не спрашиваю, что пишут люди в своих книгах, которые не способны думать, а способны лишь передавать информацию дальше тем, кто на это способен. Я спрашиваю у тебя твоё личное мнение. Ты, видимо, полагаешь, что миллиарды людей не могут ошибаться, но поверь, они ошибаются. Они ошибаются каждую секунду, они ошибаются каждое мгновение, они ошибаются прямо сейчас.
И десятки млрд и сотни млрд людей тоже будут ошибаться и тебе несказанно повезёт, если ты найдёшь реально думающих людей среди всего этого сброда. Это как иголку найти в стоге сена.
143 7513
>>7509
>>7511
Идите читайте, что такое предел, короче, а то у вас каши в головах.
144 7514
>>7513
Что такое предел понятно из одного только названия. У вас в голове не каша? Если вы не в состоянии 3-4 простейших логических шага проделать хотя бы на листочке?
145 7527
p × q ≠ q × p, где p и q — им-
пульс и координата квантовой частицы
Почему? Почему не работает коммутативность?
146 7529
>>7527
Векторное произведение не коммутативно в принципе.
sage 147 7581
>>7527
потому что операторы в общем случае не обязаны быть коммутативными.
148 7618
Посоветуйте годных калькуляторов которые все могут. А особенно решать такие нехорошие штуки, как интегралы и пределы с решением желательно.
149 7619
>>7618
wolframalpha
mathematica
matlab
python+scipy+numpy
150 7620
Как найти площадь между кривой, определяемой параметром, и касательной к этой кривой? Вопрос в том, что я не могу связать параметрическую функцию и простую в интеграл, чтобы найти площадь. ( по отдельности могу, не могу понять, что делать с ними в одном интеграле вместе, y1-y2)
151 7622
кто-нибудь знает как доказать, что сумма пределов равно пределу суммы, если дано 2 последовательности, у первой предел конечное число, а у второе бесконечность.
Я знаю, что когда у обеих последовательностей пределы конечные числа, то надо воспользоваться свойством модулей, а тут что делать?
152 7629
>>7620
Возьми разность интегралов вместо интеграла разности.
>>7622
Пусть x->a,y->inf, Е и е - произвольные числа больше 0.
С какого-то места у>Е-а+е, |x-a|<e откуда x>a-e.
Тогда x+y>E.
153 7631
Есть тут картофаны, которые могут пояснить за ядра Бергмана?
154 7634
Как сделать мою программу математически проверенной? Типа, как l4 доказанная математическая надежность. Как сделать такое? Куда копать?
155 7639
>>7634
Ну почитай как L4 доказывали и другие проекты.
Так то там какой-нибудь Coq небось использовали. По Coq есть годная книженция Software Foundations.

Если сам принцип: формализуют язык реализации, в пруверах типа Coq/Agda/Lean доказывают, что всё норм и опционально генерят код из доказательства.
mathematics-wallpaper-2560x1600.jpg800 Кб, 2560x1600
156 7643
Ребята я хочу Java + Математика при её помощи, это хороший выбор?

И что посоветуете?
157 7645
>>7643
Почему именно Джава? Если ты хочешь изучать конкретно математику, то тебе не стоит распылять свои силы ещё на программирование, потому что много времени будет уходить на велосипеды. Используй математические пакеты (MatLab, Wolfram Mathematica). Если ты хочешь всё и сразу, то лучше Python тогда.
158 7646
Коль стали говорить про яп, то поясните за R, что такое и с чем едят.
159 7648
>>6888
Еще же Арнольд, да и Вербит говорили, что эти задачи/тесты хуйня, чем больше у человека мат. бэкграунд тем сложнее ему решать подобные задачи, если это не его сфера интереса.
160 7649
>>7643
Почему не C++?
161 7650
>>7648
Есть ссылки?
162 7652
>>7645
А Maple что?
163 7656
>>6887
Ну и говно, особенно вопрос про города. При чём здесь интеллект вообще?
164 7661
сап матач, помоги найти теоремы о разложении многочлена на неприводимые множители над полем С и над полем R (с доказательствами). Весь инет перерыл, нихуя не нашел толкового.
165 7662
>>7661
Основная теорема алгебры.
166 7665
Поясните за игру Жизнь на клеточных автоматах. Я пока лишь играл. Нарисовал свастику, а оно разрисовало на два противоборствующие лагеря. В чем суть вообще?
167 7667
>>7443

> они посадили меня в комнату 101


>101


Так бы и сказал, что К.О.З.У в Убежище на инженера сдал. То же мне, ОДИНОКИЙ СТРАННИК.
168 7668
>>7665
Гугл тебе на что?
169 7673
Привет форум!
Изучаю понятие полного дифференциала функции нескольких переменных. Вопрос почему f'x(x0 + Odx, y0 + dy, z0 + dz) = f'x(x0,y0,z0) + alpha
Куда пропали дельта приращении??
170 7674
зачем нужен матан можете объяснить? я уже заебался сидеть и какие то доказательства строить, у меня ещё и не получается нихуя и как то все выглядит тупо и вообще тошнит уже от пределов, скучно надоело. кажется, что это все абсолютно неприменимо нигде за пределами самого матана. чё за хуйня
171 7675
>>7674
Задачник Бермана полистай, там много прикладных задач после вычислительных примеров бывает. А без пределов ты далеко не уплывёшь в дифференциальном и интегральном исчислении. Применение сами пределы имеют как минимум в определениях и несобственных интегралах.
172 7676
Подскажите где посмотреть, как решаются подобные задачи.

Построить граф и график соответствия 8 между множествами X и Y. Указать упорядоченные пары соответствия.

X = {1,10,11,15} Y ={2,8,10,16},S X<=Y
173 7678
>>7673
f'x(x0 + Odx, y0 + dy, z0 + dz)
и
f'x(x0,y0,z0)
— просто два числа.
Когда есть два числа x и y, всегда можно написать x=y+c, где c — какое-то число (это называется разность).
174 7719
>>7673
Бери x=y-Δx и не парься.
175 7726
Не могу понять зачем нужен квазиполином, что это, и каким боком он к ДУ
176 7733
>>6879 (OP)
Почему предел считается описываемым значение функции в месте где значение функции неопределено?
177 7746
Посоветуйте справочник по математике, чтобы аж от счета начиналось. Сейчас читаю Выгодского "Справочник по элементарной математике", вроде даже не плохо. Но думаю, может, что-то посоветуете новое и с нескучными картинками.
178 7748
>>7746
Гугли Математика в техническом университете. Её тут часто советуют.
179 7749
>>7746
серия Справочная математическая библиотека, а >>7746 хуйня
Бля, у меня дежавю
180 7750
>>7748 - хуйня*
Фикс
181 7751
>>7746

>чтобы аж от счета начиналось


>с нескучными картинками.


Учебник за первый класс
182 7752
Зачем нужно деление с остатком? И почему до некоторого времени его активно используют, а потом нет?
184 7757
>>7746
Читаю 5 томник, П. С. Александров, А. И. Маркушевич, А. Я. Хинчин - Энциклопедия элементарной математики

Люто доставляет элементарность объяснений, хотя не всегда очевидно.
185 7762
как доказать непрерывность логарифма на области определения кто-нибудь знает? я пытался гуглить и смотреть на math.stackexchange, но не въезжаю. объясните плиз
186 7764
>>7757
Я что-то не могу нагуглить электронную версию. Есть в pdf?
188 7766
>>7765
Спасибо. И что, в самом деле понятно?
189 7770
>>7762

толстовато. это же аналитическая функция
190 7776
Можно ли сегодня заниматься математикой самостоятельно, как ученый-одиночка, ученый-любитель. Хватит ли тех самых "карандаша и бумага" + добротного компьютера, или, современная сложность вычислений требует использования суперкомпьютеров всяких, а в будущем возможно и компьютеров основанных на квантовых технологиях. Т.е. теперь недостаточно просто заниматься математикой в своё удовольствие? Нужно обязательно получать высшее образование, а с ним доступ к вычислительной технике?
191 7778
>>7776

>Нужно обязательно получать высшее образование?


Да.

>Доступ к вычислительной технике


Это вообще последнее, что тебя должно волновать при изучении математики.
192 7780
>>7778

>Да.


Т.е. концепция "ученого одиночки", каким был Пьер Ферма, например, который не был математиком, а работал гос. служащим, или Декарт и множество других математиков, примеры научного пути которых были описаны в книге Белла "Творцы математики" умерла?
В смысле, невозможность заниматься математикой как наукой не имея высшего образования упирается во что? В бумажку, статус, или современную сложность вычислений и необходимости в дорогом оборудовании для этих вычислений?
193 7781
>>7780
Посмотри годы их жизни.
Надеюсь, после этого придет понимание абсолютной тупости твоего вопроса.
194 7783
>>7781

>понимание абсолютной тупости твоего вопроса.


Скорее на меня накатывает понимание тупости твоего ответа.
Во что упирается необходимость получения ВО для математика? Прочти книгу Белла "Творцы математики" где как раз и рассказывается, что сотни математиков не имели ни то, что высшего, а вообще никакого университетского образования, где попутно расписывались их биографии.
Сейчас же я спрашиваю о современном положении математика и во что упираются его проф. возможности
195 7787
>>7783
Вижу, с тобой бесполезно вести дискуссию.
Ты задал вопрос с целью получения конкретного ответа.

Если ты так уверен в своих силах, то вот тебе либген, скайхаб, вот оверфлоу - пиздуй и учись.
196 7788
>>7783
Ты ограничен только уровнем своего интеллекта.
Если ты охуенно умён, а ты должен быть таким для того, что бы заниматься математикой на высшем уровне, то получение ВО, если оно вообще нужно, для тебя не будет проблемой.
И это весьма очевидно.
197 7789
Из "учёных-самоучек" вырастают Рыбниковы и Говоровы.
Ничего против этих ребят не имею, но это, видимо, не то, к чему ты стремишься
и фермаисты :)
198 7790
>>7788

>Ты ограничен только уровнем своего интеллекта.


Пфффф, а кто же не ограничен?
Или ты о том секретном способе, для достижения >2 Grot?
Звучит, как "Мы те, кто мы есть на самом деле!"
199 7791
>>7790
,
Само выскочило
200 7796
>>7776
Нет, нельзя. Тебе для входа в исследовательскую деятельность нужны будут, во-первых, преподаватели, которые направят твои усилия в нужное русло и дадут базовые (не все, а только базовые!) знания предмета, а во-вторых - материалы, над которыми надо работать. Ты не можешь с потолка взять и решить какую-нибудь проблему, потому что ты банально не знаешь, что решать. За сложные же задачи браться сходу - путь либо в никуда, либо к научному фричеству.
201 7805
>>7783
Чтобы заниматься математикой, строго говоря, формальное образование не нужно. Но есть две большие проблемы. Первое, если не иметь подходящих дипломов получать деньги за занятие математикой (и наукой вообще) очень сложно. Второе, стать нормальным математиком в изоляции от других математиков, судя по всему, просто невозможно; завязывание же таких контактов в процессе получения нормального математического образование происходит довольно естественно.

Подводя итого, технически возможно (я даже знаю один живой пример), но по большому счету это метод создать массу дополнительных проблем самому себе.
202 7806
>>7805

>я даже знаю один живой пример


Гельфанд, Рамануджан?

>этот метод создать массу дополнительных проблем


Мягко говоря, а на деле это, как осознанное самобичевание. Нахуя изолировать себя, когда в коллективе и с научруком куда проще и лучше?!

Школьник под впечатлением от попсовой книжки решил наебать системутм и стать математиком самостоятельно. Не он первый, не он последний
Только зачем здесь так много развернутых ответов? Достаточно всего двух слов: "Не выйдет."
203 7807
>>7806

>Гельфанд, Рамануджан


ну и цыганские подвалы туда же
204 7810
почему замечательные пределы нельзя использовать в функции?
205 7811
>>7810
в сумме?*
206 7814
>>7806

>Нахуя изолировать себя, когда в коллективе и с научруком куда проще и лучше?!


А форумов и этой доски хватит? Просто далеко не школоло или студота уже, работаю, и второе формальное образование получать некогда. Математика интересует в разрезе computer science - теория типов, теория категорий, матлогика и прочая абстрактная поебень.
207 7817
>>7807
Почему Принстон называют цыганскими подвалами?
k-theory.jpg60 Кб, 600x543
208 7822
Где я ошибся?
Ответ не сходится с вольфрамом
209 7823
Сяп, двач, завтра экзамен, короче, задача такого плана "Составить уравнение плоскости, проходящей через прямую l: (x-1)/2 = (y+2)/-3 = (z-2)/2, перпендикулярно к плоскости L:3x+2y-z-5=0", объясните что куда втыкать?
210 7824
Как запомнить признаки делимости? Может подскажите стишок? Или это надо зазубрить?
211 7825
>>7824
Прочитать их доказательства
212 7836
>>7822
так сложноо ответить?
213 7839
>>7836
Что вольфрам показывает? Вообще, что ты как маленький, каждое действие посмотри отдельно и всё.
214 7840
215 7841
>>7840
Это и есть твой ответ, просто не упрощённый.
216 7842
>>7841
мне просто кажется,что где то ошибся в упрощении
лан
217 7844
>>7806

>Гельфанд, Рамануджан?


Я же говорю живой (вроде ему слегка за 40). И не то, чтобы человек очень выдающийся, но вполне компетентный.

>Нахуя изолировать себя, когда в коллективе и с научруком куда проще и лучше?!


Ну я и сказал, что в изоляции стать математиком невозможно, что означает, что ходить на научные семинары и фактически иметь научного руководителя/руководителей нужно будет в любом случае.
>>7814

>А форумов и этой доски хватит?


Нет.
218 7845
>>7844
Если вопрос, на который ты отвечаешь, вообще имел какой-то смысл, то он, вероятно, подразумевал следующее:
Возможно ли в ходе самостоятельных занятий (без университета) выйти на тот уровень, когда тебе уже понадобится

>ходить на научные семинары и фактически иметь научного руководителя


Ответ к данной формулировке вполне очевиден: да.
219 7849
>>7733
Бамп вопросу.
220 7850
>>6879 (OP)
Почему все что дальше прикрепленного треда представляет из себя помойку из вхоренов, тапалогий и петушений на ЛЖР?
221 7852
>>7850
Потому что проще пиздеть про тусовочку, чем изучать и обсуждать математику.
222 7853
>>7823
у тебя есть плоскость, берешь нормаль, решаешь
вуц.png2 Кб, 316x70
223 7857
АНОН, ХЕЛП
224 7858
225 7862
26 лет, высшего образования нет.
2 года назад влюбился в математику.
Понимаю, что о математической научной карьере в 26 лет можно забыть. Но, появилось лютое желание стать преподавателем математики, чтобы так сказать вовремя направить школьё или студентоту на путь истинный (если у таковой имеется интерес к математике).
Кто как считает, сильный ли зашквар будет в 26 лет поступать в университет учиться на педагога по математике и информатике?
226 7863
>>7862
Какая разница, в каком возрасте начинать карьеру?
227 7864
>>7863

>Какая разница, в каком возрасте начинать карьеру?


Есть ведь такие понятия, как "молодой ученый", "соискатель", которые сильно зависимы от возраста. Да и в карьерном росте больше питают надежд, перспектив и симпатий к молодым специалистам.
228 7865
>>7864

> Есть ведь такие понятия, как "молодой ученый"


Есть еще понятие как "просто ученый". Возраст - оправдание.
229 7866
вот у нас есть определение производной:
lim h->0 (g(x+h)-g(x))/h
Можно значит выразить разницу значений функции:
g(x+h)-g(x)=(d(g(x))/dx)*h+o(h), h->0
что означает о малое и зачем она нужна?
230 7867
>>7862
По бурбакам математик становится слишком старым после достижения 50 лет. У тебя есть 24 года, пиздуй учить.
231 7869
>>7865

>Есть еще понятие как "просто ученый". Возраст - оправдание.


>>7867

>По бурбакам математик становится слишком старым после достижения 50 лет. У тебя есть 24 года, пиздуй учить.


Спасибо за мотивацию
232 7870
>>7862
Без паники.
Да, время упущено. Ты не будешь столь же эффективен, как если бы в 14 пошел в матшколу и дальше без перерывов занимался бы математикой (получив степень как раз примерно в свои 26). То есть, скажем, получить медаль Филдса надежд нет.
Но это не означает, что все безнадежно. Знаю несколько человек, которые перешли в математику заметно за 20. Надо признать, что звезд с неба они не хватают. Но при этом они весьма мотивированы и способны делать вполне содержательную работу. Я бы сказал, что они заметно лучше той части математиков, которые в какой-то момент, по-существу, потеряли интерес к математике, но почему-то не сменили карьеру.
233 7871
>>7870
Ну до Филдса у него еще 14 лет есть. Сложно конечно думаю вообще без шансов, без вузовской базы что-то там пытаться. Но после время будет, кто его знает, вдруг.
234 7872
>>7871

>думаю вообще без шансов


Ровно это я и имел ввиду.
235 7873
>>7870

>способны делать вполне содержательную работу


Только это (а ещё и огромное удовлетворение от занятия математикой) мне и нужно. А цели получить медаль Фильдса я совсем не приследую.
>>7870

>Без паники.


Спасибо
236 7875
>>7873

> а ещё и огромное удовлетворение от занятия математикой


Это самое главное. А медали, награды, премии, это все наживное, и от возраста я думаю не зависит, но не особо.
237 7877
>>7866
что такое о малое зачем его пишут? разве диффернециал не будет больше разницы функций всегда зачем там что то прибавлять
 .png9 Кб, 388x522
239 7880
Решал неравенства методом интервалов и тут вылезло это.
Пикрелейтед. Я совсем не понял как это произошло. Я бы понял если бы в знаменателе осталось 2-х, но тут я совсем в непонятках. Объясните пожалуйста.
240 7881
>>7880
Что тут объяснять, очевидная опечатка/ошибка.
241 7882
>>7881
Вот пидоры, я тут мозг сломал, дело в том, что у них она и дальше там в решении идет. Думал может там какое деление хитрое на -1/2.
242 7883
>>7879
да это то я почитал, зачем его в определении дифференциала писать?
243 7884
>>7883
Чтобы было верное равенство?
244 7885
>>7883
Что значит зачем, раздели всё на h, устреми h к нулю и получи тот предел, что у тебя в начале написан. Что тебе не понятно?
245 7888
Мне 14, посоветуйте хорошие материалы по математике, пожалуйста.
Horen!!htiXWTUYyY 246 7889
>>7888
"Теория множеств" (Николя Бурбаки).
247 7890
>>7888
Учебник за 9 класс
248 7891
>>7888
Афлутова-Устинов и Гашков
249 7892
>>7888
И. М. Гельфанд, А. Шень: “Алгебра”. Весь курс школьной алгебры по 9 класс
Р. Курант, Г. Роббинс: “Что такое математика?”
П. С. Александров: “Введение в теорию групп“
В. Б. Алексеев: “Теорема Абеля в задачах и решениях”
Д. Пойа: “Как решать задачу“
Д. Пойа: “Математическое открытие“
Н. Я. Виленкин: "Рассказы о множествах"
250 7895
Анон, а у тебя, случаем, не завалялось лаконичной методички по численным методам математической физики? Нынче сильные проблемы с концентрацией, поэтому хочется чтоб кратко и по сути. Или таких в природе не существует и надо таки упарываться ноотропами (или чем там для концентрации упарываются)?
251 7897
>>7862

>26 лет, высшего образования нет.


>2 года назад влюбился в математику.


>Понимаю, что о математической научной карьере в 26 лет можно забыть. Но, появилось лютое желание стать преподавателем математики, чтобы так сказать вовремя направить школьё или студентоту на путь истинный (если у таковой имеется интерес к математике).


>Кто как считает, сильный ли зашквар будет в 26 лет поступать в университет учиться на педагога по математике и информатике?


Такая же телега. Только я хочу и матаном заняться, и финансами. В смысле и рыбку съесть и косточкой не подавиться. Но мотивации нет настолько, что уже трижды учинял попытку суицида). Старый, ещё и поступать в 25 на 1 курс со вчерашними школьниками. Гордость и амбиции, ага.
252 7898
>>7897
Хеллер также в НМУ шел, с ним на одном курсе 10-классник был, и нормально всё.
253 7899
>>7862
>>7897
Ебать вы закомплексованные, ишь какие, со школьниками учиться не хотят. Тут не надо ни о чём волноваться, когда вы начинаете заниматься математикой, вы обнуляете свой возраст и перерождаетесь подобно фениксу. Тот, кем вы были до занятия математикой и то существо, каким вы стали после - это два совершенно разных организма, не имеющие ничего общего друг с другом, бренное тело и его возраст перестаёт что-либо значить, только сознание истинно.
254 7900
Из-за математики я чувствую себя тупым. Как исправить?
255 7901
>>7900

> Как исправить?


Больше математики.
256 7902
>>7901
У меня какая-то фобия к математике
257 7903
>>7902
Это пройдёт. Адаптация.
258 7906
>>7903
Я не он. Но вот действительно, хоть институт уже и закончил правда специальность - не чистый математик.

Занимаюсь самообразованием, и вот чувствую какое-то ощущение - что вот это(некоторые вещи для младшекурсников) мне трудно читать и прорешивать,а есть юди, которые это ПИСАЛИ. (хотя, конечно, понятно, что это работа несколько иного толка - компиляция источников, выбор лучшиъ эпизодов и связывание это в одну сюжетную нить)
Есть такая штука что, ВРОДЕ БЫ, я просто тупо быстрее устаю из-за разных не особо серьёзных, но мешающих болезней - и получается медленнее,чем возможно, хотя время сейчас есть "в избытке"(на самом деле его всегда мало... именно из-за того, что медленный). Это только я или многие так?
259 7907
>>7906

> мне трудно читать и прорешивать,а есть юди, которые это ПИСАЛИ


Таки разбираться в чужом сложнее, чем писать самому.
260 7918
почему мы из иксов составляем последовательность в определении предела по гейне, там ведь на оси абсцисс x принадлежит R, а ведь множество действительных чисел не счетно, а мы же пытаемся тут вроде как установить биекцию между множеством натуральных и множеством этих иксов, составляя последовательность. что за бред?
261 7919
как мы вообще можем составить последовательность из иксов, которые являются подмножеством вещественных чисел????
262 7926
>>7919
Используя аксиому выбора.
263 7947
Как перестать смотреть на математиков, как на шарлатанов? А то смотришь символы, тут бах-бабах и нет его, поменялось все местами, непонятные фокусы с наперстками и ничего не ясно. Как из первоначальных символов получили конечные? При этом разве что указан способ, который надо заучить, что похоже на трюк.
264 7949
>>7947
Вникать в определения. Математика – наука о семантике/содержании понятий, а не о манипуляциях с символами.
265 7952
>>7947
Стать одним из них.

Ну и научиться в аксиоматический подход и читать доказательства.
266 7954
Докажите, что отрезок не гомеоморфен интервалу.
267 7957
Посоветуйте музыку для обучения.
268 7964
https://ru.wikipedia.org/wiki/Лемма_Ферма#.D0.94.D0.BE.D0.BA.D0.B0.D0.B7.D0.B0.D1.82.D0.B5.D0.BB.D1.8C.D1.81.D1.82.D0.B2.D0.BE
почему в лемме ферма когда рассматривают односторонние пределы стоит нестрогое неравенство, там ведь должно быть просто равенство нулю, нет?
269 7965
>>7957
Никакой музыки во время обучения. Она отвлекает и мешает думать.
270 7971
>>7954
Топология разная. Интервал может быть открыто-замкнутым.
271 7973
>>7971
Я хочу увидеть полное доказательство. Что бы буквально была доказана невозможность построить такой гомеоморфизм.
cd64f38059b26f9f3c3c11c770c28da5.png26 Кб, 513x268
272 7975
>>7964
реально объясните чего я не догоняю.
правосторонний и левосторонний пределы должны быть строго равны, только тогда существует предел, предел это ведь просто характеристика. почему в доказательстве какие то неравенства, односторонние производные в точке совпадают -> производная существует и равна им. там в док-ве левосторонний и правосторонние пределы равны это даже можно в вольфраме проверить или самому решить и увидеть. откуда неравенства
273 7978
>>7975

> там ведь должно быть просто равенство нулю


> правосторонний и левосторонний пределы должны быть строго равны


Это то, что нужно доказать.
Неравенства получаются из того, что f(x_0) максимум и знака (x-x_0).
274 7979
>>7965
Даже какой-то нью-эйдж?
276 7981
>>6879 (OP)
Ребят, сюда несколько раз вбрасывали список тип
с темами которые должны изучаться в школе и на 1 2 3 4 5 курсах

Забыл фамилию составителя, киньте его пожалуйста
277 7982
>>7978
Так само значение в обоих этих равносторонних пределах будет 0. Значение правостороннего и левостороннего предела 0, это не надо доказывать мы же можем это просто посчитать. Какая разница с какой стороны подбираться, значение предела будет одно и то же. Понятно, что там слева во всех точках кроме максимума значение производной больше нуля, а справа меньше. Но если рассматривать конкретный предел с стремлением икса к максимуму неважно ведь с какой стороны подбираться значение будет одинаковым, почему именно в односторонних пределах стоит неравенство?
278 7983
>>7980
Странное исследование. Понятно же, что поп-музыка будет отвлекать. Там же много слов, всяких "ах-ух-трибиух".
280 7985
>>7984
ВООООООО, спасибо огромное
выручил
282 7987
>>7984
>>7986
Не похуй ли, когда начинать учить математику?
283 7988
>>7987
Еще один. Попробуй в программе "курс" прочитать как "этап обучения" а "мат школьник" как минимальный набор основ, необходимых для развития культуры. Смысл не изменится.
284 7989
>>7988
Как скажешь, бро.
285 7991
>>7982
я кажется понял что имелось ввиду, именно в процессе изменения дельты которая зависит от эпсилона знаменатель будет меньше нуля в первом пределе, числитель понятно больше. Конечная левосторонняя производная конечно будет равно 0. Но нам надо зафиксировать как то тот факт что изменяется значение производной и поэтому поставили неравенства, это конечно правильно но запутывает, лучше бы придумали какой нибудь эпсилон сдвиг конечного предела и написали там строгое неравенство.
286 7995
>>7986
Спасибо за 3-ю ссыль, не видел её раньше.

Прочитал "Бурбаки" - Архитектура математики, сейчас читаю несколько книг, Голблатта - Топосы; Элементарную энциклопедию Александрова; Ван дер Вардена; Ленга - Алгебру; Прасолов - Геометрия. Позднее постараюсь перейти на англоязычную литературу. Пока теоремы не доказываю. Можете расписать, как лучше шагать от алгебры к другим областям?
287 7996
>>7995
Ван дер Варден и Ленг сразу? Читай лучше Ротмана или Роуэна/Айзексона.

>как лучше шагать от алгебры


Зачем тебе к другим областям? 98% математики это алгебра.

>Голблатта - Топосы


Не_нужно.
288 7997
>>7996

>Зачем тебе к другим областям? 98% математики это алгебра.


поддерживаю
именно поэтому не учу матан и функан
289 7998
>>7997
Зря не учишь, матан и функан это тоже разделы алгебры.
290 7999
>>7998
да хуйня это все, завтра ещё ебучая сессия по топологии

мне вообще кажется что матан функан комбинаторику и теовер стоит вырезать из математики, это лженауки
Horen!!htiXWTUYyY 291 8000
>>7995

>Позднее постараюсь перейти на англоязычную литературу.


Нет. Переходи сейчас. Иначе потом будет поздно.

>Пока теоремы не доказываю.


Нужно доказывать с самого начала. Доказательство - главное в математике, что отличает её от всего остального в этом мире.

>Голблатта - Топосы.


Если только тебе сносит крышу с них в хорошем смысле.

>Можете расписать, как лучше шагать от алгебры к другим областям?


Другие области не нужны.

>Прочитал "Бурбаки" - Архитектура математики.


Умница. Продолжай их читать.
292 8001
>>8000

>Доказательство - главное в математике


Ага, после теорем. А теоремы после определений.
Horen!!htiXWTUYyY 293 8003
>>8001
...и в итоге приходим, что самым главным является знак. А так как доказательство в математике строится на допущении их перемещать, приходим к первоначальному выводу.
294 8004
>>8002
>>8003
Ни к чему мы не приходим, долбоебы. Гротендика не читали, Манина хоть почитайте.
8CF70EC0-5AB0-4F81-ABD2-AC588B28635Emw1024mh1024s.jpg87 Кб, 1024x768
295 8005
Кстати, если серьёзно
На этой борде сидят реальные математики?
Или только студенты 1-2(3) курса?
296 8019
>>7996

>Роуэна/Айзексона.


Не видел таких. Можешь линкануть?

>>8000

>Нет. Переходи сейчас. Иначе потом будет поздно.



Почему?
297 8021
>>8019

>Не видел таких. Можешь линкануть?


Isaacs, Rowen, книжки со вловами Algebra в названии.
298 8023
Как понять красоту математики, мать твою?
299 8024
>>8004

>Гротендика не читали, Манина хоть почитайте


Первый, я так понял, обсуждаемые вопросы затрагивает в Récoltes et semailles, а у Манина где читать то? А то я французского ещё не знаю.
300 8026
>>8024
Математика как метафора.
math.ru/lib/files/pdf/manin.pdf
Можешь еще про мистицизм почитать у вербита, в конце треда о топологии ссылка.
301 8028
Я прочитал калькулус томаса второй раз.
В третий раз буду его читать и попутно решать задачки из него и демидовича.
После третьего раза я смогу вкатится в ландау-лившица или нужно еще что то для погружения в математику?
302 8029
>>6879 (OP)
Всем привет, надо понять фильтр калмана. Нужно для оценки величины параметра, зависящего косвенно от двух других величин. Интересует сигма точечный фильтр калмана(Unscented Kalman filter).
Кто что может посоветовать почитать, или пояснить?
303 8030
http://biblio.mccme.ru/node/5780
Это перевод, или на русском выходит первой?
304 8033
>>8005

>На этой борде сидят реальные математики?


>Или только студенты 1-2(3) курса?


В какую из этих двух категорий попадет человек, если он по возрасту приват-доцент или профессор, а по уровню знаний – второкурсник в смысле вербицкого?
Ты понимаешь, что это идиотский вопрос, в принципе?
305 8034
>>8033
Лучьше ответь про фильтры калмана плиз, а не на вопросы какого то еблана.
306 8053
Нужно ли уметь находить интегралы вручную? Дрочить всю эту таблицу, приемы, если есть ПО которое это делает намного лучше? Как они вообще устроены? Я про неопределенные, также как и человек решают или есть какой-то универсальный топорный алгоритм, который сложен для человека, но прост для компа? Определенные понятно, что можно тупо перебором решить с заданной точностью.
307 8055
>>8053

>Нужно ли уметь находить интегралы вручную


Да.

>Дрочить всю эту таблицу, приемы, если есть ПО которое это делает намного лучше?


Есть ПО, но лучше таблицу дрочить конечно.

>Как они вообще устроены


Кто они?

>Я про неопределенные, также как и человек решают или есть какой-то универсальный топорный алгоритм, который сложен для человека, но прост для компа?


Алгоритм Риша.
308 8065
Сап двач. На работе у меня появилась такая интересная задача.
Что мне известно:
На пациента идет ток, я могу его регулировать и я знаю сколько тока идет на него.
Я знаю диапазон сопротивлений пациента от 50 до 1000 ом

Что я не знаю:
Напряжение которое идет на пациента

Что мне нужно найти:
Мощность идущую на пациента.

Может быть можно как то приблизительно определить без непосредственного измерения напряжения?
309 8066
>>8065
Хули этот вопрос делает в /math?
P=I^2*R
Screenshot20170115-135902.png212 Кб, 1920x1080
310 8068
Анон, пытаюсь дрочить машинное обучение, но не тяну математику.
Что конкретно надо читать, чтобы смочь в нейронные сети и прочее?
на пик - хайкин, нейронные сети, полный курс. Нихуя не понятно
311 8071
>>8068
Вам же целый номерной тред сделали https://2ch.hk/pr/res/900102.html (М) там в шапке ссылки на все нужное. Что вы к этому Хайкину привязались, я сам у этого шизика нихуя не понимаю даже по абсолютно ясным для меня алгоритмам типа карт Кохонена. Возможно, все дело в замечательном русском переводе, в оригинале я его не читал. Математика в нейроночках простейшая - матанализ и теорвер. Если еще по дискретке что-то читал - вообще заебись.
312 8074
>>8028
Бамп
313 8080
>>8028
>>8074
Это шутка такая? Если ты школьник, то советую начать с анализа Шварца. А про дважды прочитанный полтора тысячный текст для умственно бессильных будешь потом байки рассказывать, как бездарно тратил свое время в детстве.
314 8081
>>8080

>то советую начать с анализа Шварца


Ты там не охул ли а щенок? Сам-то с чего начинал? Небось, како-нибудь Ильина-Поздника.

> А про дважды прочитанный полтора тысячный текст


Повторение мать учения. По-хорошему, учебник через месяц-два, после того, как прочитал, следует ещё раз прочитать, по ходу доказывая теоремы. Базарю, результат будет хороший, да и после первого чтения можно многое упустить.
315 8082
>>8080

>анализа Шварца


Хорошо пошутил, молодец. Когда закончишь ебаться в жопу тапалогиями покажешь мне где в физике применяются теории множеств
316 8084
>>8082
Причем тут теория множеств?
317 8085
>>8082

>где в физике применяются теории множеств


Масса - мера множества.
318 8086
Как сессия?
319 8087
>>8081
Если бы я начинал с говна, то я был бы только рад, если бы мне на это указали и подсунули что-нибудь хорошее взамен. И где было написано, что повторять изученный материал --- плохо?
320 8088
>>8084
Потому что анализ шварца на множествах и топологиях в первых ста страницах.

>>8085
Кулстори бро.
321 8089
>>8087
Ящик посчитал?
322 8090
>>8086
Не очень.
blob163 Кб, 260x343
323 8091
Есть у кого на русском?
324 8092
>>8088
Про множества там страничек двадцать-тридцать, типа что такое отображение, фактормножество и мощность. Топология в Шварце тоже не экзотическая, самые основные понятия и десяток важных теорем. И даже такие базовые понятия находят применение в физике. Например физическое пространство в квантовых масштабах имеет неархимедову метрику. Метрика -- отображение. Для ее определения необходимо определить множество действительных чисел, мощность которых континуальна, а сами они являются классами эквивалентости последовательностей коши рациональных чисел.
Собственно можешь дальше перечитывать талмуды калькулуса для студентов технических специальностей, но это малополезно.
325 8095
>>8092
Нахуя мне квантовая метрика, если ученые подкопченые в говне моченые до сих пор не разобрались какая интерпретация самая правильная и для разных ситуаций применяют кардинально отличающиеся?
Блядь, да они даже с физикой ядра не разобрались какую модель применять и для каждой подзалупной хуйни свою дичную модуль придумали. Оболочечная модель, капельная, статистическая, сверхтекучая, хуясекучая. Только электричество более менее проработали, и то, на основаниях термодинамики и химии, что считается смежными дисциплинами.

Это я к тому что разъяснение нахуй не нужной хуйни шварцем через три пизды колено о приращении функции мне не нужно.

Ну и потом, если бы вся физика стояла на математике получалась бы теория супер-пупер струн, а не физика.
IloQeszzaiQ.jpg449 Кб, 1102x1960
326 8096
Подсажите, зачем во втором номере циллиндрические координаты и как это сделать? Вроде уравнение плоскости написано....
327 8099
>>8095
Нельзя понять теорию относительности, не зная, что такое риманово многообразие, ассоциированная с метрикой связность, тензор кривизны, касательное пространство и еще кучу разъяснений Шварца.
328 8100
необходимое условие существования предела? не могу сообразить в гугле нету
329 8101
>>8100
Последовательность имеет предел тогда и только тогда, когда она фундаментальна.
87461248.png172 Кб, 1488x2000
330 8102
331 8104
>>8029
Бамп поцаны, погибаю ваще, как переложить википедию на практические вещи? хеелп
332 8106
>>8099
Зачем она мне нужна, если для того чтобы применить её на практике нужно оборудование которое в гараже не соберешь?
Да, приборы которые нужны для измерения всяких расщеплений сверхтонкой структуры можно собрать на токарном станке с помощью кузьмича, но прибор этот больше ни на что не сгодится.

>>8102
Уже все теории старой математики доказал чтобы в множества вкатится?
333 8107
>>8106
Какие еще теории старой математики? Теория множеств меня не интересует. Ты меня с кем-то перепутал.
334 8108
>>8106
Это тебе не нужно, то тебе не нужно. Ладно, расскажи, зачем тебе нужно прочесть Ландау-Лифшица.
335 8109
Назовите самый топовй ВУЗ в СНГ и России для математика
336 8111
>>8071
Спасибо, ты боженька, анон.
Добра тебе ❤
337 8119
>>8109
Зависит от области, причём очень сильно.
Чистая математика -- матфак ВШЭ, однако только 3 или сколько-то там направлений, если интересных тебе среди них нет, то МехМат МГУ.
Прикладная -- МФТИ (все факультеты сильные и заточены под математику, но под разную; но наиболее ориентированный на математику факультет -- ФУПМ, ну или ФОПФ, кафедра проблем теорфиза, но там больше абстрактной математики), СПбАУ, ВМК МГУ
Прикладная экономическая -- совместный бакалавриат ВШЭ и РЭШ. Очень сильный факультет, не подумай, что экономика -- штука гуманитарная.
338 8120
>>8095
Ну и не учись нахуй, еблан. Пиздец теперь, может вообще физику не учить никому, пока сам Господь Бог с небес не спустится и не скажет, что за закон двигал им, когда он творил вселенную? Интерпретации -- это блядь физический смысл, для прикладных задач, в которых используется квантмех, вообще поебать, сколько там миров и существует ли близкодействие и прямые причинно-следственные связи.
339 8121
>>8106
Значит, нужен только теормех, электричество да термодинамика? Без остального проживём? Только вот нахуй тебе теормех, электричество да термодинамика, если ты всем этим сразу в гараже одновременно не займёшься? Надо что-то одно выбрать.
340 8122
>>7998
Когда это матан и функан стали разделами алгебры? Определения тебе не завозили?
Матан (в широком смысле, то бишь включая функан, инфинитезимальный анализ и блабла) -- такой же самостоятельный раздел математики, как геометрия (включая топологию), алгебра, матлог и дискретка (и то, комбинаторика с теорией графов тоже самостоятельные области). Все они изучают математические объекты и связи между ними, и все объекты разной природы.

Кстати, очень интересно, что же за 98%? Считаем как алкоголь -- по объёму? Так зайди на arxiv.org.
По важности и глубине открытий? Так уже давно алгебраическая геометрия является доминирующей темой, ибо нужна одновременно (правда, в разных областях) в теорфизе и робототехнике. Да и премии последние годы дают геометрам в основном.
341 8123
>>8087

>И где было написано, что повторять изученный материал --- плохо?


>А про дважды прочитанный полтора тысячный текст для умственно бессильных

342 8126
>>8119
А какие конкретно направления есть на матфаке ? Что-то мне не найти. Сайт у них конечно не очень дружелюбный
343 8131
>>8108
>>8121
Чтобы все охуели как я могу из двух ржавых железок и мотка медного провода собрать магнитогидродинамический генератор на парах цезия питаемый дедовской буржуйкой 18 века.
Как можно показать небольшими преобразованиями, очевидно что мне в хуй не уперлись множества и квантмех. Разве что соберусь собрать фузор чтобы показать тянучке какой я умный, радиацию обуздал.

>>8120
Да и математику можно не учить уже. Компютеры же есть, они все посчитают.
Пруф ми вронг.
20170116132142.jpg1,7 Мб, 3264x2448
344 8133
Решите пожалуйста, срочно!!!
blob266 Кб, 2153x1204
345 8135
>>8133
Опоздал, ну да ладно.
346 8136
>>6879 (OP)
Хз где спрашивать. Какой сейчас самый норм учебник по логике на русском который можно купить в бумажном виде? вообще логике, не только математической.
347 8140
>>8136
колмогоров-драгалин
348 8143
>>8126
бамп
349 8147
>>8123

>А про дважды прочитанный полтора тысячный текст для умственно бессильных


>для умственно бессильных

350 8148
>>8131
Ну тогда ты понимаешь, что тебе нахуй не нужно читать некоторую часть томов? И что весь этот срач, только от того, что ты толком не можешь объяснить, что тебе от нас нужно?
Поэтому можешь показать своей тянучке этот тред, чтобы она охуела от твоей тупости.
351 8149
>>8148
Ты дурачок?

>Я прочитал калькулус томаса второй раз.


>В третий раз буду его читать и попутно решать задачки из него и демидовича.


>После третьего раза я смогу вкатится в ландау-лившица или нужно еще что то для погружения в математику?



>Ответы: >>8074 >>8080

2144523.png167 Кб, 757x648
352 8150
>>8131

>мне в хуй не уперлись множества


Ты наглухо отбитый? Посмотри содержание главы про множества в этой книге, чего ты к ним так приебался? Как можно этого не знать? Эта глава читается за максимум час-два.
353 8152
>>8150
И это мне говорит человек который на вопрос "мне хватит таких знаний чтобы понять такой то учебник" простой вопрос на который есть два ответа "да, хватит" и "нет, не хватит. нужно еще знать Х " взрывается говняной бомбой с криками "*ШТО ЗА ХУЙНЮ ТЫ ЧИТАЕШЬ??????? ЧИТАЙ НАРМАЛЬНОЕ"
354 8153
>>8149
Если вкатиться значит прочесть все эти книги, то ответ нет >>8102

А так весь конфликт в том, что тебе говорят что этот раздел ты не поймешь, ты говоришь что он тебе не нужен и мы тут гадаем, какие тома тебе не всрались, очень весело на самом деле.
355 8154
>>8152
А что может значить ответ "твой учебник --- говно"? Только то, что не можешь ты нихуя понять физику.
356 8155
>>8153

>этот раздел ты не поймешь, ты говоришь что он тебе не нужен


Нужно быть пинкертоном чтобы понять что мне не нужно читать том про квантмех, после того как я сказал что том про квантмех я читать не буду?
Выяснили уже, что квантмех без теории множеств и топологий я не пойму, я сказал что он мне не нужен. Сказать хватит или не хватит томаса для всего остального так сложно?
357 8156
Может запилите кулстори как вы стали математиками, какие отношения с ней были в детстве и т.д?
358 8157
>>8154
Давай напишешь мне в каких условиях существуют гладкие решения уравнений навье-стокса, раз ты физику знаешь.
359 8158
>>8155
Потом выяснилось, что и главы теории полей по теории относительности ты не осилишь.
360 8159
>>8158
Векторные пространства у томаса херово описаны, но понять их я смогу.
361 8160
>>8155

>без теории множеств


Блядь, ты про главу в Шварце? Без нее ты не поймешь, что такое действительное число.
362 8161
>>8159
Векторные пространства изучаются в курсе линейной алгебры.
Эти понятия >>8099 ты игнорируешь?
363 8162
Назовите топовые вузы для математиков в США
364 8163
>>8155
Почему ты такой необразованный и при этом агрессивный? Ты даже не знаешь, что наивная теория множеств это считай логика, как ты собрался думать о любой физике без логики?
>>8131

>Чтобы все охуели как я могу из двух ржавых железок


Ясно, физика и математика тебе не нужны. Пиздуй в какой-нибудь слесарный раздел.
>>8131

>Да и математику можно не учить уже.


Верно, тебе можно не учить.
365 8164
>>8091
Мне тоже интересно.
366 8166
>>8163
Ну с другой стороны у этой агрессии имеются основания, все таки мы разрушаем чью-то мечту. Это область еще неизведанной природы великовозрастных превозмогающих личностей, пытающихся заняться математикой. Кто все эти люди, которые толпятся на вводных лекциях в нму? К чему они стремятся? Имеются ли подобные явления в других областях науки и в других странах?
367 8167
>>8161
Это не мешает им быть в книжке томаса.
Плюс я векторы/тензоры дополнительно читаю у беклемишева и еще пары авторов, чтобы посмотреть на них со всех возможных сторон.
Эти понятия, очевидно, я не смогу применить на практике. Знать их мне сейчас не нужно. Может потом когда я смогу запускать ракеты в космос.

>>8163
Логика это не математика, так же как физика это не математика. Математика изъясняется языком логики, физика изъясняется языком математики, но это не означает что принцип изъяснений это сама суть изъясняемого.
Если ты считаешь иначе, реши тогда с помощью математики логическую задачку у которой только ОДИН правильный ответ,который ты должен обосновать

Если почесать угубку за ухом, он начнет довольно шипеть. Если угубок довольно зашипит, то молоко поблизости скиснет.
a. если не чесать угубка за ухом, то молоко поблизости не скиснет;
b. если почесать угубка за ухом, молоко поблизости скиснет;
c. молоко вдалеке никогда не скисает от чесания угубков.

>>8160
Вот из за этого я и не хочу теорию множеств даже километровой палкой трогать.
Июо она задается тупейшими вопросами уровня "почему слово лес не звучит как собтвенно лес, скопление большого количества рядом растущих деревьев?"
368 8168
>>8167

>Логика это не математика, так же как физика это не математика.


Верно, физика это умение склеивать две ржавых железки, тебе не нужна математика, иди в слесарный раздел.

Ландау-Лившица не читай, там задаются тупейшими вопросами о том как найти экстремумы функционалов, симметриями каким-то и прочей бесполезной абстракщиной, просто заучи законы ньютона и формулы для расчёта электрических цепей, т.е. школьной физики тебе хватит.
369 8169
>>8168
Атлищная аргументация моей неправоты. Все так понятно и обоснованно разложил по полочкам.
370 8170
Всем привет.

Немного срочно пишу, поэтому (уж извините ФАГи не читал)

Кто знает годный материал по вычислению интегралов с помощью вычетов?

В гугле не забанен, но из более-менее годного находил лишь скучные лекции какого то советского профессора из которых ничего не понятно.

студент средней тупости, раздолбай, завтра экзамен
14816610550313.jpg52 Кб, 542x520
371 8171
>>8170

>завтра экзамен


удачи

>Кто знает годный материал по вычислению интегралов с помощью вычетов?


Первые несколько ссылок в гугле ведут на сайт(где есть примеры) и на форум какого-то вуза, где рассматриваются более сложные случаи
372 8173
>>8167

>не хочу теорию множеств даже километровой палкой трогать


>Июо она задается тупейшими вопросами


Ты не знаешь какими вопросами она задается.

В Ландау-Лифшица после Томаса не вкатится.
373 8174
>>8171
Хех, не помешает
mathhelpplanet вроде норм, все по полочкам. Но это скорее справочник, там самое нужное без примеров и подробностей
А я искал скорее гайд
вероятность.jpg325 Кб, 1280x720
374 8175
Анончики, помогите с теорией вероятности
совсем забыл. Можете кинуть решение с объяснением?
375 8176
>>8169
Я тебе совет даю по твоему оригинальному вопросу.

Ты хочешь клеить ржавые железки, Л-Л это курс теоретической физики. Зачем тебе он/его кусок? Просто иди клеить ржавые железки в гараже, формулы по ходу гуглить будешь.

Если ты настаиваешь, что тебе нужен первый том Л-Л, то можешь читать. Вывод уравнений Лагранжа (и, наверное, ряд других результатов) придётся принять на веру не то чтобы он там давал сколь-либо полные выводы anyway, потому что там нужно знать вариационное исчисление, которое опирается на понятие функциональных пространств богомерзские МНОЖЕСТВА пробрались и сюда и близости точек в таких пространствах о боже, почему свитой Ландау не защитил нас от ереси ТОПОЛОГИИ. Не дай бох ещё придётся вникать в основы функционального анализа глубже, там уже палка никакой длинны не поможет. Может даже Шварца придётся полистать.

Получится, что ты вместо заучивание законов Ньютона заучишь уравнения Лагранжа и узнаешь какие бывают дифуры с произвольным лагранжианом для каких-то базовых ситуаций классической теоретической механики. Лично мне не понятно зачем это тебе для склеивания двух ржавых железок.
376 8178
>>8173

>Ты не знаешь какими вопросами она задается.


Опять обоснование моей неправоты на высоте.
Гомер и Аристотель плачут кровавыми слезами, и не от того что ебут друг друга в жопу, а от твоей аргументации.

Блесни умом что ли, покажи и распиши как распределятся импульсы между двумя телами, если одно из тел имеет несравненно меньшую массу чем другое в случае их столкновения.

>>8176
Тобишь по твоему нужно каждому обязательно вшивать в мозг при рождении теорию множеств с топологией, а теорию бесконечно малых и объяснение производной как "скорости" изменения значения функции печатать на латыни в книжках о истории темных веков?
К чему я это? Ты уверен на все сто, что предмет изучения топологии с множествами нельзя, ну вот совсем нельзя, от слова никак, выразить через другие понятия? Калькулус томаса и стюарта тогда что такое и для чего придуманы? Тупым инженерам проектирующим дамбы, дома и ядерные реакторы?
Или если объясняют математику доступным языком то это уже не математика?
И множества ваши я трогать не хочу из за вас долбоебов. Ибо все кто хоть как то знают множества ведут и себя и поступают как имбецилы с синдромом дауна. Тот же ящик ебучий посчитать не могли три треда с помощью вольфрама.

И да, ты можешь что то другое кроме курса ландау-лившица предложить?
377 8179
>>8178

>Опять обоснование моей неправоты на высоте.


Там над выделенной тобой строчкой есть еще одна, в ней написана причина. Вот еще раз. >не хочу теорию множеств даже километровой палкой трогать
Как ты можешь знать что-то о теории множеств, если ты не можешь осилить элементарнейшие ее определения? Пересечение, объединение, биекция, классы эквивалентности, мощность. Там в той главе материал посильный ученику начальной школы.
21435312.png6 Кб, 642x139
378 8180
>>8178

>Тот же ящик ебучий посчитать не могли три треда с помощью вольфрама.


Это связано с тем, что это выдуманная тобой процедура, несуществующая в природе.
379 8181
>>8178

>Калькулус томаса и стюарта тогда что такое и для чего придуманы?


А в ведении что-нибудь про это написано?

>предмет изучения топологии с множествами


Давай еще раз про множества. Разберемся с ними, хотя с топологией почти то же самое. Есть какие-то общие определения, которые включаются в вводные главы почти всех учебников для младшекурсников, это просто базовые понятия алгебры, геометрии и анализа. Иногда в введении упоминается, что предполагается, что читатель знаком с ними. Все люди писавшие учебники математики последние лет сто знают эти определения. И Ландау знали их, и Лифшиц знал их.

>все кто хоть как то знают множества ведут и себя и поступают как имбецилы с синдромом дауна


Поэтому это не настоящая причина твоего нежелания пробежать глазами тридцать страниц элементарного текста.

>И да, ты можешь что то другое кроме курса ландау-лившица предложить?


Тебе не нужно знать ни математику, ни физику, чтобы просто применять готовые формулы. Просто задайся целью сконструировать в своем гараже конкретный аппарат. А дальше по-ходу разберешься. Вот чем тебе знание

>как распределятся импульсы между двумя телами, если одно из тел имеет несравненно меньшую массу чем другое в случае их столкновения


поможет примотать скотчем друг к другу две железяки?
14701732916530.png9,9 Мб, 3442x2456
380 8182
>>8156
Бамп вопросу!
381 8184
>>8178
Я не понимаю почему теория множеств у тебя вызывает такой ужас и почему ты её ставишь в один ряд с топологией. Ничего сложного в ней нет и она возникает естественным образом как только ты начинаешь заниматься математикой.

>Или если объясняют математику доступным языком то это уже не математика?


>выразить через другие понятия


Без теории множеств? Нет, нельзя, она неявно используется и в книжках по калькулюсу.

И в книжках по калькулюсу не объясняют математику. В книжках по калькулюсу учать... считать, что неудивительно, ведь с латыни это именно так и переводится — "расчёт". Ты читаешь гайд по расчётам, принимаешь всё сказанное там на веру и применяешь как тебе хочется. Если ты хочешь убедиться, что всё действительно так или получить общее виденье предмета вместо коллекций трюков по расчётам, то тебе нужно доказывать теоремы. И тут уже нужно явно говорить о множествах.

>Ты уверен на все сто, что предмет изучения топологии с множествами нельзя, ну вот совсем нельзя, от слова никак, выразить через другие понятия


Во-первых, топология довольно естественная наука, говорит она о том, что такое "близко", "окрестность точки". Т.к. это центральные понятия анализа — топология сразу же полезна тут. Можно использовать эпсилон-делта формализм как завещали прадеды, но даже тут тебя ожидают страшная теория множеств и квантор существования/всеобщности.

Во-вторых, отвечу в общем: есть два пути. Первый это построить башню определений, в которых твои утверждения будут достаточно простыми. Второй это использовать "доступный язык" и простые определения, но все утверждения будут со всё возрастающей сложностью. Определения это способ прятать сложность и делать рассуждения доступными для мозга человека.

>И да, ты можешь что то другое кроме курса ландау-лившица предложить?


Для каких целей конкретно? Для общего понимания классической физики? Или всё таки чтобы научиться строить шлоебень в гараже?
382 8186
>>8179
Немного википедии, немного разговоров из тредов, немного из дхду.
Ну и сообразить что между нулем и единицей бесконечно много чисел труда не составляет. Так же как и сообразить что между нулем и двойкой чисел тоже бесконечно много, но их ровно в два раза больше чем в первом случае.
Но это заслуга не калькулюса а учебников лузина.

>>8181

>A great discovery solves a great problem but there is a grain of discovery in the solution of any problem. Your problem may be modest; but if it challenges your curiosity and brings into play your inventive faculties, and if you solve it by your own means, you may experience the tension and enjoy the triumph of discovery.



Ты не ответил на вопрос и ушел в демагогию и в то что всем все очевидно. Если всем все так очевидно, почему до сих пор преподают и выпускают учебники в дедовских стилях, с приращениями, пределами и т.д.
Ведь если все реально так просто, нужно с прогрессивных множеств все начинать преподавать всем и везде.

Ну раз ты знаешь лучше меня какая у меня причина, то не буду переубеждать.

В приматывании скотчем ничем. А для определения ударной вязкости какого то сплава ниобия с танталом очень даже. Плюс, формулы иногда после небольших преобразований, реально небольших, могут сильно облегчить вычислять ту же ударную вязкость.

>>8184
Если она так естественно возникает, почему арнольд от нее плевался? Или это тут врётё в тредах было? Не то чтобы я сильно арнольда знал, но описание на вики и профильных ресурсах прочитать смог.

Где там в том же томасе все на веру заставляют принимать? Там есть доказательства с примерами, без всяких хитровыебаных виляний.
А если на веру все заставляют принимать, то почему растянули все на тыщу страниц?
Ну и как множества помогут доказать что два плюс два, таки да, четыре?
Учти, я не говорю что теория множеств хуита. Может даже я не прав и она таки да для успешных стильных и молодежных математиков, но она слишком сложна для того чтобы доказать что два плюс два равно четыре. Это все равно что пытаться промышленным гидравлическим прессом забить в доме гвоздь для вешалки. Можно конечно, но зачем?

Так то получается что топология прям везде применяется, раз она такая базовая и вообще изобретение покруче чем изобретение умножения. Даже два плюс два без топологии нельзя посчитать. Или можно?

И при чем тут пути?

Для практического применения знаний, я же уже писал.
382 8186
>>8179
Немного википедии, немного разговоров из тредов, немного из дхду.
Ну и сообразить что между нулем и единицей бесконечно много чисел труда не составляет. Так же как и сообразить что между нулем и двойкой чисел тоже бесконечно много, но их ровно в два раза больше чем в первом случае.
Но это заслуга не калькулюса а учебников лузина.

>>8181

>A great discovery solves a great problem but there is a grain of discovery in the solution of any problem. Your problem may be modest; but if it challenges your curiosity and brings into play your inventive faculties, and if you solve it by your own means, you may experience the tension and enjoy the triumph of discovery.



Ты не ответил на вопрос и ушел в демагогию и в то что всем все очевидно. Если всем все так очевидно, почему до сих пор преподают и выпускают учебники в дедовских стилях, с приращениями, пределами и т.д.
Ведь если все реально так просто, нужно с прогрессивных множеств все начинать преподавать всем и везде.

Ну раз ты знаешь лучше меня какая у меня причина, то не буду переубеждать.

В приматывании скотчем ничем. А для определения ударной вязкости какого то сплава ниобия с танталом очень даже. Плюс, формулы иногда после небольших преобразований, реально небольших, могут сильно облегчить вычислять ту же ударную вязкость.

>>8184
Если она так естественно возникает, почему арнольд от нее плевался? Или это тут врётё в тредах было? Не то чтобы я сильно арнольда знал, но описание на вики и профильных ресурсах прочитать смог.

Где там в том же томасе все на веру заставляют принимать? Там есть доказательства с примерами, без всяких хитровыебаных виляний.
А если на веру все заставляют принимать, то почему растянули все на тыщу страниц?
Ну и как множества помогут доказать что два плюс два, таки да, четыре?
Учти, я не говорю что теория множеств хуита. Может даже я не прав и она таки да для успешных стильных и молодежных математиков, но она слишком сложна для того чтобы доказать что два плюс два равно четыре. Это все равно что пытаться промышленным гидравлическим прессом забить в доме гвоздь для вешалки. Можно конечно, но зачем?

Так то получается что топология прям везде применяется, раз она такая базовая и вообще изобретение покруче чем изобретение умножения. Даже два плюс два без топологии нельзя посчитать. Или можно?

И при чем тут пути?

Для практического применения знаний, я же уже писал.
383 8188
>>8186

>с прогрессивных множеств


Теории множеств уже сто с лишним лет. Более того 1 том Зорича по Матану начинается именно что с наивной теории множеств.

> почему арнольд от нее плевался?


Арнольд плевался от Бурбаков, а это совершенно другое.

>топология прям везде применяется


Почти везде, другое дело, что это порой нелегко заметить.

>Даже два плюс два


Ну два плюс два это больше к алгебре

>Для практического применения знаний


Для практического применения знаний надо было в ПТУ идти, там научат как балки скреплять и прочую залупу. Ну или на инженегрофак.
384 8189
>>8186

>сообразить что между нулем и единицей бесконечно много чисел труда не составляет. Так же как и сообразить что между нулем и двойкой чисел тоже бесконечно много, но их ровно в два раза больше чем в первом случае.


Во-первых, нет, действительных чисел там одинаковое число. Во-вторых, речь про главу в Шварце, про тридцать страниц, а не про то, чтобы бы глубоко разбираться в тонкостях теории множеств. Можешь про это пояснить? Почему все имеющие отношения к физике и математике люди знают содержание этой главы, а ты не хочешь? Три десятка страниц элементарных понятий, которые позволят тебе не ходить под себя на таких элементарных вопросах, как количество действительных чисел между нулем и единицей, и нулем и двойкой.
385 8192
>>8136
Непейвода
386 8193
>>8186

>Ты не ответил на вопрос и ушел в демагогию и в то что всем все очевидно.


Он не в демагогию ушёл, он тебе как раз пояснил ситуацию очень просто. "Страшная" теория множеств даётся в первых главах учебников для первокурсников. И применяется она для всего. Как что-то можно доказать не зная теорию множеств я себе слабо представляю. И как в Томасе выглядят доказательства без теории множеств боюсь себе представить.

>Ну и как множества помогут доказать что два плюс два, таки да, четыре?


Какой-то ебанутый пример. Множества помогут доказать, что для каждого действительного числа d, существует такое e, что выполняются такие-то условия и тем самым определить предел, производную и т.п. Сразу становится понятно, что же такое предел. Понятие функции тоже определяется через теорию множеств и сразу же проясняет суть этого понятия в том числе и в расчётно-прикладных контекстах.

>Если она так естественно возникает, почему арнольд от нее плевался?


Владимир Игоревич Арнольд (12 июня 1937, Одесса — 3 июня 2010, Париж[4]) — советский и российский математик, автор работ в области топологии, теории дифференциальных уравнений, теории особенностей гладких отображений и теоретической механики. Один из крупнейших математиков XX века.

>Или это тут врётё в тредах было?


Не воспринимай слишком близко к сердцу культурные споры, тут много тонкостей в контекстах. Арнольд, конечно, прекрасно знал математику, и не был просто калькулятором интегралов.

> Даже два плюс два без топологии нельзя посчитать


Калькулюс для этого тоже не нужен.

>И при чем тут пути?


Я объяснил зачем нужны определения, которые тебе кажутся сложными.

>Для практического применения знаний, я же уже писал.


Не читай Л-Л, читай гайды по сборке шлоебеней в гараже и по физике (механике) конденсированных сред, термодинамике, химии, электротехнике и всём том, что тебе посоветуют на профильной слесарной доске/форуме.
387 8194
>>8186

>Ну и сообразить что между нулем и единицей бесконечно много чисел труда не составляет. Так же как и сообразить что между нулем и двойкой чисел тоже бесконечно много, но их ровно в два раза больше чем в первом случае.



Потрясающе, дальше можно и не разговаривать.
388 8198
Поцаны, так чо с фильтрами Калмагорова мне делать? Тут никто не знает ниче о них? Куда мне тогда с ними? Пару ответов бы, дальше уж сам смог бы...
389 8200
Какова связь между аксиоматикой евклидовой геометрии и аксиоматикой R? Если выкинуть 5ый постулат и аксиому паша можно построить изоморфизм?
234342.png2 Кб, 397x39
390 8204
391 8205
>>8204
изоморфизм моделей поищи
2017-01-17053854.png120 Кб, 732x107
392 8207
Хелп
393 8209
Filth.jpg153 Кб, 640x320
394 8213
Анон, как искать платиновые учебники по нужным темам?
Смотреть первые ссылки в гугле по запросу "влияние пива и сметаны на аппетит пиявок pdf"?
Или есть получше методы?
Сейчас вот ищу нормальный учебник по линейному погромированию (оптимизация, симплекс-метод, вот эта хуйня).
На английском тоже подойдут.
Но не могу читать древние учебники с хуёвыми сканами, да и вообще со СКАНАМИ
395 8214
>>8207
Гугли определитель матрицы.
14642756561941.jpg132 Кб, 1920x1080
396 8215
Имя пожалуйста, я забыл
397 8217
>>8213
Лучше спроси у знающих людей, которые занимаются этим. Зайди на dxdy, mathoverflow.
398 8218
>>8215
Николай Вавилов
399 8220
>>8213
В разделе же куча списков платиновых учебников в шапках.
400 8222
>>8213
Васильева двухтомник.
Vopros.jpg64 Кб, 850x476
401 8223
Привет.
Несколько вопросов касающихся использования терминологии.

1. Как только в математическом тексте встречается слово "норма" или "скалярное произведение", то это значит что речь идёт о каком-то векторном(линейном) поле?
(то есть есть какое-то поле скаляров и множество векторов)

И нет другого использования этих слов?
(кроме неверного)

2. Какие свойства(упорядоченность, непрерывность, ...), если можно так сказать, используются для определения функции SIN? То есть её можно определить на произвольном множестве? И что в ней такого особенного?

Заранее спасибо.
Надеюсь больше стану понимать благодаря вашим ответам.
402 8224
>>8215
Лучше спроси у знающих людей, которые занимаются этим. Зайди на dxdy, mathoverflow.
403 8225
>>8223
1) Да.
2) Разложение в ряд тейлора. Это наиболее общее определения, позволяющее взять синус о матрицы, например. Это функция тесно свзянна с комплексными числами, от сюда идут корни её геометрического и матанного смысла.
404 8226
>>8224
Платиновый ответ.
405 8229
>>8213
Алсо, когда мне не нравится скан, я его сам допиливаю в scantailor'е, нихуя сложного, зато глазам приятно.
406 8235
>>8225
Благодарю.

Небольшое уточнение по 1) пункту:

Но при этом не обязательно это векторное пространство будет на R(действительными числами)?

По 2) пункту не могу ничего уточнить, так как мало понимаю. Спасибо за направление.
407 8240
>>8235

>Но при этом не обязательно это векторное пространство будет на R(действительными числами)?


Да.
408 8242
>>8235
Это при том, что "норма" и "скалярное произведение" вещественно-значные функции.
SEdOOZOJZwQ.jpg11 Кб, 605x182
409 8245
Ребят, хэлп пожалуйста с одним вопросом

Вроде бы очень простой вопрос, но я могу понять, какой ответ правильный
410 8246
>>8245
Думаю АВ, но я не уверен
411 8247
>>8246
Если не уверен, значит чего-то не понимаешь.
412 8248
>>8247
А что тут понимать то? Просто если а из R, то a может быть = 1, 1 - действительное число
413 8249
Бля в тхреде ваще есть математики? Чот никто нихуя не может ниче решить.
414 8250
>>8248

> Просто если а из R, то a может быть = 1, 1 - действительное число


И в чём проблема?
415 8251
>>8240
Добра тебе.

Буду дальше ковыряться.
416 8252
>>8250
Получается ответ AБВ?
Сорян, просто завтра на экзамене вообще ни разу нельзя ошибиться, поэтому чет параною
417 8253
>>8252
Ответ АВ.
Б не подходит, так как 2*1=2, ок? Что такое подпространство, вспомни, какое условия должны выполняться?
418 8254
>>8188
Но ты не будешь отрицать что математика и там и там применяется?

>>8193
Зачем мне доказывать то что и так очевидно? Зачем доказывать что интеграл это площадь под кривой бесконечно большого количества бесконечно малых трапеций?

И вы так и не обосновали существование калькулусов без теории множеств.
419 8255
Простите, больше некуда обратиться с этим трешем

Не могу дорешать уже больше месяца эту очень простую задачу, из за того, что не могу понять, как найти базис пересечения подпространств
В интернете есть несколько примеров решений, но мой препод отвергает их

До уравнения

a1x1 + a2x2 + a3x3- b1y1 - b1y2 = 0
[Из которого по совету препода я получил
a1x1 + a2x2 + a3x3- b1y1 = b1y2
И расширенную матрицу (x1 x2 x3 b1 | b2)]

Все правильно, дальше не понимаю что делать, прочитал уж е все что можно, с остальными темами в алгебре и матане все отлично
420 8257
>>8255
На скринах мое последнее решение, но оно врядли правильное
421 8258
>>8255
Просто скажите, что делать после этого шага

>a1x1 + a2x2 + a3x3- b1y1 - b1y2 = 0



И все
422 8261
>>8255
Что такое d1, d2, d3?
Вектор, который лежит в пересечении ты обозначил как a1x1+a2x2+a3x3, он же равен b1y1+b2y2.
Вот ты получил систему уравнений

>a1x1 + a2x2 + a3x3- b1y1 - b1y2 = 0


Как её решать? Да всё тем же ступенчатым методом, собственно, так ты и начал. После этого тебе нужно просто посчитать этот самый a1x1+a2x2+a3x3 и всё.
423 8262
>>8254

>Зачем мне доказывать то что и так очевидно


Во-первых, "очевидно" не значит верно. Во-вторых, не всё очевидно и вообще это субъективное понятие. В-третьих, уточнения понятий и доказательства теорем углубляет понимание.

>И вы так и не обосновали существование калькулусов без теории множеств.


Калькулюсы — гайды по тому как считать определённые вещи. И то там ТМ используется явно и неявно. Потому что это базовая вещь.
424 8264
Взял Сканави из вашего списка и понимаю, что это слишком сложно для меня. Вот почему можно рассказывать о натуральных числах, а потом сразу же шибануть альтернативные системы счисления?
234645432.png60 Кб, 801x181
425 8265
>>8254
Свой провал ты комментировать не собираешься?

>Ну и сообразить что между нулем и единицей бесконечно много чисел труда не составляет. Так же как и сообразить что между нулем и двойкой чисел тоже бесконечно много, но их ровно в два раза больше чем в первом случае.



Кстати, вот снимок третьей страницы одиннадцатого издания.
Страница номер три. Назови номер своего уникального издания калькулуса, в котором этого нет.
426 8267
>>8254
Как ОПРЕДЕЛЕНИЕ может быть очевидным, это как вообще?

мимошёл
427 8275
>>8264
Альтернативные помогают глубже взглянуть на десятичную
428 8276
>>8262
Ну хорошо, вот докажи мне что объем конуса одни треть высоты на площадь основания.

>>8265
Не собираюсь. Что там еще комментировать если неправильно?

Вот эта книжка.
http://www.aazea.com/book/calculus-early-transcendentals-8th-edition/
Написана просто и понятно с обсасыванием.
Хуйни уровня "сечение множества верхнего класса содержит число меньшее чем числа сечения множества нижнего класс" тут просто нету. Как и нету того что читаешь, вроде все понятно, а потом постепенно непонятнее и непонятнее, пока совсем станет нихуя не понятно, даже если пять раз перечитать две страницы.
429 8277
>>8264
Альтернативные это какие? Двоичные восьмеричные?
Так у человека то пальцев на которых удобно считать десять а если вспомнить про суставы, то можно даже до шестидесяти досчитать двумя руками и не сбится. Дальше сам.
430 8280
>>8277
Если палец принять за бит, то можно считать до 2^10
431 8282
>>8276
Не ебу, что такое сечение множеств, что такое верхний, нижний класс и подобная ебола. В Шварце этого нет. Что ты на это скажешь?
По твоей ссылке нужно регистрироваться для скачивания и чтения, я этого делать не буду.
На скрине из прочитанного тобой дважды учебника выписаны определения теории множеств, как ты это можешь прокомментировать?
432 8283
>>8280
Ну давай, досчитай мне до 739 на пальцах и не запутайся.
433 8284
>>8276

>Ну хорошо, вот докажи мне что объем конуса одни треть высоты на площадь основания.


Это называется посчитать объём конуса и с чего я тебе буду что-то доказывать или считать? Тебе уже выдали советов, иди читай книги, не испытывай терпение.
14586987175980.gif78 Кб, 857x875
434 8285
Расскажите про разделы математики. На этом пике все есть, или вы бы добавили, изменили что-нибудь?
435 8286
Дело недели тренировок
436 8287
437 8290
Аноны, а я могу здесь задать вопрос по задачке с параметром, которая у меня не получается?
Снимок.PNG1 Кб, 231x31
438 8291
>>8290
У меня там вроде бы как квадратное тригонометрическое уравнение с параметром, но я, решая это уравнение, каждый раз получаю новый ответ. В общем, я уже запутался и не знаю, что со мной не так.

Я не прошу решить это задание, я прошу натолкнуть меня на верный ход мыслей.

Условие такое: при каких p уравнение имеет корни?
439 8292
Вы вроде нормальные ребята, обсуждаете что-то. Но как мне роботяге с завода понять математику до уровня эллиптических кривых?
440 8294
>>8276
При попытке прочитать происходит переход на вредоносный сайт, но я нашел шестое издание.
http://www.vnua.edu.vn/khoa/fita/wp-content/uploads/2014/10/stewart.pdf.Chuvien.gui_.pdf
Вот что было в аппендиксе (скрин 1). Это все вынесено в конец, чтобы слабые студенты (ты) могли себе помочь, что указано в самом начале книги (скрин 2). Можешь еще поискать в этом документе cartesian product, one-to-one correspondence может там еще что-нибудь есть из теории множеств. Ну ладно, собственно я трачу на все это силы в надежде, что ты выдашь какой-нибудь перл уровня >>8194
Но вряд ли это уже произойдет, так что вот еще кое-что. Детей десятилетия травят в школе математикой, которая им не нужна. Такие курсы калькулуса, которые ты показываешь --- это дальнейшее продолжение этого процесса. Детям на пальцах объясняют то, к чему они в жизни более никогда не притронутся. Такое вот объяснение. Там в аппендиксах имеется глава с доказательствами на одиннадцать страниц. То есть меньше процента книги занимают доказательства, остальное время детишек приучают считать. Причем эти доказательства уровня упражнений в обычных учебниках математического анализа.
54634.png18 Кб, 293x303
441 8295
>>8294
Второй скрин не про этот аппендикс, но ссылка на него есть на 12 странице, там же противные для тебя теоретико-множественные картиночки есть, они там повсюду. Зачем я трачу на это время?
442 8296
>>8285
Картинка дебильная абсолютно. Современная математика делится примерно на:
1) Арифметику и миксование арифметических структур с другими (гипотезы Вейля, программа Ленглендса, теоремы Делиня, ...)
2) Геометрию и геометризацию (пучки, гомологии, кобордизмы, геометрическая логика, ...)
3) Схематизацию и отчищение от догм теоретико-множественного мышления (группоиды, категории, схемы, топосы. мотивы, ...)
4) Деформацию и квантование (нелинейность, некоммутативность, квантовость, q-объекты, ...)
5) Рефлексию теорий над самими собой (классификационные теории, неподвижные точки, монструозные модели, элементарные/неэлементарные классы, ...)
443 8297
>>8296
дед, не уходи, дай свою почту или телегу
444 8298
>>8291
1. Как геометрически (в координатах y,p) выглядят решения уравнения y^2 +py-p^2 + 1= 0?
2. а если y = sin(x)? (т.е. -1 <= y <= 1)?
445 8299
>>8296
А есть текст поподробнее на эту тему?
Хотя бы второй уровень классификации подробнее или чтобы гуглилось, а то в 4) как-то совсем абстрактно получилось: "деформация", "нелинейность"?
446 8300
>>8299
4 вполне конкретные вещи: noncommutative topology/geometry/probability/functional analysis/homotopy/differential geometry, quantum groups, тысячи их - но дух у них примерно один и тот же, взять коммутативную теорию и попытаться сделать её некоммутативный аналог, за некоторые из этих наук люди филдса пополучали.

Про второй уровень не понял вообще - это геометрия не гуглится что ли?
447 8301
>>8296
Да твой пост не особо лучше.
Гипотезы вейля, этальные когомологии и геометрический ленглендс это не "миксование арифметических структур", что бы ты под этим не понимал, скорее наука про автоморфные пучки и зеркальную симметрию

>пучки, гомологии, кобордизмы


Чего здесь общего? "Топосы, когомологии, к-теория", пиздец, у тебя каша в голове, что ли

>отчищение от догм теоретико-множественного мышления


Отчищение, ага. Ты в курсе, что в теории категорий используется теория множеств, вообще-то? На, почитай
https://ncatlab.org/nlab/show/ETCS

>схемы, топосы. мотивы


Что ты этим хочешь сказать, снова? Мотивы имеют отношение разве что к алгебраической к-теории, которую ты даже не упомянул в своей классификации. По сути наука о линеаризации категории топологических пространств.

>группоиды


Это h-spaces имеются в виду?
В общем, глупость, как по мне. Свою версию классификации я уже давал. Математика это:
а) циклические гомологии, "некоммутативная геометрия", аддитивная к-теория
б) этальные когомологии, гипотезы Вейля, геометрический Ленглендс
в) спектральная геометрия Лури и эллиптические когомологии
г) когомологии Галуа и мотивы воеводского
д) кристаллические когомологии и p-адическая теория Ходжа
>>8297
Ты теперь у всех спрашивать будешь?
448 8302
>>8301
Половина слов - тотальное непонимание всего на свете - уровня "что общего у топосов и когомологий", другая половина - какие-то передёргивания уровня /б/ вида "статья на ncatlab о ETCS доказывает, что теория множеств нужна для категорий". Даже шквариться о дальнейший разговор с тобой не хочу.
449 8303
>>8302
Ну я-то могу тебе объяснить, что такое когомологии, если спросишь более вежливо. Просто перебирание терминов в духе "слон, стол, соль" выглядит не особенно умно.

>передёргивания уровня /б/ вида


Кто что передергивал? Ты здесь пишешь глубокосмысленные фразы про "избавление от догматов теории множеств", между тем, как только дурачок-первокурсник может сказать, что теория множеств там не нужна. Не нравится статья, читай книгу Sets for mathematicians от создателей теории категорий, например.
Намиксовал слов на википедии типа умный, и баттхертит. Не шкварься об разговоры, пожалуйста.
450 8306
>>8300

>Про второй уровень не понял вообще - это геометрия не гуглится что ли?


Вторым уровнем я назвал то, на что каждый пункт можно поделить так же.

>>8300

>4 вполне конкретные вещи: ...


Понял, спасибо.
451 8307
>>8306
Обращайся еще, эксперт с мировым именем, связывающий разные области по наличию в их описании слова "квантовый", всегда рад ответить. Только если ты не зашкваришь его излишне глупым вопросом, тогда не ответит.
452 8309
>>8307
Тебе тоже спасибо за классификацию, няша если это ты >>8301. По ней ничего спросить не могу, т.к. не знаю этих наук. Ну разве что: это просто активные области или достаточно мощные теории, чтобы описать всю современную математику?
453 8310
>>8309
Достаточно общие.
454 8315
Поигрался немного, и понял что a^n-b^n=(a+b)^ (n-1) без коэффициентов. Можно это как то доказать ?
455 8316
>>8315
Ты неправ. При a=b=1 уже неверно.
456 8319
>>8292
Список от опа из шапки глянь.
14838154177970.jpg210 Кб, 1024x770
457 8320
>>8276

> Не собираюсь. Что там еще комментировать если неправильно?


Не прав ты или неправы математики?
458 8323

>>8321


Нет, это не так интересно.
459 8335
>>6879 (OP)
Ктонибудь читал хеллерское поделие? Не совсем догоняю, что от меня требуется в упражнении 1.2
Выведите формулу для эквиваленции:
a <-> b = 1 + a + b
460 8336
>>7649
Напильник для ААА проектов, не нужен
461 8338
Как пользоваться транспортиром? В интернете объяснения усложнены как-то
462 8343
>>8338
В чём проблема? Главное центром приложить к той точке от которой будешь угол откладывать
463 8348
Хм, оказывается я неплохо понимаю математику, просто много пропустил в школе по различным причинам. До этого я учил в разброс, а сейчас стараюсь последовательно. Всем спасибо.
Сова.jpg45 Кб, 500x407
464 8350
>>6879 (OP)
Есть ли книги по вышей математики которые логично объяснены и понятно иллюстрированные. Вот пример книги которую я ищу https://rutracker.org/forum/viewtopic.php?t=5228801 только по вышей и дискретной математики, где все с первого раза стает понятно.

Все остальные книги которые я скачивал их было около 50-70 штук были написаны дегенератами которые ничего сами не понимают, веди если ты не можешь ПОНЯТНО объяснить материл, так чтобы это понял 5 ребенок значит ты сам ни черта не знаешь.
465 8351
>>8350

>Все остальные книги которые я скачивал их было около 50-70 штук были написаны дегенератами которые ничего сами не понимают, веди если ты не можешь ПОНЯТНО объяснить материл, так чтобы это понял 5 ребенок значит ты сам ни черта не знаешь.


Такое может прокатить с грёбанной физикой, но не с математикой. Потому что, у большинства математических понятий не физического смысла, а значит нет и простого объяснения.
Пошел на хуй, ленивый пидор.
466 8352
>>8351

>Потому что, у большинства математических понятий не физического смысла



Непизди, все можно свести к реальному миру и объяснить на простых элементарных примерах, и математика не исключения. Так что иди ты нахуй.
467 8353
>>8352
Сделай это с топологическим пространством.
468 8355
>>8352
Или же скажи мне физический смысл основной теоремы алгебры, червь.
469 8356
>>8350

>дегенератами которые ничего сами не понимают


Ты типичный воинствующий невежда (человек, который винит в своём незнании плохих учетелей, а не свою лень), научно-популярная экспозиция материала не даёт понимания, как бы этого не хотел, это всегда упрощение материала, которые привносит неточности. Фейнман, которого ты тут цитируешь, тоже самое говорил.

Так что либо ты читаешь научно-популярные тексты и получаешь просто какое-то представление о предмете. Либо ты читаешь книги и разбираешься в сути. Научно-популярных учебников не существует, тем более по "высшей математике".
470 8357
>>8353
>>8355

Не сделаю, так как нету понятных материалов чтобы дойти до таких областей математики.
471 8358
>>8350

>если ты не можешь ПОНЯТНО объяснить материл, так чтобы это понял 5 ребенок значит ты сам ни черта не знаешь


Доказательства этого утверждения имеются?
472 8359
>>8356

>человек, который винит в своём незнании плохих учетелей, а не свою лень



Я знаю очем говорю, я пытался вникнуть но иза этих некомпетентных людей которые только и могут что кидаться умными словами а пользы от них нулевая, нихрена непонятно.
473 8360
>>8358

>Доказательства этого утверждения имеются?


Я предоставил ссылку на книгу, это единственная найденная мною книга где ПОНЯТНЫМ языком описывается элементарная математика, так чтобы было интересно и понятно.
474 8361
>>8359
Ну о каких 50-70 книгах идет речь? Ты открывал Зорича или Шварца? На какой странице стало непонятно?
475 8362
>>8360
Это доказывает, что если человек не может объяснить материал так, чтобы это понял пятилетний ребенок, то он сам ничего не знает? Да или нет?
476 8363
>>8361
Вот я открыл Зорича, Математический анализ. С 22 страницы уже пошла непонятная ересь. Книга дно.
477 8364
>>8363
Нет, просто ты тупой и ничего не понимаешь. Тебе не стоит заниматься математикой, иди вон, как местный клован делай из жравых железок приборы, чтобы впечатлить шкур.
478 8365
>>8362
Мне даже тебе доказывать ненужно ведь все и так очевидно, открой любой учебник и посмотри что там написано, легче китайский выучить чем это понять.
479 8366
>>8357
Мудила, ты не можешь это сделатб потому что, эти вещи не связаны с физикой и у них нет физического смысла, ну по крайней мере у второго точно нет.
Безымянный.png211 Кб, 1366x768
480 8367
>>8363
Какая строчка непонятна? Если не это издание, то сделай скрин, разберемся.
481 8368
>>8365
Очевидно, что ты тупой. Зачем ты вообще паришся о матеше и лезешь в эту область?
482 8369
>>8365
Дело в том, что доказательств у этого утверждения нет.
483 8370
>>8350
Кстати, у тебя очень хорошие ПРОЕКЦИИ! Давно я таких не видел, честно. Ты чувствуешь себя тупым дегенератом, но не осознаешь это, и переносишь эти чувства на авторов учебников, типа это не я кретин, а они такие ебланы мне объяснить не могут.
484 8371
>>8359

>Я знаю очем говорю


Нет, не знаешь. Я понимаю, что ты этого не понимаешь, иначе ты бы не был воинствующим невеждой.

И так, как я уже сказал — не существует учебников с цветными картинками про "высшую математику". Огромная куча не самых умных людей разбирается с математикой по книжкам без картинок. Если тебе это не под силу, то очевидно, что дело в тебе.

Читай это:
Математическая грамотность:
https://www.amazon.com/Mathematical-Proofs-Transition-Advanced-Mathematics/dp/0321797094/
https://www.amazon.com/Naive-Set-Theory-Paul-Halmos/dp/1781394660/
Velleman "How to prove it"

Сам калькулюс:
https://www.coursera.org/learn/single-variable-calculus
Зельдович Я.Б., Яглом И.М. "Высшая математика для начинающих физиков и техников"

Если ты проявишь упорство и после долгих раздумий и рефлексии над материалом тебе будет непонятен Зельдович, то просто смирись и забей. Это учебник для ПТУшников.

до дискретке ничего не посоветую, разве что книжку по алогритмам Кормэна "Алгоритмы: построение и анализ"
22.png290 Кб, 1900x943
485 8372
>>8367
Вот обвел красным. Сразу пошли сложные примеры, не объяснено почему это так и зачем, нету графических примеров с картинками, просто в книге влепили что нибудь и разбирайся почему где и зачем.
486 8373
>>8372
Проиграл в голос.
487 8374
>>8372

>Математическая грамотность:


>https://www.amazon.com/Mathematical-Proofs-Transition-Advanced-Mathematics/dp/0321797094/


>https://www.amazon.com/Naive-Set-Theory-Paul-Halmos/dp/1781394660/


>Velleman "How to prove it"


Вот тут расскажут про этот язык.
488 8375
>>8370

>Ты чувствуешь себя тупым дегенератом


Раньше чувствовал. Но после того как посмотрел видео с этого сайта http://interneturok.ru/algebra/7-klass и как понятно объясняют. Я понял что не я тупой дегенерат а тупые те кто не может понятно объяснить.
489 8376
>>8372
В первом прямоугольнике выделены утверждения, первое из которых доказывается далее по тексту. Тебе понятно, что означает первое утверждение? Дополнение объединения равно пересечению дополнений.
Выдели теперь импликацию во втором прямоугольнике, которая тебе не понятна.
490 8377
>>8375
Ты семиклассник?
491 8378
>>8375
Нет, как раз, это ты тупой дегенерат. Нормальные люди читают без дополнительных разъяснений и жвачки для мозгов. И если тебе надо такое разъеснение, как на сайте, то ты либо быдло, либо школьник.
492 8379
>>8372
Если тебе не хватает базовых знаний для того, чтобы что-то понять, то это не значит, что авторы учебников тупы.
493 8380
>>8376
Я словами так не могу воспринять информацию, мозг отказывается так воспринимать, только графически с картинками могу понять. Ведь непонятно с какимы сущностными работают множества с примеров 2 - 3- 4, если с числами, то почему в книге ради примера не написано и графически не отображено.
494 8381
>>8377
Нет мне 25. Но в школе учился на 2. Сейчас захотел немного понять. Так ка занимаюсь программирование хотелось бы вкатится в игровую разработку а не делания сайтов.
495 8382
>>8380
https://ru.wikipedia.org/wiki/Множество
Держи, картинко-дебил.
496 8383
>>8379
Базовую часть множеств я хорошо знаю, но то что дальше, уже теряю логическую нить.
497 8385
>>8382
Там в картинках простые примеры, я это понимаю. Но там где сложные выражения, без картинок и числовых массивов.
498 8386
>>8380
Ты понимаешь, что такое дополнение объединения множеств?
499 8387
>>8386
Да понимаю.
mattred2.png89 Кб, 900x800
500 8388
>>8385
Эта картинка не теряет своей актуальности.
В Зориче кратко написано про множества, и предполагается, что ты их знаешь.
501 8389
>>8387
А что такое пересечение дополнений множеств?
secrt3.png65 Кб, 487x854
502 8390
Анон, если ты хочешь занятся созданием игр, то ты понимаешь, какая тебе для этого нужна математика? Чем ты конкретно хочешь заниматся? Движком, текстурами, звуком и т.д. ?
503 8391
>>8383
Ладно, давай так, на каком конкретно шаге у тебя возникает непонимание? Доказательство же совершенно элементарное.
множества задания.png77 Кб, 1032x781
504 8392
>>8391
В книге есть математические элементы которые мне незнакомы.

Вот еще пример деграданского образования, проходил на сайте https://stepik.org/course/Дискретные-структуры-83/syllabus но на задании застопорился и было попыток 20 сразу пошли сложные задания.
Здесь более наглядный пример чем в книге. Красным обвел то что я считаю правильным, но на сайте не принимает такой ответ.
505 8393
>>8390
Кстати, этот человек с оп-пика начал публиковать свою книгу. Он кстати во многом прав. Называя свою область деятельности взрослой, он как бы противопоставляет себя гротендиковскому детскому видению. В содержании книги указаны основные главы дифференциального и интегрального исчисления, дифференциальные уравнения, можно будет отсылать к этой книги две-железяки-дибилов и картинко-даунов.
506 8394
>>8390
Нахождения кратчайшего пути, чтобы боты небыли тупыми и адаптировались к условиям боя, а я так понял что нужно знать теорию графов, но чтобы знать теорию графов нужно знать множества и сопутствующие материалы.
507 8395
>>8392
Просто забей на все это, это не твое. Есть же какие-то видео-уроки по математики в видео-играх. Там конкретные примеры, все наглядно. У юнити и унриал точно такие есть. Тебе лучше в /gd сходить. Зачем тебе вся эта муть? Людям чтящим строгость в рассуждениях сложно понять ответ "В книге есть математические элементы которые мне незнакомы" на вопрос "на каком конкретно шаге у тебя возникает непонимание?".
508 8396
>>8389

>А что такое пересечение дополнений множеств?


a= 2, 4 и b = 4, 6 пересечения будет 4.
509 8397
>>8396
Это пересечение множеств.
А пересечение дополнений множеств?
510 8398
>>8394
Нужно просто поискать на хабре темы про это, гуглить незнакомые слова. Математика тебе почти не нужна, особенно доказательства.
511 8399
>>8398

>хабр


За такие советы пора вводить уголовную ответственность. Нормального человека туда вообще пускать не стоит.
512 8400
>>8399
Для людей, не способных самостоятельно доказать законы де Моргана, статьи на хабре могут оказаться полезны.
513 8401
>>8399

>Нормального человека туда вообще пускать не стоит.


>Нормального человека


Поэтому тебя туда и посылают.
514 8402
>>8398
Я так правильно понял? что такие алгоритмы не строятся на математических понятиях а используется программная логика.
515 8403
>>8397
Это когда подмножество содержится в пересечении множеств.
516 8405
>>8403
Какое еще подмножество? Ебать. Давай объясню. Смотри. Есть множества. Есть их дополнения. Пересечение этих дополнений -- то, что нам нужно. Ты сейчас слишком занят? Можешь пройти тестик http://ru.iq-test.cc/ там всего 30 вопросов.
mochizuka.jpg35 Кб, 750x548
# OP 517 8406
Перед перекатом предложу обсудить шапку.
Что в ней можно изменить? Изначально предполагалось, что вы будете принимать в составлении разных списков участие, в частности в списке мемов. Но движуха быстро утихла. Особенно спискосрач. Стоит ли вообще шевелить тему списков? По теме книг, скорее всего нет. Хотя, если кто-то хочет, чтобы я добавил книги в текущий список, я это сделаю.

Дальше, вместо списка статей оставлю ссылку на scihub.
Список с видеолекциями тоже стоит убрать, поскольку никто не предлагает лекций, да и сам список никто не смотрит, если верить статистике на пастебине. Хотя, если кто-то накидает список видеолекций, то он останется.

Далее, в шапку стоит добавить пошаговые калькуляторы.

С мемасами дело обстоит так, я могу заняться созданием pdf-файла, но не смогу самолично описать все мемы. Нужна помощь в материале по ним. Даже если кто-то из вас напишет небольшую статью с каким-то мемом, то это будет хорошая наработка для дальнейшего pdf-файла.

Также жду предложений по шапке, может чего-то не хватает?

Под конец, думаю, стоит сделать единый список сохранённых тредов /math, где будут лежать все заархивированные треды раздела с датами их создания/завершения.
518 8407
>>8406
Хотелось бы, чтобы шапка огораживала от залетных, клевещущих на святые писания анализ Шварца, например, не способных понять определение топологии и утверждающих бесполезность теоретико-множественных обозначений. Можно предложить мем для оп-пика? >>8102
519 8408
На что похоже удовольствие от математики?
# OP 520 8409
>>8407
Эту проблему можно решить, созданием faq с часто задаваемыми ньюфажными вопросами.

>Можно предложить мем для оп-пика?


Да. Хорошо сделал.
521 8411
>>8406

>Что в ней можно изменить?


Добавить минималистичный список по наивной теории множеств, логике и доказательствам (математической граматности, если угодно) (типа такого >>8374).
Почему минималистичный (буквально 3-5 книг)? Чтобы нубы не обсыкались от огромных списков и не впадали в ментальный ступор.

По лекциям, можно добавить такое: https://www.youtube.com/watch?v=ihoATq9jSlQ&list=PLgqZ7cC8KvvZcMjXYLP53SaXCG35hF_E8
522 8412
>>8406
Алсо, в какой-нибудь список, где есть про теорию категорий можно всунуть https://arxiv.org/abs/1612.09375 . Я не читал, но Баез вроде рекомендует.
523 8413
>>8406
Мемы не нужны. Лучше запили ссылки на скачивание учебников
524 8414
>>8405

>Какое еще подмножество?


Подмножество которое содержится в пересечении множеств.

Прошел тест. IQ 97.
# OP 525 8415
>>8413

>Качать книги тут:


>http://libgen.io


Кстати, где-то была ссылка с торрентом из книг. Так же кто-то кидал сайт с множеством книг. Найду, добавлю.
526 8416
>>8414
Смотри, у нас есть какое-то множество, в котором мы рассматриваем подмножества. пусть это будет R, тогда дополнение A это R/A, т.е. все числа, не содрежащиеся в A, но содержащиеся в R. Ясно?
blob12 Кб, 479x174
527 8417
>>8414
Ты случаем не ударялся в голову? Тест же простой.
528 8418
>>8414
Т.е. пересечение дополнений множеств -- это подмножество, которое содержится в пересечении множеств?
529 8419
>>8406
списки оставь, с пдфкой погоди, сейчас паренек допилит форум, обещал прилепить вики, там и будем работать
530 8420
>>8419
что за форум?
531 8421
>>8416
да
532 8422
>>8417
Половина теста сложна.
533 8423
>>8418
Да так и есть.
534 8424
>>8423
нет, это не так.
535 8425
Я вот одного не пойму, если математика от 9 класса до вышей, узко специализирована то зачем ее обучают если не везде она нужна?
536 8426
>>8408
Головные боли.
537 8427
>>8425
Нужно занять чем-то детей, чтобы они меньше всякой хуйни вытворяли. Вот чем может себя занять человек с iq 97?
538 8428
>>8427

>Вот чем может себя занять человек с iq 97?


Играми и просмотрами видео на ютубе.
539 8429
>>8424
Перепутал, думал что дополнения это содержится в множестве.
540 8430
>>8428
Прям как я.

iq 140
541 8431
>>8427
Или чтобы перегрузить мозги и отключить критическое мышления чтобы сделать народ покорным перед властью.
542 8432
>>8430
iq не показатель человек существо ситуативное, есть области в которых человек лучше всего подходит и наоборот, главное найти то к чему есть талант.
543 8433
>>8429
Ладно, меня заебали эти игры в пятнашки, я думал, что ты сообразишь в конце, что пересечение дополнений множеств и дополнение их объединений -- это одно и то же, а это и есть доказываемое у Зорича утверждение. Но ты чересчур туп. ари ви дер чи ))))
544 8434
>>8425
А что такое "неузкоспециализированное знание"? Химия 7го класса узкоспециализирована? А физика 6го?
545 8435
>>8434
Для 99% людей нужно только, 5 класса математики, 4 класса русского языка, 5 класса географии, ну и по мелочи музыкалка физра и т.д, все больше среднему человеку и ненужно.

>Химия 7го класса узкоспециализирована? А физика 6го?


Да это тоже узкоспециализировано и в реальной жизни обычными людьми не используется.
546 8436
>>8435
Я тоже так думаю, тогда непонятно, почему притензии именно к математике 9го-11го класса, а не ко всему комплексу знаний выше 5го.
547 8437
Матан Зорича потому и любят, что он не косноязычен, будто для дебилов писался.
Охуеваю с этого пацана, даже для 7-ми классника он чересчур туповат
548 8438
Совсем странный вопрос. А что значит символика ":=" в математических книжках? Это же не паскаль.
blob44 Кб, 600x320
549 8439
Прошел один упомянутый здесь тест. Собственно, на достижения каких математических способностей с подобным уровнем развития интеллекта я могу рассчитывать? После черепно-мозговой травмы я совсем уже изменился, обленился и прочие -лся.
550 8440
>>8438
По определению.
551 8441
>>8438
Некоторые люди при помощи := определяют тот или иной объект.
552 8442
>>8440
>>8441
Спасибо. Это должно было быть очевидно. Но я сомневался что-то.
553 8452
>>8284
Покажешь что знаешь стереометрию, или интегралы, и как важны доказательства.

>>8294
И опять возвращаемся к вопросу о доказательствах и смысла существования таких учебников.
Если уйти в доказательства то уйдешь в бесконечный цикл в котором нужно все доказывать и доказывать. Одно доказательство стоит на другом, другое на третьем, и так до бесконечности. Это показано у бурбаков.
Учат считать ты так говоришь будто это что то плохое.

И опять же, если множества так просты и элементарны и фундаментальны, почему их не разжевывают в учебниках типа этого?

И вопрос такой для вас обоих, почему в учебниках по математике все прячут за сложной и непонятной аксиоматикой без пояснения этих аксиом? Взять хотя бы того же фихтенгольца. У него сразу же на 12 страничке идет про сечения множеств на классы, хотя до этого определялись операции сложения/умножения/деление/отнимания.
И это не только проблема учебников этого еврейчика. У всех что я качал такая хуйня. Складывается такое чувство что сами авторы слабо шарят в теме и прячутся за сложными терминами чтобы звучало по умному и вызывало уважение.

>>8320
Неправ я, представляешь. Я думал что так, а оно не так. Охуеть вообще, да?!
554 8454
>>8371

>не существует учебников с цветными картинками про "высшую математику"


Are you fucking kiddin me?
555 8455
>>8406
Так там вроде анон хотел вики на своем хостинге поднимать. Может дождаться его, да заливать туда? Лично я на пастебинех.ком вообще никогда ничего не читаю, например. Юзайте https://notehub.org/new , пожалуйста. Алсо, было бы лучше иметь не просто списки, а пошаговую инструкцию по выбору книжки. Ну или хотя бы какое-то подобие туториала.

Вообще, вики надо, конечно.
556 8458
>>8407
Пиздарики, тред для НАЧИНАЮЩИХ захватили олдпердуны которые требуют сделать из треда пристанище для гауссов, лобачевских и римманов.
557 8459
>>8454
А есть манга на русском о школьном уровне?
558 8460
>>8409
Надо про треды и модерацию, кстати, в факе пояснить, а то тут недавно два дня объяснял одному нытику, что ему нехуй делать в треде про топологию.

Какие еще платиновые вопросы есть?
- пошаговый калькулятор
- книги по школьной математике
- мне эн лет, поздно ли вкатываться
- не могу в логику\матнотацию, посоветуйте
- а как вы понимаете это а вам правда нравится а удовольствие получаете

Что еще?
559 8461
>>8458
Тоже верно. И так-то вообще обсуждать шапку надо не в этом треде, а в отдельном. А тут для вопросов типа. бля не репортуйте мой пост, жалко ж, если потрут

Вот про это тоже в фак надо написать?
560 8463
>>8460
А где о последнем найти много?
561 8465
>>8459
Сомневаюсь.
Подразумевалось что ты настолько умный чтобы понять школьны курс, но настолько туп чтобы не понять учебник по вышмату.
562 8467
>>8463
Может нарезкой про гомотопический хаос на это отвечать? Ну или пояснять кратенько за философию математики, мол мы тут идеальные миры исследуем, цыц.
563 8468
>>8452

>Если уйти в доказательства то уйдешь в бесконечный цикл в котором нужно все доказывать и доказывать. Одно доказательство стоит на другом, другое на третьем, и так до бесконечности



нет

>Это показано у бурбаков.



нет

>И опять же, если множества так просты и элементарны и фундаментальны, почему их не разжевывают в учебниках типа этого?



Потому что существуют люди типа тебя с iq 97, не способные понять простейших вещей по физиологическим причинам.

>сложной и непонятной


Мне было легко и понятно.

>без пояснения


Мне пояснений было достаточно.

>сложными терминами


У меня термины не вызывали сложностей.

>это не только проблема учебников


это исключительно проблема тупых неосиляторов

>Неправ я


Во многом, ты неизлечимо туп.
564 8469
>>8465
Да я мимо проходил. Сейчас учу последовательно, где-то в 7 классе уже почти. Хоть и блеск озарениями бывает, но все-же скучновато. Хотелось бы, вообще, что-то вроде как дети изучают с помощью игры обучаются.
565 8470
Кто там пиздел "ЛОГИКА ЭТО МАТЕМАТИКА"?
Заходим на вики и зрим что

>Логика (др.-греч. λογική — «Философский термин», «способность к рассуждению» от др.-греч. λόγος — «рассуждение», «мысль», «разум») — раздел философии, нормативная[1] наука о формах, методах и законах интеллектуальной познавательной деятельности, формализуемых с помощью логического языка...... Кроме философии, логика также является подразделом математики, а булева алгебра одной из основ информатики.


Заходим в математическую логику и, о ужас

>Математическая логика (теоретическая логика, символическая логика[1]) — раздел математики, изучающий математические обозначения, формальные системы, доказуемость математических суждений, природу математического доказательства в целом, вычислимость и прочие аспекты оснований математики[2]. В более широком смысле рассматривается как математизированная ветвь формальной логики[3] — «логика по предмету, математика по методу»[4], «логика, развиваемая с помощью математических методов»[5].



Так что с помощью математической логики из двух посылок "Сокрта - человек" и "Все люди смертны" нельзя вывести следствие "Следовательно Сократ смертен".
566 8472
>>8468
Переходим на личности, мм, /Ь/ратишка?

Поясни тогда, почему мне очевидно когда тянучку нужно идти ебать тепленькую, а ты поясняешь на сосачах дурачкам типа меня что то про множества и отрицание замкнутости петли доказательств, которые основываются на субъективном чувственном опыте понятия числа?
567 8473
>>8470
Абстрагируйся от этого. Математическая логика может это вывести, ведь она оперирует отношениями.
568 8474
>>8458
Вообще-то я сам начинающий и Шварца еще не дочитал. Просто есть люди, которые и не притронутся к математике, а в тред придут срать о том, чего знать не знают.
569 8475
>>8470
Конечно, нельзя. Математическая логика это дурацкое название, люди путаются. Более удачное – "логика математики". Типа "какая в ваших рассуждениях логика?". Метадисциплина такая, обсуждает методы, что в математике допустимо и корректно, а что нет.
570 8476
>>8469
В седьмом классе дроби учат складывать, ты этого не умеешь делать?
571 8478
>>8476
Да путаюсь постоянно. Да еще и только сегодня вспомнил и запомнил, какие углы есть прямые, острые и тупые. Я учился в 7 классе как-то не стараясь вникать в суть.
572 8479
>>8472
Ты умственно отсталый. Ты не способен понимать математику, тебе это не нужно, и ты пришел сюда срать.
573 8481
>>8478
Ладно, нужно дать себе слово, что я больше не буду здесь появляться со следующего треда, эта доска отнимает слишком много сил.
574 8482
>>8481
Это из-за меня?
575 8484
>>8472
Блять, самое несправедливое, что после всего этого ты будешь считать, что ты прав. Типа ты настолько туп, что обосрался на простейшем теоретико множественном утверждении, что учебники твои написаны для даунят, которые математикой после последнего сданного экзамена заниматься не будут, что физические законы останутся для тебя не познаваемыми, после прочтения этого говна, но прав ты, потому что дурака называть дураком грубо.
576 8487
>>8481
Не вступай в долгие диалоги с упёртыми неосиляторами и времени будет больше.
577 8491
>>8487
This
578 8515
sup! матемач. Как доказать пикрил 2.12, про то, что на последнем шаге коэффициенты в представлении нуля взаимно просты (НОД(q0, q1) == 1)
579 8536
>>8439
Пиздец там вопросов
580 8537
>>8479
Мне очень важно твое мнение. Держи меня в курсе.

>>8484

>учебники твои написаны для даунят, которые математикой после последнего сданного экзамена заниматься не будут,


Сильное утверждение. Доказывать ты его конечно же не будешь, ведь это ОЧЕВИДНО КАК МОЖНО ПОКАЗАТЬ ПОСЛЕ НЕБОЛЬШИХ ПРЕОБРАЗОВАНИЙ ЧТО И ТРЕБОВАЛОСЬ ДОКАЗАТЬ!?
И да, я прав. Понятные и просто написанные учебники нужны и важны. Учебники в духе "очевидных небольших преобразований", которых сейчас ебаное большинство, можно использовать только с тем кто уже знает курс учебника.
Ну и ты так и не показал мне почему я не прав, хотя я и согласиля что я не прав в непрерывности чисел между нулем и двойкой
581 8539
>>8537

>И да, я прав. Понятные и просто написанные учебники нужны и важны. Учебники в духе "очевидных небольших преобразований", которых сейчас ебаное большинство, можно использовать только с тем кто уже знает курс учебника.


Дело в том, что нормальным людям эти учебники понятны и доступны. Просто ты особый случай, пытаешься изучать то, к чему твой мозг не приспособлен.
А потом из-за ущемлёного чувства достоинства проецируешь свои чувства по отношению к себе на авторов учебников. ЭТО НЕ Я ДАУН, ЭТО АВТОРЫ ТУПЫЕ НИЧЕГО МНЕ ОБЪЯСНИТЬ НЕ МОГУТ, ВЕДЬ НУЖНО ПОНЯТНО, ЧТОБЫ КАРТИНКИ БЫЛИ, А ТО ДЕГЕНЕРАТЫ НЕ ПОНИМАЮТ, КАК ДЕЛАТЬ, ДОКАЗАТЕЛЬСТВА ТАМ КАКИЕ-ТО)))
Но в глубине души, ты осознаешь, что ты тупой даун и не можешь понять, то что на самом деле ОЧЕВИДНО.
Помогите с блядским экзаменом 582 8541
Сап, аноны. Прошу не кидайтесь гавном и помогите с этой парашей, нихуя не въезжаю
1.Кибернетические системы. Входные и выходные воздействия.
2. Основные определения алгебры логики. Двоичный набор. Функция алгебры логики. Фиктивные аргументы и метод их выявления.
3.Структура системы. Элементы систем. Оператор системы. Векторы входных и выходных сигналов. Возмущающие воздействия. Отрицательная обратная связь. Задачи исследования систем
4. Элементарные функции алгебры логики.
5.Элементы теории множеств. Диаграммы Эйлера-Венна. События. Разность событий. Объединение и пересечение событий.
6. Таблица функции двух переменных.
7.Элементы функционального анализа. Расстояние. Метрические пространства. Евклидова метрика.
8. Аналитическая запись логических функций. Характеристическая функция единицы.
9.Евклидовы пространства . Образ шара.
10. Дизъюнктивное представление логической функции.
11.Пространства Гильберта
12.Условные понятия степени аргумента логической функции. Использование дизъюнкции, конъюнкции для описания функций алгебры логики.
13.Скалярное произведение векторов в Евклидовом пространстве . Расстояние между элементами данного пространства.
14. Таблица соответствия для выходных слов и внутренних состояний конечных автоматов (последовательных схем). Глубина памяти. Стохастические автоматы.
15.Иллюстрация понятия расстояния в пространствах .
16. Законы алгебры логики. Формулы Де-Моргана.
17.Ортонормированный базис линейного пространства. Базис пространства, ортогональность векторов.
18. Свойства конъюнкции и дизъюнкции. Сочетательный, переместительный и распределительный законы.
19.Расстояние между векторами в пространстве Гильберта в . Норма вектора. Скалярные произведения векторов в . Ортогональный базис.
20.Условные понятия степени аргумента логической функции. Использование дизъюнкции, конъюнкции для описания функций алгебры логики.
21.Обобщенный спектральный анализ в пространстве Гильберта. Ряд Фурье по ортогональной системе.
22. Геометрическая интерпретация логической функции двух аргументов с примерами раскраски вершин единичного куба.
23. Построение ортонормированного базиса методом Грамма-Шмидта.
24. Задание функции алгебры логики. Таблица истинности и покрытие нулевых и единичных вершин единичного куба дизъюнкциями и конъюнкциями.
25.Основные понятия математической логики. Высказывания, предикаты, алгоритмы, конечные автоматы.
26. Таблица функции двух переменных.
27.Конечные автоматы. Входной и выходной алфавит. Таблица соответствия для выходных слов. Комбинационная схема.
28. Полные системы функций в классе R. и Mn. Полная система в базисе И-ИЛИ-НЕ.
29. Таблица соответствия для выходных слов и внутренних состояний конечных автоматов (последовательных схем). Глубина памяти. Стохастические автоматы.
30.Минимизация логических функций методом покрытия нулевых вершин и рёбер куба дизъюнкциями.
Помогите с блядским экзаменом 582 8541
Сап, аноны. Прошу не кидайтесь гавном и помогите с этой парашей, нихуя не въезжаю
1.Кибернетические системы. Входные и выходные воздействия.
2. Основные определения алгебры логики. Двоичный набор. Функция алгебры логики. Фиктивные аргументы и метод их выявления.
3.Структура системы. Элементы систем. Оператор системы. Векторы входных и выходных сигналов. Возмущающие воздействия. Отрицательная обратная связь. Задачи исследования систем
4. Элементарные функции алгебры логики.
5.Элементы теории множеств. Диаграммы Эйлера-Венна. События. Разность событий. Объединение и пересечение событий.
6. Таблица функции двух переменных.
7.Элементы функционального анализа. Расстояние. Метрические пространства. Евклидова метрика.
8. Аналитическая запись логических функций. Характеристическая функция единицы.
9.Евклидовы пространства . Образ шара.
10. Дизъюнктивное представление логической функции.
11.Пространства Гильберта
12.Условные понятия степени аргумента логической функции. Использование дизъюнкции, конъюнкции для описания функций алгебры логики.
13.Скалярное произведение векторов в Евклидовом пространстве . Расстояние между элементами данного пространства.
14. Таблица соответствия для выходных слов и внутренних состояний конечных автоматов (последовательных схем). Глубина памяти. Стохастические автоматы.
15.Иллюстрация понятия расстояния в пространствах .
16. Законы алгебры логики. Формулы Де-Моргана.
17.Ортонормированный базис линейного пространства. Базис пространства, ортогональность векторов.
18. Свойства конъюнкции и дизъюнкции. Сочетательный, переместительный и распределительный законы.
19.Расстояние между векторами в пространстве Гильберта в . Норма вектора. Скалярные произведения векторов в . Ортогональный базис.
20.Условные понятия степени аргумента логической функции. Использование дизъюнкции, конъюнкции для описания функций алгебры логики.
21.Обобщенный спектральный анализ в пространстве Гильберта. Ряд Фурье по ортогональной системе.
22. Геометрическая интерпретация логической функции двух аргументов с примерами раскраски вершин единичного куба.
23. Построение ортонормированного базиса методом Грамма-Шмидта.
24. Задание функции алгебры логики. Таблица истинности и покрытие нулевых и единичных вершин единичного куба дизъюнкциями и конъюнкциями.
25.Основные понятия математической логики. Высказывания, предикаты, алгоритмы, конечные автоматы.
26. Таблица функции двух переменных.
27.Конечные автоматы. Входной и выходной алфавит. Таблица соответствия для выходных слов. Комбинационная схема.
28. Полные системы функций в классе R. и Mn. Полная система в базисе И-ИЛИ-НЕ.
29. Таблица соответствия для выходных слов и внутренних состояний конечных автоматов (последовательных схем). Глубина памяти. Стохастические автоматы.
30.Минимизация логических функций методом покрытия нулевых вершин и рёбер куба дизъюнкциями.
583 8543
>>8541
Тебе все билеты объяснить надо?
584 8544
>>8541
>>8543
хоть что-нибудь
585 8545
>>8541

>10. Дизъюнктивное представление логической функции.


Вот про это как раз вчера читал у шлюхоеба. Легкотня.
недоучка-кун
586 8546

>3.Структура системы. Элементы систем. Оператор системы. Векторы входных и выходных сигналов. Возмущающие воздействия. Отрицательная обратная связь. Задачи исследования систем


>Элементы систем


Осколки империи элементы систем.
И тот кто был всем - тот станет никем.

Вообще трэш и угар какой-то. Формулы де Моргана, спектральная теория оператора на гильбертовом пространстве и конечные автоматы в одном билете.

Нужно было ещё добавить что-то про нётеровы схемы, представления конечных групп, ракетостроение и лечение ЗППП подорожником.
587 8547
>>8546
И джаваскрипт, без него сегодня никуда
588 8548
>>8539
Ясно.
Все как всегда, нихуя не помогли, но хуями обложили.
589 8549
>>8539
Скажи еще что вот ему
>>8541
тоже

>Дело в том, что нормальным людям эти учебники понятны и доступны. Просто ты особый случай, пытаешься изучать то, к чему твой мозг не приспособлен.


>А потом из-за ущемлёного чувства достоинства проецируешь свои чувства по отношению к себе на авторов учебников. ЭТО НЕ Я ДАУН, ЭТО АВТОРЫ ТУПЫЕ НИЧЕГО МНЕ ОБЪЯСНИТЬ НЕ МОГУТ, ВЕДЬ НУЖНО ПОНЯТНО, ЧТОБЫ КАРТИНКИ БЫЛИ, А ТО ДЕГЕНЕРАТЫ НЕ ПОНИМАЮТ, КАК ДЕЛАТЬ, ДОКАЗАТЕЛЬСТВА ТАМ КАКИЕ-ТО)))


>Но в глубине души, ты осознаешь, что ты тупой даун и не можешь понять, то что на самом деле ОЧЕВИДНО.

590 8550
>>8548
Тебе помогли сразу же и очень терпеливо потом разжовывали что не так ты делаешь. До тебя не дошло и ты продолжал ныть про картиночки — справедливо получил хуев за воротник.
591 8551
>>8548
Ты сам начал обкладывать авторов учебников, это во первых. Во вторых, тебе помогли. Тебе сказали, что ты слишком тупой, чтобы понимать нормальные учебники и что виноваты не авторы, а лично ты. Так что либо развиваешь свой интелект, чтобы ботать нормальные книги, либо идёшь нахуй. Специально для таких умственно отсталых как ты не будут писаться книги по математике отдельно, это того не стоит.
>>8549
У него и у тебя разные вопросы и отвечать на них следует по разному. Расписывать все билеты я не буду, пусть у одногруппников материал спросит, большу толку будет.
592 8552
>>8439
Плохой тест. Я ИРЛ гений, а тест показал что я идиет
сетка Багуа - копия.jpg428 Кб, 1600x900
593 8553
>>8550
Про какие картиночки? Ищи где я что то про картиночки писал.

>>8551
Открываем учебник фихтенгольца на 26 странице, где он сразу и без предупреждение начинает множества ебашить.
Это по твоему нормально? Или все кто не понял с наскока, без подготовки, умственно отсталые?
594 8556
>>8553
А что тебе тут не понятно? Слово "множество"?
595 8557
>>8553

>Это по твоему нормально?


Да. Как по твоему должна выглядить построика действительных чисел в курсе матана?

>Или все кто не понял с наскока, без подготовки, умственно отсталые?


А сколька у тебя времени было, чтобы переварить это? За два дня, что тыт тут ноешь, можно было въехать.
596 8558
>>8553
Повторяю: тебе сразу помогли и сказали что делать, если ты не понимаешь математику и не хочешь разбираться. Сказали что делать, если ты не понимаешь и хочешь разбираться.

Какие ещё вопросы у тебя?
597 8559
>>8556
Да. Как же это без доказательства понимать слово "множество"?

>>8557
Зачем она там нужна, если оперирует понятиями уровня первого класса сельской приходской школы времен 18 века?
Ничего более школьного курса, кроме доказательства бесконечности дробных частей иррациональных чисел, там нету.
598 8560
>>8558
Почему существуют калькулусы если они неправославны по мнению доски мейлручей?
599 8561
>>8560
Чтобы ПТУшники и школьники научились считать элементарные интегральчики и дифуры.
600 8562
>>8561
Хорошо, почему тогда теория о бесконечно малых преподаваемая на основании теории множеств представляет из себя переусложненное изложение того, что написано в книжке лузина на первых 80 страничках?
601 8563
>>8562

>теория о бесконечно малых


Ору.

>на основании теории множеств


Что тебя смущает в множествах?

>из себя переусложненное изложение того, что написано в книжке лузина на первых 80 страничках?


Что за книга? И что там конкретно написанно?
602 8564
>>8562
Я не знаю, что там писал Лузин на своих страничках.

Лена, иди читай книги, хватит сюда писать хуйню.
603 8565
>>8563
http://www.vixri.ru/?p=236
Сайт конечно хуевенький, но у меня регистрации не просит чтобы прочитать.
604 8566
По чему тебя не устраивает множественный формализм?
605 8567
>>8565
У Фихтенгольца правда ебанутый и старый способ определение действительных чисел, который был придуман ещё до множеств. У Зорича они определяются так же, как и в Лузине.
606 8568
>>8567
У Зорича, насколько я понял, аксиоматически они определены. А есть явные конструкции действительных чисел из рациональных, или там из геометрии, прости господи.
607 8569
>>8567
Как тут выяснили, множества появились в 19 веке, а фихтенгольц родился в 1888 году. Он просто не мог не знать множества.
608 8570
>>8568
Что значит явные?
Зорич доказывает, что точки на прямой изоморны полю действительных чисел - вот и твоя явная интерпритация.
610 8573
>>8569
До множеств один из способов определение действительных чисел через сечения и классы. Потом на этот способ натянули множества, назвав классы множествами, лол. У Харди такой способ определения, без множеств, но с классами.
611 8575
>>8567
В чем ебанутость дедекиндовых сечений? Аргументируй или убей себя.
612 8577
>>8570
Значит как в современных учебниках, вроде Аман-Эшера. То как делает Зорич имхо даже вредная хуита, т.к. не имеет формального смысла, только "напальцевый".
613 8579
>>8575
Устаревшее определение, которое нужно только для ввода действительных чисел и мало связанно с матаном. Не очень красивое, кстати. Через фундаментальные последовательности лучше и полезнее.

>>8577
Зорич сначала говорит о формальным смысле и аксиомах, а потом о наглядном смысле с прямой.
614 8580
>>8579
Где он говорит о формальном смысле? Он просто предлагает поверить в аксиоматику, мол умные дяди обо всём позаботились.
615 8581
>>8580
Что не так с аксиоматикой?
616 8582
>>8581
Естественный вопрос: почему именно такая, а не другая?
617 8584
>>8582
А какую аксиоматику ты ожидаешь увидеть?
618 8585
>>8581
Всё так, но если есть явное построение его тоже следует изучить.
619 8586
>>8585
Зачем?
620 8587
>>8586
Чтобы понимать с чем работаешь.
621 8589
>>8587
Для этого и аксиоматики достаточно.
622 8590
>>8589
Если ты инженер — безусловно. Если математик — нет.
623 8591
>>8590
У тебя в корне неправльное представление о математике, она часто работает с аксиоматически методом. И математики им пользуются. Ты только что назвал дохуища математиков, в том числе Бурбаков и Гилберта инженерами. Молодец.
Объясни, как явное построение позволяет лучше понимать объект. То же сечение Дедикинда просто архаичный мусор, который бесполезен для математика.
624 8592
>>8591
*Дедекинда
625 8593
>>8591

>У тебя в корне неправльное представление о математике, она часто работает с аксиоматически методом.


Я в курсе, спасибо. Не знать конкретно конструкцию действительных чисел для математика зашквар. И Гильберт, и Бурбаки её знали.

> архаичный мусор


Я понял твою точку зрения, во мне понимания ты не найдёшь.
626 8594
Посоветуйте книгу для инженеров.
627 8595
>>8593

>Объясни, как явное построение позволяет лучше понимать объект.


>И Гильберт, и Бурбаки её знали.


Я говорил про аксиомы, знали ли они её или нет имеет связи с тем, что я сказал. Я говорил про аксиоматический методв целом.

>Не знать конкретно конструкцию действительных чисел для математика зашквар.


Чем конструкция с аксиомами хуже?

>Я понял твою точку зрения, во мне понимания ты не найдёшь.


Тогда скажи, где она применяется кроме определения действительных чисел? Какая от неё польза в самом анализе или за его пределами?
628 8596
>>8591

>сечение Дедикинда просто архаичный мусор


А не пошел бы ты нахуй, собака.
Дедекинд это величайший математик в истории после Гаусса, такой Андре Вейль (и Ленглендс, в одном лице) 19-го века. Среди его изобретений, помимо идеала и модуля
-римановы поверхности в современном определении
-дедекиндовы сечения
-канторовская теория множеств
-аксиоматика пеано
И все это, согласно задумке Дедекинда, должно было объединить математику.
629 8597
>>8595

>где она применяется кроме определения действительных чисел


Того, что эта удивительная по красоте конструкция в некотором смысле аналогична модулю над кольцом, уже не достаточно что ли?
Ты охуел совсем, я посмотрю.
630 8598
>>8595
Если ты не понимаешь дополнительной ценности в явной конструкции математического объекта, я тебе ничем помочь не могу.
631 8599
>>8596
Это же не значит, что все, что он сделал имеет одинаковую ценность. Те же сечения отвратительны и бесполезны.

>>8597
Нет, сечения не красивы. Совсем. Если для тебе этот мусор красивый, то это значит, что у тебя дурной вкус. Вот последовательности Коши. Да хорошая вещь.

>>8598
Я действительно не понимаю, объясни мне.
632 8605
>>8594
Зельдович математика для техников и инженеров
633 8606
Так, хорошо, давайте тогда самую чоткую книжку в которой в вышмат вкатываются прямо напрямую через топологию.
634 8607
>>8596

>Дедекинд это величайший математик в истории после Гаусса


Святая толстота.
Давай посмотрим, в честь дедекинда назвали йоба пушку, кратер на луне и график нормального распределения?
635 8608
>>8606
Шварц анализ.
636 8609
>>8608
Еще
637 8617
>>8607
Способен судить только по названиям? Открою секрет, теорема Гаусса-Бонне не имеет никакого отношения ни к Гауссу, ни Бонне.
Дедекинд просто сильно недооценен, в отличие от какого-нибудь Галуа, который дохуя переоценен.
638 8619
>>8617
Я тебе тоже открою секрет.Гаусс не имеет никакого отнощение к кратеру имени себя и к пушке имени себя.
Но из за того каким он был человеком ему все высказывают уважение в такой манере.

Теперь давай посмотрим на твоего декаденса. Что он сделал значительного чтобы в его честь назвать астероид какой то?
639 8624
>>8617

>Дедекинд просто сильно недооценен,


Действительно, ведь сечения Q такая полезная вещь! Встречается в математике на каждом углу. Без неё не топологию не построить, в алгбре без неё никак, в геометрии тоже. Сам удивляюсь, как в анализе без неё обходятся и отправили такую вещь на свалку истории. Эх. Что-что? Явное построение? Ох, без него никуда! Мнение парочки коструктивистов очень важно, а значит эти сечения должны быть не в специализорованный литературе по математической логике и основаниям а в книге анализа, правильно, нечего веровать в аксиоматику, веру плодить себя губить. Да, как же можно заниматся анализом не зная, что такое сечение, даже в Демидовиче на первых страницах задачи с сечениями, ух, всё по Фихтенгольцу.
423413224.png287 Кб, 1123x660
640 8629
Я склепал дебильный мемчик, оцените пожалуйста.
641 8632
>>8629
Тепепь это ненависти к Дедекинду тред.
642 8633
>>8629
Как же я проиграл с Рехорда.
643 8634
>>8619
>>8629
Я уже понял, что ты не математик, и знаешь о Гауссе только по рельсогану. Перечислишь реальные достижения Гаусса?
>>8619
>>8624

>что он сделал значительного


>кривляется про сечения


1. Понятие идеала и модуля, nuff said, наука о модулях это и есть определение математики
2. Риманова поверхность как поле функий, понятие связавшее комплексный анализ и комбинаторную топологию
3. Теория множеств, которую спиздил Кантор (Дедекинд был настолько хорошим человеком, в отличие от Гаусса, что не стал претендовать на приоритет, хотя публиковал работы; вместо этого он стал пропогандировать работы Кантора. Гаусс же только умел ссылаться на неопубликованные результаты, за что его м хуесосили)
4. Аксиоматика Пеано до Пеано уже была у Дедекинда.
Все? Или еще надо обосать?
644 8635
>>8634
Я ещё раз говорю, если он сделал полезные вещи, не значит, что всё, что делал полезно. Сечения Q - мусор, просто признай это, и место этому архаизму в книге Харди и Фихтенгольце.
1) Спорно, Вербиту не понравилось, он сказал, что это определение, как сепульки.
3) Пруфов не будет?
645 8636
>>8635

>Вербиту не понравилось


Причём тут ваще Вербит? Если тебе так важно мнение Вербита иди спроси у него нужно ли студентов учить явной конструкции действительных чисел.
646 8637
>>8584
Обоснованную. Еще раз: почему у действительных чисел именно такая аксиоматика? Почему она полна и непротиворечива?
647 8638
>>8636
Ты даже не видел его критику, а уже агришся. Вне математике, фраза модуль над кольцом не имеет смысла. А теперь смотри, давая определение математики через неё саму, мы даём определение, понятиям входящих в неё, через самих себя.
648 8639
>>8638

>Ты даже не видел его критику


Херасе, ты что ли всевидящий? Откуда тебе знать, что я не видел?
649 8640
>>8639
Тогда зачем ты продолжаешь говорить, что математика наука о модулях над кольцами, эти приводит к порочному кручу.
650 8641
>>8635
1) Ты не прочитал, либо не понял, что он написал. Утверждений там было два
а) это контекстуальное определение (что совершенно очевидно)
б) данное определение ничем не лучше определения математики как науки о пучках
Знающий человек поймет о чем речь: проективные модули эквивалетны локально свободным когерентным пучкам. Прочесть об этом можно в "коммутативной алгебре" Бурбаки.
2) Любая книга по истории математики второй половины XIX века:
Dedekind's more foundational work in mathematics is also widely known, at least in parts. Often acknowledged in that connection are: his analysis of the notion of continuity, his introduction of the real numbers by means of Dedekind cuts, his formulation of the Dedekind-Peano axioms for the natural numbers, his proof of the categoricity of these axioms, and his contributions to the early development of set theory (Grattan-Guinness 1980, Ferreirós 1996, 1999, 2016b, Jahnke 2003).

What happens in Was sind und was sollen die Zahlen?, in the context of Dedekind's logicist reconstruction of the natural numbers, is that the adoption of set-theoretic techniques is raised to a new level of clarity and explicitness. Dedekind not only presents set-theoretic definitions of various mathematical notions, he also adds a systematic reflection on the means used thereby (and he expands that use in certain respects). Consequently, the essay constitutes an important step in the rise of modern set theory. We already saw that Dedekind presents the notion of set, together with those of object and function, as fundamental for human thought. Here an object is anything for which it is determinate how to reason about it, including having definite criteria of identity (Tait 1997). Sets are a kind of objects about which we reason by considering their elements, and this is all that matters about them. In other words, sets are to be identified extensionally, as Dedekind is one of the first to emphasize. (Even as important a contributor to set theory as Bertrand Russell struggles with this point well into the twentieth century.) Dedekind is also among the first to consider, not just sets of numbers, but sets of various other objects as well.
650 8641
>>8635
1) Ты не прочитал, либо не понял, что он написал. Утверждений там было два
а) это контекстуальное определение (что совершенно очевидно)
б) данное определение ничем не лучше определения математики как науки о пучках
Знающий человек поймет о чем речь: проективные модули эквивалетны локально свободным когерентным пучкам. Прочесть об этом можно в "коммутативной алгебре" Бурбаки.
2) Любая книга по истории математики второй половины XIX века:
Dedekind's more foundational work in mathematics is also widely known, at least in parts. Often acknowledged in that connection are: his analysis of the notion of continuity, his introduction of the real numbers by means of Dedekind cuts, his formulation of the Dedekind-Peano axioms for the natural numbers, his proof of the categoricity of these axioms, and his contributions to the early development of set theory (Grattan-Guinness 1980, Ferreirós 1996, 1999, 2016b, Jahnke 2003).

What happens in Was sind und was sollen die Zahlen?, in the context of Dedekind's logicist reconstruction of the natural numbers, is that the adoption of set-theoretic techniques is raised to a new level of clarity and explicitness. Dedekind not only presents set-theoretic definitions of various mathematical notions, he also adds a systematic reflection on the means used thereby (and he expands that use in certain respects). Consequently, the essay constitutes an important step in the rise of modern set theory. We already saw that Dedekind presents the notion of set, together with those of object and function, as fundamental for human thought. Here an object is anything for which it is determinate how to reason about it, including having definite criteria of identity (Tait 1997). Sets are a kind of objects about which we reason by considering their elements, and this is all that matters about them. In other words, sets are to be identified extensionally, as Dedekind is one of the first to emphasize. (Even as important a contributor to set theory as Bertrand Russell struggles with this point well into the twentieth century.) Dedekind is also among the first to consider, not just sets of numbers, but sets of various other objects as well.
651 8642
>>8640
*кругу
652 8643
>>8640
Это не я говорю, я другой анон.

Я на минуту подумал, что ты тот поехавший, которому конструкция R не нужна.
653 8644
>>8643
Лол. А зачем она нужна, ну скажи же.
654 8645
>>8644
Так ты тот поехавший и ты же >>8635? Это важно.
655 8646
>>8634
Дружище, я вообще тут ни к месту, я просто мемы клепаю.
656 8647
https://plato.stanford.edu/entries/dedekind-foundations/
>>8640
Какой порочный круг, ты тупой? Определения бывают разными, это – контекстуальное. А сказав про пучки, тифарет по сути согласился со мной, смотри выше.
>>8643
Про ненужность R это я набрасывал, а не он. Про мистику и эзотерику тоже я.
657 8648
>>8645
Я не говорил, что конструкция R не нужна. Просто есть фундаментальные последовательности, которые гораздо лучше, чем дедекиндский мусор.
658 8653
Вы вызывали у себя эйдетические образы?
659 8655
>>8647

> Про мистику и эзотерику тоже я.


Это где лол? Про Манина и Гротедника что ль в этом треде?
660 8656
>>8655
В маттредах sci до создания этого раздела.
661 8663
>>8648
Определение Кантора несколько более громоздко, чем определение Дедекинда, когда дело доходит до определения элементарных функций. Например, возведение в степень по Дедекинду определяется в несколько слов. А знаешь ли ты, как определить возведение в степень с помощью фундаментальных последовательностей?
множества пример.png86 Кб, 1920x1080
662 8664
Кто тут пиздел что невозможно приставить высшую математику в простых примерах.
Вот такие должны быть учебники на простых элементарных примерах, как на картинке, где обычный человек видит и начинает понимать что к чему и в голове уже сопоставляются логические связи. Даже обезьяна поймет тот пример который я нарисовал.
663 8669
>>8664
Не писать же под каждым очевидным фактом пример только ради таких как ты.
664 8671
>>8669
Учебники должны делаться для того чтобы люди поняли и усвоили материал, а не для того чтобы автор книги увеличивал свое чсв что он умную непонятную книгу написал.
665 8672
>>8671

>666


>а не для того чтобы автор книги увеличивал свое чсв что он умную непонятную книгу написал.


Ну тебе, конечно, виднее, для чего ученые и люди знаний пишут книги, дьявол.
666 8674
>>8671
Они для этого и делаются, просто ты тупой и тебе нужны ОСОБЫЕ учебники для умственно одарённых.
667 8676
>>8664
Почему ты решил, что обычный человек не может понять конкретно выделенное тобой утверждение без простого примера?
668 8677
>>8674
Я не обязан знать все на свете.

Людям нужны понятные разжеванные учебники а не инопланетное чтиво.

>>8676
Я его не понял значит и другой человек не поймет, я не говорю за всех, но большинству нужно примеры чтобы понять.
669 8679
>>8677

>Я не обязан знать все на свете.


Правильно. А ещё ты не обязан знать математику.

>Людям нужны понятные разжеванные учебники а не инопланетное чтиво.


Тот же Зорич или Фихтенгольц, особенно последний, понятно разжёванные учебники. Если разжёвывать материал в твоём даунском стиле, то учебники будут неподъёмными, все будет засранно тривиальными объяснениями и картинками. По таким учебникам ты ничего не выучишь, хотя бы из-за того, что колиство страниц станет гиганским.

>Я его не понял значит и другой человек не поймет


Ложное следствие. По твоему, если я понял, значит и другой поймёт? Я не говорю за всех, но особо одарённым нужны примеры, чтобы понять.

>но большинству нужно примеры чтобы понять.


Никто не запрещает тебе придумывать примеры самостоятельно. Можешь ещё и картиночки к каждой теореме рисовать.
670 8681
>>8677

>большинству


Большинству обычных людей нужен пример, чтобы понять дистрибутивность пересечения множеств относительно объединения?
671 8682
>>8679
Скачал и посмотрел Фихтенгольца и Зорича, это непонятное дно.

>Если разжёвывать материал в твоём даунском стиле, то учебники будут неподъёмными, все будет засранно тривиальными объяснениями и картинками. По таким учебникам ты ничего не выучишь, хотя бы из-за того, что колиство страниц станет гиганским.



Лучше потерять время и нормально изучить на простых примерах и понять, нежели потерять время расшифровывая "китайскую грамоту" и ничего не выучить.

>По твоему, если я понял, значит и другой поймёт?


Есть сопредельный процент людей, кто может понят, и той процент очень мал.

>Никто не запрещает тебе придумывать примеры самостоятельно. Можешь ещё и картиночки к каждой теореме рисовать.


Я могу нарисовать, но не факт что я пойму материал правильно и не ошибусь в самом начале.

Иза таких некомпетентных авторов книг и учителей вузов, в мире не повляются гении типа эйнштейн, теслы, гаусса и т.д.
672 8684
>>8681
В книге даже не объясняется что такое дистрибутивность.

>Большинству обычных людей нужен пример, чтобы понять дистрибутивность пересечения множеств относительно объединения?



Да нужен пример, не только на это но на все выражения в книге. Тогда это будет полноценный качественный учебник.
673 8685
>>8684
Ты считаешь, что большинству обычных людей нужен пример, чтобы понять дистрибутивность пересечения множеств относительно объединения. Никто кроме тебя здесь так не считает. Как ты можешь это прокомментировать?
674 8686
>>8685
Да я так считаю.

>Никто кроме тебя здесь так не считает.


Ты что у каждого спрашивал и проводил опрос?
675 8687
>>8682

>Скачал и посмотрел Фихтенгольца и Зорича, это непонятное дно.


Прекращай проецировать, дно ты раз не можешь понять Фихтенгольца, который создавался для ниженеров. По чему ты не признаёшь того, что ты тупой?

>Лучше потерять время и нормально изучить на простых примерах и понять, нежели потерять время расшифровывая "китайскую грамоту" и ничего не выучить.


Не факт. Учёба не бывает простой, это тебе не лекции для семиклассников и не научпоп. И есть мнение, что как раз при такой расшифровке ты и учишся чему-то.

>Есть сопредельный процент людей, кто может понят, и той процент очень мал.


Понимаешь, процент людей, занимающихся математикой очень мал и они не собираются все расписывать для того, чтобы, чтобы тупой, понял то, что умному очевидно. Тебе ясно?

>Иза таких некомпетентных авторов книг и учителей вузов, в мире не повляются гении типа эйнштейн, теслы, гаусса и т.д.


Во первых, это не так. Эти люди могли понять то, во что ты не врубаешься.
676 8688
>>8686
Ты первый, кто за 70+ тредов начал ныть о таком и назвал авторов учебников дегенератами. Так, что да. Никто кроме тебя так не считает.
677 8689
>>8686
Опроса не было, но все ответы на твои сообщения содержат несогласие с твоими суждениями. Но предположим, что ты считаешь так, а я считаю по-другому. Доказательств у тебя нет и у меня нет. Тогда к чему эти вбросы?
бля, ребята, закругляйтесь 678 8694
Ссылку на вики уже оставили, так что пусть братан-картофан пилит новый тред, этот скатился в убогую полемику уёбищный флейм между первокурсником и у.о. 7-классником.
679 8699
>>8687

>По чему ты не признаёшь того, что ты тупой?


Потому что я не тупой, я тоже могу написать книгу в которой ты ничего не поймешь и будешь тоже тупой.

>И есть мнение, что как раз при такой расшифровке ты и учишся чему-то.


Расшифровка информации только тратит время, заместо объяснений.

>Понимаешь, процент людей, занимающихся математикой очень мал


Потому и мал что книги пишутся некомпетентными людьми.

>Эти люди могли понять то, во что ты не врубаешься.


Не спорю есть уникумы на свете.

>>8689

>Тогда к чему эти вбросы?


Чтобы открыть людям глаза на то, что не они виноваты в том что не понимают математику и остальные предметы, а виновны авторы учебников, учителя вузов и система образования.
680 8701
Как доказать, что декартово произведение множеств ассоциативно?
681 8702
>>8701
Расскрой по определению произведения результирующее множество слева и справа равенства. Убедись, что получилось одно и то же выражение.
682 8703
>>8699
Госпади, какой же ты еблан. Это не авторы учебников виноваты, а именно ты, кретин. Глаза он собрался открывать.
Госпади, это самый тупой посетитель /math.

>Потому что я не тупой, я тоже могу написать книгу в которой ты ничего не поймешь и будешь тоже тупой.


Смотри, люди понимают учебник, а ты особенный тебе нужен специальный учебник для даунов. Ты не можешь учится по нормальным, потому что они слишком сложны для тебя. Суть - ты тупой. ТУПОЙ.

>Расшифровка информации только тратит время, заместо объяснений.


Какой информации? Что ты говоришь, кретин. То, что для нормальных людей ОЧЕВИДНО, то для тебя нет. Тебе нужны дополнительные объяснения.

>Потому и мал что книги пишутся некомпетентными людьми.


Объясни с чего это вдруг они некомпетентные? Ты ведь понимаешь, что у тебя проблеммы с логикой - в твоей невозможности осилить учебники элементарного анализа виновать ты, а не их авторы. Если ты не был тупый, то понял. С такой логикой тебе не стоит приблежатся к математике вообще.

>Не спорю есть уникумы на свете.


Эти уникумы обычные люди.
683 8707
>>8688
А то что тренды по матеше) тянутся на 70 трендов тебя не настораживает?
Ведь если бы были простые учебники, которые можно было бы сделать универсальным ответом на любой вопрос "что посмотреть из аниме? боко но пику", "двач я покакал, что делать? сделай бочку/сосни хуйцов", почему никто до сих пор не постил на вопросы о гауSSе-боннне какого то штурма или рому михайлова?
Или тут с первого же треда обсуждают теорию хаоса и вычисляют как взмах бабочки на одном континенте вызывает ураган на другом?

>>8699

>Чтобы открыть людям глаза на то, что не они виноваты в том что не понимают математику и остальные предметы, а виновны авторы учебников, учителя вузов и система образования.


Нет, виновато наследие совка. Сравни классический учебник по математике на русском языке и на английском. Дам подсказку, у саксонцев это томас, а у нас ебаный демидович.
684 8709
>>8707

>ебаный демидович


Это не учебник.
685 8713
>>8709
Да, это сборник задач. Есть у демидовича еще и краткое вкатывание в вышмат, но не про него сейчас.
Давай скажи тогда, что сейчас считается Ъ учебником по вышмату в МГУ, МГИМО, МФТИ, т.д.
686 8714
>>8707

>тянутся на 70 трендов тебя не настораживает?


Потому что мимокрокодилы приходят.
По анализу/калькулюсу есть уже устоявшийся ответ: Зельдович (для ПТУшников), Зорич (для обычных пацанов), Шварц (для умняш или знакомых в целом с другой математикой или второго знакомства с анализом)
687 8716
>>8702
Так выражения получаются разные.

Смотри, пусть A = {1,2}, B = {a,b}, C = {x,y}.
AxB = {(1,a), ... }
(AxB)xC = { ((1,a),x), ... }

BxC = {(a,x), ...}
Ax(BxC) = {(1,(a,x)), ... }

Пара ((1,a),x) не равна паре (1,(a,x)). Множества (AxB)xC и Ax(BxC) не могут быть равны.
688 8717
>>8713

>Давай скажи тогда, что сейчас считается Ъ учебником по вышмату в МГУ, МГИМО, МФТИ, т.д.


ФихтенгольцИльин-Позняк
689 8719
>>8714
Тогда почему ты ведешься на провокацию, как ты утверждаешь, и не советуешь

>Зельдович (для ПТУшников), Зорич (для обычных пацанов), Шварц (для умняш или знакомых в целом с другой математикой или второго знакомства с анализом)


?
Тебе больше нечем заняться, кроме как на сосачах двачевать? Ты чувствуешь себя мессией от лобаческого? Ты хочешь популярности уровня матх? К чему вся эта энтропия?
690 8720
>>8716
Всё верно. Но между ((a,b),c), (a, b, c) и (a, (b, c)) есть изомрофизм.
691 8721
>>8719
Я другой анон и Зельдовича тебе сразу посоветовал два дня назад. И три книги по мат. грамотности и курс на курсере элементарный. >>8371
692 8722
>>8721
А еще более другой анон который зашел на огонек.
693 8723
>>8720
Но ведь наличие изоморфизма не даст нам равенства. Ax(BxC) не будет равно (AxB)xC.
694 8725
В чём проблема понять учебник? Можно быть сколь угодно глупым человеком, но если не нашёл способа, при котором для тебя информация усваивается - это край, и порог вхождения не преодолеть. А если не искал способа понять - какого дьявола надо об этом ныть на борде?
Я прямо сейчас могу назвать как минимум 4 способа понять "непонятный" учебник:
1) просмотр/посещение лекций по предмету;
2) приведение примеров для данного определения - отличный метод, кстати;
3) конспектировать, перечитывать на свежую голову - понимание станет глубже;
4) посмотреть в другом учебнике или ином источнике, авось иной стиль изложения поможет понять.

Думаю, этот список с лёгкостью можно продолжить.
695 8726
>>8723
Строго говоря да, не будет равно)
696 8727
>>8726
Значит, декартово произведение не ассоциативно?
697 8728
>>8727
Верно.
698 8729
что посоветуете почитать по производным и интегралам?
699 8730
>>8729
Зельдович Я.Б., Яглом И.М. "Высшая математика для начинающих физиков и техников"
700 8731
>>8729
Халмош Теория меры
701 8733
>>8728
Ну охуеть теперь.
702 8734
>>8725

>В чём проблема выебать тян? Можно быть сколь угодно глупым человеком, но если не нашёл способа, при котором не можешь выебать тянку - это край, и ВОЗ ввела для таких как ты стату инвалида. А если не искал способа поебаться - какого дьявола надо об этом ныть на борде?


>Я прямо сейчас могу назвать как минимум 4 способа выебать тян:


>1) просмотр/посещение кино с тян;


>2) приведение тянки в ресторан- отличный метод, кстати;


>3) конспектировать, перечитывать переписку с тянкой на свежую голову - понимание станет глубже;


>4) посмотреть в ютубе или ином источнике пикап видео, авось иной стиль изложения поможет понять.


>


>Думаю, этот список с лёгкостью можно продолжить.



Посыл понял или нет?
Если нет, то скажи почему одним нужно таскать штангу на триста кило чтобы накачать мускул на пол сантиметра а у других мышцы от онанизма растут?
Почему у восьми людей на земле половина денег всего земного шара, а другие умирают от голода в гнусной халупе?
Почему у одних больных спидом в крови осталось полтора лейкоцита, но они не подхватывают даже простуду, а у полностью здоровых людей нос постоянно течет на ниагара?
703 8736
>>8734
Вот и всё. Ты сам ответил на свой вопрос, ты тупой и не способен заниматся математикой. Не трать время на картинки и поиск простых учебников. Уходи.
704 8737
>>8727
Но получившиеся множества равномощны, что в принципе от них и требуется.
форман.jpg60 Кб, 720x951
705 8738
>>8736
Ты так ничего и не понял и хочешь продолжать кидаться какашками.
Правильный ответ, потому что все люди разные.
Если ты будешь продолжать судить всех по себе ты нихуя не понял а я все понял значит ты дурак и не сможешь становится на чужое место ты так и будешь продолжать кидаться какашками в отстойниках типа сосачей.
706 8739
>>8737
Ну так и AxB равномощно BxA. Что, их тоже равными считать?
707 8740
>>8738
Тогда зачем ты кидаешься какашками в авторов учебников?
math.jpg158 Кб, 998x913
708 8741
https://www.tutorialspoint.com/discrete_mathematics/discrete_mathematics_tutorial.pdf

Вот нашел книгу где есть элементарные минимальные примеры, скрин прилагается. Не понимаю зачем вы дрочите на ущербное совкове образования и книги.
709 8742
>>8739
Равномощными, можешь проверить.
710 8743
>>8740
Ты конченый или окончательно конченый? Я тебе прямо указал на фатальные недостатки твоих же слов.
711 8744
>>8738

>Правильный ответ, потому что все люди разные


И кто-то не может осилить разжёванное для состояния каши.

>>8741

>посмотреть в другом учебнике или ином источнике, авось иной стиль изложения поможет понять

712 8745
>>8743
Долбаёбище, ты начал кидаться говно в авторов учебников. Причём, ты называл их тупыми дегенератами и после этого ты привел пример с тяночками. Это же пример показывает, что ты кретин и тебе не надо даже близко подходить к матеше, пусть этим займутся местные альфачи, а ты дрочи свои картинки в кулачок.
713 8746
http://alas.matf.bg.ac.rs/~mi10164/Materijali/DS.pdf
http://discretetext.oscarlevin.com/pdfs/dmoi-tablet.pdf

Вот еще, есть же элементарные примеры, зачем себе мозги задрачивать и терять попусту время.
714 8747
>>8744
Тогда почему ты до сих пор не мистер олимпия с заработком 300к в секунду и с напомаженной венерой вместо девушки? Ведь есть куча книг про успех, атлетизм и пикап в которых все разжевано до состояния каши?
715 8748
>>8745
Отлично.
Ответь тогда на один последний вопрос, почему учебники по вышмату продолжают упорно выпускать тысячи авторов по всему миру, если достаточно один раз написать мега учебник в котором будет все обо всем и про всех?
716 8749
>>8748
Потому что математика наука которая развивается и учебники устаревают.
717 8750
>>8747
Потому что

>куча книг про успех, атлетизм и пикап


наебалово на деньги. Говорю как прочитавший несколько книг об атлетике, и каждая гласила своё.
718 8751
>>8749
Какие новые математические теоремы открыли за прошлый год, из за которых стоило бы переписывать абсолютно все учебники по математике за позапрошлый год?
719 8752
>>8751
Дело тут скорей в технологическом прогрессе. Нахуй нужен учебник в котором все упражнения решаются через wolframalpha? Чему он научит человека?
720 8753
>>8750
Почему тогда математика это не наебалово на деньги? Потому что её можно проверить, так?!
Что тогда тебе мешает проверить врут или нет про то что за 10 недель можно увеличить количество подтягиваний с нуля до пятидесяти?
721 8754
>>8752
Зачем тогда вообще нужна математика если все компьютер в маткаде посчитать может? Чему она научит человека, если сборище кремния, меди и пластика может сделать то же самое но гораздо быстрее и качественее?
722 8755
Вкуснотища.
723 8756
>>8753
Потому что на результатах математики и областей, её использующих, держится нынешняя человеческая среда обитания, не? Ты какой-то ебанутый.
Алсо, проверял - за 10 недель не достичь.
724 8757
>>8754
Тогда зачем тебе её учить? Зачем человеку делать игры, если тоже самое может сделать компуктер гораздо быстрее и качественее?
725 8758
>>8703

>Смотри, люди понимают учебник, а ты особенный тебе нужен специальный учебник для даунов. Ты не можешь учится по нормальным, потому что они слишком сложны для тебя. Суть - ты тупой. ТУПОЙ.


>Какой информации? Что ты говоришь, кретин. То, что для нормальных людей ОЧЕВИДНО, то для тебя нет. Тебе нужны дополнительные объяснения.



Я не хочу тратить время, задрачивая инопланетянские иероглифы и терять зрения и приобретать головные боли от перенапряжения, так само другие люди не хотят, информацию можно подать более простым способом постепенно увеличивая сложность, а не сразу с 10 страницы выдавать несуразную хуету.

>Объясни с чего это вдруг они некомпетентные?


Не могут подать информацию на элементарных примерах хотя бы числовыми массивами, или объектами из мира.

>Эти уникумы обычные люди.


Бетховен и Моцарт тоже обычные и без задней мысли вот так просто и начали сочинять музыку, да?
726 8759
>>8754
Математика не учит считать, а учит понимать. Каждый второкурсник должен понимать доказательство теоремы Ферма, а не то, как считать интеграл.
727 8760
>>8758

>Бетховен и Моцарт тоже обычные и без задней мысли вот так просто и начали сочинять музыку, да?


https://en.wikipedia.org/wiki/Category:18th-century_classical_composers
просто без задней мысли посвятили музыке всю свою жизнь.
728 8761
>>8760
Много людей посвящает музыке жизнь но не все добиваются успеха, есть же особый талант, генетика.
729 8762
>>8742
Да, но суть ассоциативности не в равномощности, а в равенстве.
Почему тогда в учебниках пишут, что декартово произведение ассоциативно?
730 8763
>>8756
Не на математике, на моделях использующих математику. Если скажешь что нет, то найди мне в природе закон кулона/всемирного тяготения/любой другой.
Ну и кому ты врешь? Почему я за два с половиной года научился подтягиваться на одной руке с поддержкой, а ты не смог в элементарные обычные подтягивания?

>>8757
Зачем человеку вообще что то делать если найдется кто то кто сделает что то лучше быстрее и качественнее чем он?

>>8759
И какой прок с теоремы ферма? Что ты будешь с ней делать? Показывать богу/пришельцам чтобы они охуели как мы можем? Прекращай толстить, я начинаю терять интерес к этой дискуссии, которая на самом деле не дискуссия, а дебаты.
731 8764
>>8758

>Бетховен и Моцарт тоже обычные и без задней мысли вот так просто и начали сочинять музыку, да?


Почему ты приводишь в пример гениев? Разне нужно быть гением, чтобы по Фихтенгольцу вкактиться в матан? Нет.

>Не могут подать информацию на элементарных примерах хотя бы числовыми массивами, или объектами из мира.


У тебя наивное предстваление о математике. Я ещё раз спрашиваю, зачем тебе вообще заниматься матешей, если ты ничего не понимаешь без примеров, там где можно без них обойтись?

>Я не хочу тратить время, задрачивая инопланетянские иероглифы и терять зрения и приобретать головные боли от перенапряжения, так само другие люди не хотят, информацию можно подать более простым способом постепенно увеличивая сложность, а не сразу с 10 страницы выдавать несуразную хуету.


У тебя нет знаний, чтобы вкатиться в матан? Если для тебя написанная человеческим языком вводная часть - несуразная хуета, то ты даун.
>>8761
А теперь ссылку на пруфы, что есть ген с понимаем математике. А хотя, постой. Я понел, у тебя плохая генеита, лишняя хромосома все дела. Я тебе не завидую.
732 8765
>>8763

>Зачем человеку вообще что то делать если найдется кто то кто сделает что то лучше быстрее и качественнее чем он?


Вот и не занимайся матешей. Всё просто. Уходи от сюда. И не возвращайся.
733 8766
>>8765
Только с тобой, сынок.
734 8767
>>8763

>Если скажешь что нет, то найди мне в природе закон кулона/всемирного тяготения/любой другой.


Проведение соотвестующего экспермента.
735 8768
>>8763

>И какой прок с теоремы ферма? Что ты будешь с ней делать?


Это не раздел для инженеров, уебывай. Открой, например, arxiv.org, почитай чем занимаются математики.
736 8769
>>8751
В учебниках в принципе не отражено ничего из того, в чем были совершены открытия в прошлом году. Никакие открытия в науке не могут изменить эти учебники. Следовательно, все эти учебники - говно мамонта.
737 8770
>>8766
Я тебе в отцы гожусь, чёрт малой. Пиздуй от сюда смотреть лекции для семиклассников.
738 8771
>>8767
И он даст тебе какие то данные. И ты эти данные будешь использовать для того чтобы предсказать некие явления.
Но самого закона ампера/кулона/т.д. в мире нет. Есть только математический образ того что мы называем законом ампера/кулона/т.д.
739 8772
>>8771
У тебя не только с математикой проблеммы но и физикой. Как ты это докажешь?
740 8773
>>8768
Ясно.

>>8769
Молодец, довел до абсурда и рад что не понял сути той мысли которую я пытался до тебя донести.

>>8770
Тебе 55? Потому что меньше чем в 15 лет мало кто может поднять свой писюн силой мысли, а еще меньше сможет вставить этот писюн в нужное место чтобы дети были.

К чему я это? Надоели вы мне. Пойду я спать.
741 8774
>>8773
В норме учебник - это компиляция свежих и интересных статей, дополненная справочной информацией, такой как общеизвестные определения и обозначения. Учебник по общей топологии должен меняться каждые два-три года, например.
742 8775
>>8774
Ну и расскажи, что должно быть в учебнике 2017. Изложение через локали?
В общей топологии уж лет 40 ничего не происходит.
bitva.png12 Кб, 468x462
743 8776
744 8778
>>8775
Должен быть обзор всех тех идей, которыми пользовались авторы https://arxiv.org/list/math.GN/16
Или хотя бы самых перспективных из них.
745 8779
>>8762
Можно наверное назвать это абузом нотации. Типа чтобы не писать каждый раз "есть биекция между (a, b, c), ((a, b), c) и (a, (b, c))". А в каких учебниках пишут, что ассоциативно? Открыл Мункресову Топологию, так такого нет. На википедии тоже явно сказано, что не ассоциативно.
746 8780
>>8772
Два тела притягиваются друг к другу с силой прямо пропорциональной массам тел и обратно пропорциональной квадрату расстояния между ними. Это называется законом всемирного притяжения.

Этот закон основывается на трех законах кеплера
-каждая планета вращается по эллипсу в одном из фокусов находится солнце
-за равные промежутки времени радиус вектор ометает одинаковые площади эллипса
-квадраты периодов обращения планет относятся как кубы больших полуосей орбит

Законы кеплера основываются на многолетних наблюдениях браге выполненных со всей возможной точностью деревянных визиров и телескопов циклопических размеров собранных вручную и скрупулезной записью всех наблюдений в специальную книжку с записью времени.

Теперь найди момент в который появилось то что мы называем законом всемирного притяжения.
747 8781
>>8758
Да, Моцарт в пять лет без задней мысли внезапно взял и начал писать музыку. Ты бы сперва хоть немного узнал про тех, чьи фамилии употребляешь.
748 8782
>>8762
ассоциативно для равных множеств.
749 8783
А почему вы считаете, что любой учебник обязательно должен быть ориентирован на дурачков? У нас наука или богадельня вообще?
750 8784
>>8764

>Разне нужно быть гением, чтобы по Фихтенгольцу вкактиться в матан? Нет.


Даун это ты, не понимаешь что от качества изложения материала зависит качество знаний и усвояемости информации, это все люди понимают кроме такого дебила как ты.

> Я ещё раз спрашиваю, зачем тебе вообще заниматься матешей, если ты ничего не понимаешь без примеров, там где можно без них обойтись?


Может я хочу программировать AI для игр, но нужно наличия знаний в теории графов.

>А теперь ссылку на пруфы, что есть ген с понимаем математике. А хотя, постой. Я понел, у тебя плохая генеита, лишняя хромосома все дела. Я тебе не завидую.



http://www.nat-geo.ru/science/47520-obnaruzhen-gen-otvechayushchiy-za-genialnost/
751 8785
>>8783
Учебник - это не научная статья. Учебник ставит своей целью научить, а не просто сообщить информацию.
752 8786
>>8778
К сожалению эти авторы пользуются идеями, которые есть в каждом учебнике по топологии: у них они даже в референсах.

Вот тебе аналогия с машиной: ты хочешь собрать гоночный болид. Тебе нужно идти на формулу-1? Нет, тебе прежде всего надо знать на зубок аэродинамику, механику, устройство ДВС... А потом уже ты сможешь приступать к современной науке.
753 8787
>>8785
Научить ставит своей целью ученик. Мне, например, показался дебильным учебник Зорича и я начал читать Шварца. А учебник Лэнга мне показался слишком сложным и я пользовался сразу несколькими дополнительными источниками, чтобы лучше усвоить материал.
754 8788
>>8787
Чего? Научить должен учитель, а ученику надо научиться.
755 8789
>>8784

>http://www.nat-geo.ru/science/47520-obnaruzhen-gen-otvechayushchiy-za-genialnost/


Я говорил про ген с понимаем математики а не гениальности. Но ты кретин и ничего так и не понял.

>Даун это ты, не понимаешь что от качества изложения материала зависит качество знаний и усвояемости информации, это все люди понимают кроме такого дебила как ты.


Еблан, плиз. В том-то и дело, что качество изложения хорошее. Просто ты тупой и не можешь въехать.

>Может я хочу программировать AI для игр, но нужно наличия знаний в теории графов.


Даже не думай, тебе столько придёться испытать боли тогда.
756 8790
>>8785
У каждого дискурса есть свой уровень сложности. Есть самый сложный уровень, обозначим его филдс-0. Есть уровень на 1 меньше самого сложного, филдс-1. Есть уровень на 2 меньше самого сложного, филдс-2. И так далее до уровня детсада.

Очевидно, что учебник может поднять уровень дискурса читателя самое большее на единицу. Учебник не может поднять уровень сразу на десяток ступеней. Нельзя написать учебник на языке дошкольников, превращающий читателя в филдсовского лауреата.

Если мы готовим учебники для филдсовских лауреатов, то они должны быть написаны на языке филдс-1. Уже филдс-2 будет недостаточно выразительным.
757 8791
>>8784

>Может я хочу программировать AI для игр, но нужно наличия знаний в теории графов.


Бери учебник по AI: в одном из них стопудово есть необходимые данные по теории графов. Можешь взять книгу "Оре Теория графов" и прочитать оттуда где-то 5-7 глав, тебе хватит с головой.
758 8792
>>8790
И что?
759 8793
>>8788
Если ты не можешь сменить учителя, то ты виноват именно ты. Учитель ничего не должен, он просто объект этого мира.
760 8794
>>8792
Это значит, что хайлевельный учебник и должен выглядеть очень сложным. Это нормально. Он же хайлевельный.
761 8795
>>8793
Чего блядь? Ты выше написал, что "научить ставит своей целью ученик". Я тебя поправил: учитель, а не ученик.
762 8796
>>8794
Но вузовский учебник не может быть хайлевельным по определению. Это раз. Называть дураками всех, кто в данный момент находится ниже филдса-эн - ошиюка и провокация. Это два.
763 8797
>>8795
Научить себя = научиться.
764 8798
>>8796
Хайлевельный - понятие относительное. Если уровень твоего дискурса филдс-n, то хайлевельные дискурсы - это дискурсы с номерами меньше чем n-1. Для филдс-50 уже филдс-48 будет хайлевельным.
765 8799
>>8796
Может. И что у тебя за определение такое? Это раз.
Называть аторов учебников дегенератами и дебилами потому что ты ничего не понял - это ошибка и провокация. Это два.
766 8800
>>8797
Ну так ученик здесь выступает в качестве учителя, просто процесс замкнут на себя. Понятия все равно разные.
767 8801
>>8799
Я и не называл никого дебилами. Я воспитанный человек.
768 8802
>>8801
Не ври. Называл.
769 8803
>>8798
Нет, эн-хайлевельные дискурсы - те, что лежат в эн-окрестности филдса-ноль. А ты говоришь об хайер-вен-левельных дискурсах.
770 8804
>>8802
Не ври. Процитируй.
771 8805
>>8803
Суть в том, что если вузовские учебники кажутся тебе сложными, то уровень твоего дискурса отстоит от филдс-вузик как минимум на две ступени.
772 8806
>>8805
При чем тут я вообще?
773 8807
>>8806
s/тебе/кое-кому/g
774 8808
775 8809
>>8807
Сложный и непонятный, впрочем, разные вещи. Как и дурак - нуб.
776 8810
>>8789

>Я говорил про ген с понимаем математики а не гениальности. Но ты кретин и ничего так и не понял.


Ты тупой, я говорил про ген который хорошо помогает в усваивании информации мозгом, а не математики.

>Еблан, плиз. В том-то и дело, что качество изложения хорошее. Просто ты тупой и не можешь въехать.


Какое должно быть качество изложения материала как минимум я предоставлял пример выше, идиота ты кусок.

>Даже не думай, тебе столько придёться испытать боли тогда.


Почему?
777 8811
>>8810

>Почему?


Ты же тупой, ничего не поймёшь и пойдёшь плакаться сюда.
778 8812
Сущестует ли в математике (да, я не ошибся) понятие минимальной ясности для решения задачи?
779 8813
>>8784
Предлагаю лично тебе забить на этого >>8789 хуесоса и заняться тем дерьмом что и я. Вводить понятие минимального понимания материала для тот кто не знаком с какой либо "предыдущей" матчастью всё понял (либо, если тут я зафейлился) просто какую-то минимальную единицу понимания, которой будет достаточно.
Ибо усложнить может каждый ебаный пидрила на этой планетке
780 8814
>>8813

>для тот


Для того чтобы тот.
781 8820
>>8812
Существует
782 8822
this >>8776 итог треда
А теперь, ленивая жопа, пили перекат.
blob245 Кб, 420x577
783 8825
почему это не будет работать?
784 8826
>>8825
Потому что длина предельной кривой неравна пределу длин кривых.
785 8850
Есть ли у кого пояснение дробей по хардкору? Нет, ну, я вроде представляю в общем виде как с ними работать, но не понимаю СУТИ дробей. Ну, и еще десятичные тоже интересуют.
786 8852
А я могу быть крутым математиком, если понимаю в верху абстрактной алгебры, но путаюсь при делению в столбик?
s-PomanMihailovDSCF1408.jpg27 Кб, 1024x684
787 8855
>>8763

>Не на математике, на моделях использующих математику.


Когда читать научишься, мудила? Я написал чётко и ясно - на результатах математики и областей, её использующих. Если ты, дегенерат, не способен дойти до того, что матмодели входят в это всё - то неудивительно, что ты не в силах осилить простенькие классические учебники.

>за два с половиной года научился подтягиваться


>10 недель


>2,5 месяца


>2,5 года


Ты реально тупой.
788 8856
>>8825
По построению предел длиный этой кривой не стремится к длинне окружности.

Погугли "square pi = 4".
789 8857
>>8606
Бамп
790 8858
Почему и зачем 0! равно 1?
791 8860
>>8858
По определению.
n!=(n-1)! * n
Подставь n=1.
792 8861
Я понял, вся математическая система себя изжила и уже неактуальна для нынешнего времени, вы пользуетесь устаревавшими понятиями и значениями когда нужно водить новые более понятные и доходчивые. Нахрена модернизировать паровую машину если есть электро автомобили. Когда создавалась высшая математика, бумага было дорогая и было невыгодно упрощать материал так как это может занять не 2 странницы а 4, не было и графических и видео материалов. Сейчас новые времена и нужна новая математика а не то гавно которым вы сейчас пользуетесь.
793 8862
>>8861
А вот и реформатор подъехал!
794 8864
>>8861
Кто-нибудь из людей, думающих так же как и ты, понимает уравнения общей теории относительности?
795 8865
>>8861
Всё правильно, математика постоянно модернизирует свой язык. Например, анализ перевели на язык топологии, соеденив огромные математические области вместе. Теперь не нужно дрочить эпсилон-дельта и решать интегральчики 5 лет, теперь можно прочитать Шварца, а интегральчики компухтер посчитает.

То, на что умнейшие люди тратили десятки лет занятий с дорогим персональным учителем, который рисовал картинки и простые примеры, можно изучить, прочитав 200 страниц Шварца.
796 8866
>>8861
Во первых, когда она создавалась не было того формального языка, а почти всё было написанно с картинками и объяснено ими, по образу начал Евклида. Ты не знаешь, о чём говоришь, самый тупой посетитель /math.
797 8867
Вас не заебало с ним спорить?
798 8869
>>8867
Нет, пища же.
799 8870
Привет! Учебник по теории множеств посоветуйте, пжлст. Про лемму Цорна, аксиомы выбора, вот это все. Что-нибудь краткое и простое, с основными понятиями. А то Бурбаки СЛОЖНА.
800 8872
>>8867
Блять, меня заебало, вот с Лощининым я никогда не пререкаюсь, а тут все анонимно и я не могу остановиться. Может прикреплять айди к постерам и менять его раз в тред?
801 8873
>>8870
В учебнике по топологии Вербицкого доказывается эквивалентность леммы Цорна, аксиомы выбора и теоремы Цермело. Еще там есть теорема Кантора и теорема Кантора-Бернштейна.
802 8874
>>8873
Вербицкого? Я чет очкую, он же поехавший, считающий, что восьмиклассники должны теорию Галуа знать. Что-нибудь энтри левела есть?
803 8876
>>8864
Я не знаю таких людей.

>>8865

>прочитав 200 страниц Шварца.


Посмотрел я шварца нету там простых примеров, значит книга устаревшая и неактуальна.
804 8877
>>8874
Не бойся, сладкий. В жизни всё надо попробовать
805 8879
>>8876

>Посмотрел я шварца нету там простых примеров, значит книга устаревшая и неактуальна.


Да, верно их там нет. Как раз потому что она относительно свежая. Хотелось бы, чтобы была такая же, но на языке модулей над кольцом.

А простые примеры Евклид давал, вот он как раз устарел. Возвращаться к такому регресс и деградация. А деграданты всегда могут пойти в сантехники и неплохо зарабатывать. Интеллект же не высшее благо для всех, сантехники тоже нужны и должны быть уважаемы в обществе.
806 8880
>>8874
Там есть пару опечаток, но в целом читабельно. Если задачи некому показывать, то не советую сильно увлекаться, а то это может растянутся на месяцы.
807 8881
>>8876

>Я не знаю таких людей.


Умелый маневр. Ну то есть так же как и ты думают только малообразованные слабые в математике личности или такие мысли приходили в голову и ученым-математикам?
808 8882
>>8406
Опчик, ты на что-то можешь повлиять в техническом плане? Ну скажем система лайков как в апачане тут была бы замечательной.
809 8884
>>8877
>>8880
>>8873

Спасибо.
810 8886
>>8703
Извиняюсь, что немного, хм, коснусь, если так можно выразиться вашей с тем требующим пояснений аноном, дискуссии, но судя по тому, как ты с ним разговариваешь, сколько брани, направленной злобы ты на него льёшь, ты сам ни чуть не лучше. И хотя я согласен с тем тезисом, что пишутся учебники для математиков, кто хочет и понимать и заниматься математикой, тогда, если взглянуть и на твоём замечание о ничтожно малом кол-ве математиков от всего кол-ва людей на этой земле, и то, что ты говоришь "нормальные люди понимают математику, учась по этим учебникам", тогда кого ты имеешь в виду? Нормальные это те, кто учится математике, чтобы стать математиком или те, кто не хочет быть математиком, но некоторые математические методы ему требуются для решения его локальных задач, в его науке? Потому что первых - очень мало, как ты заметил, а вторых куда больше, но, вот незадача, им тоже ненужна значительная часть знаний, предоставляемых математикой, отчасти потому что они не могут понять их, или из-за каких-то иных причин, но это не делает их плохими специалистами в своей области, не делает их, как ты говоришь, тупыми, они тоже НОРМАЛЬНЫЕ люди. Или по-твоему они должны быть признаны ненормальными, потому что не поняли (или для них есть проблемы с пониманием) какую-то часть анализа, но вот какой-то другой раздел им дался лучше, и они вполне себе хорошие научные сотрудники. Ты всё еще будешь считать их ненормальными? Потому как, ты уж извини, подавляющее число людей на этой земле вполне здоровые психически и физически, следовательно, нормальные люди, но вот математику они не изучают дальше high school+колледж/университет (какой-то базовый уровень университетской математики и всё, на этом всё, дальше только специализированные мат.методы и разделы, но это куда меньше чем то, что осваивает и должен освоить математик). Причем я взял сразу как людей, изучающих какие-нибудь arts, humanities, social sciences так и людей которые связаны с natural sciences (коме математиков). Многие 1-ой категорий лиц могут тебе сказать, что они либо не понимают математику, либо очень многое не понимают, кроме некоторых методов, которые и используют в своих исследованиях, но они что... тупые? Слишком смелое заявление, правда? И ведь на этой планете куда больше чем самих математиков, их - тех самых нормальных людей.

Может ты и классный математик, может ты просто с математическим умом человек, но общение с тобой - наказание для человека. И ты очень груб и озлоблен, чего кстати, о других анонах из треда я не скажу. Извини, что вмешался в диалог.
810 8886
>>8703
Извиняюсь, что немного, хм, коснусь, если так можно выразиться вашей с тем требующим пояснений аноном, дискуссии, но судя по тому, как ты с ним разговариваешь, сколько брани, направленной злобы ты на него льёшь, ты сам ни чуть не лучше. И хотя я согласен с тем тезисом, что пишутся учебники для математиков, кто хочет и понимать и заниматься математикой, тогда, если взглянуть и на твоём замечание о ничтожно малом кол-ве математиков от всего кол-ва людей на этой земле, и то, что ты говоришь "нормальные люди понимают математику, учась по этим учебникам", тогда кого ты имеешь в виду? Нормальные это те, кто учится математике, чтобы стать математиком или те, кто не хочет быть математиком, но некоторые математические методы ему требуются для решения его локальных задач, в его науке? Потому что первых - очень мало, как ты заметил, а вторых куда больше, но, вот незадача, им тоже ненужна значительная часть знаний, предоставляемых математикой, отчасти потому что они не могут понять их, или из-за каких-то иных причин, но это не делает их плохими специалистами в своей области, не делает их, как ты говоришь, тупыми, они тоже НОРМАЛЬНЫЕ люди. Или по-твоему они должны быть признаны ненормальными, потому что не поняли (или для них есть проблемы с пониманием) какую-то часть анализа, но вот какой-то другой раздел им дался лучше, и они вполне себе хорошие научные сотрудники. Ты всё еще будешь считать их ненормальными? Потому как, ты уж извини, подавляющее число людей на этой земле вполне здоровые психически и физически, следовательно, нормальные люди, но вот математику они не изучают дальше high school+колледж/университет (какой-то базовый уровень университетской математики и всё, на этом всё, дальше только специализированные мат.методы и разделы, но это куда меньше чем то, что осваивает и должен освоить математик). Причем я взял сразу как людей, изучающих какие-нибудь arts, humanities, social sciences так и людей которые связаны с natural sciences (коме математиков). Многие 1-ой категорий лиц могут тебе сказать, что они либо не понимают математику, либо очень многое не понимают, кроме некоторых методов, которые и используют в своих исследованиях, но они что... тупые? Слишком смелое заявление, правда? И ведь на этой планете куда больше чем самих математиков, их - тех самых нормальных людей.

Может ты и классный математик, может ты просто с математическим умом человек, но общение с тобой - наказание для человека. И ты очень груб и озлоблен, чего кстати, о других анонах из треда я не скажу. Извини, что вмешался в диалог.
811 8887
>>8879

>Возвращаться к такому регресс и деградация.


Да неужели, вот тут ты ошибаешься, посмотри на скрины и увидишь что это намного быстрее помогает понять и усвоить информацию чем текстовая писанина.

>>8881
Я в голову им не лазил и мысли не читал.

Вы все ошибаетесь, вы некомпетентные неучи. Я скинул пруфы смотрите, можно приставить математические понятия простым языком и объяснениями. Какой же у вас манямирок сильный что не хотите видеть очевидные вещи.
812 8888
>>8866
Неправда, самой тупой это я.
813 8889
>>8888
Квадрипл говорит правду.
814 8890
>>8886
Я просто бомбанул из-за того, что он стал наезжать на авторов учебников и называть их тупыми деградантами.
815 8891
>>8888
Гет.
816 8892
>>8887
Ну я не видел, чтобы кто-нибудь из людей, понимающих, например, общую теорию относительности, утверждал то же что и ты, напротив, они пишут статьи и книги без таких вот картинок. И ты таких людей назвать не можешь. Почему при этом прав должен быть ты?
817 8893
>>8887
По такому учебнику я бы учился раз в 10 дольше, чем по нормальному. Как раз как в древние времена. Сколько цветастой хуеты перед глазами и непонятно, где главные идеи. Зато написано, что + это плюс. Ахуеть, спасибо, что потратили моё время.
818 8894
>>8887
А где на первой картинке объяснено, что такое число пи? Там просто под ним написано, что вот три значащие цифры, а более точное значение можно посмотреть в калькуляторе. Это по-твоему удовлетворительное объяснение?
819 8896
>>6879 (OP)
Пусть есть число 0,6541651.
Как можно найти целое делимое и делитель, которые при делении дадут частное, равное этому числу?
820 8897
>>8896
Умнож на 10000000, а потом сократи дробь.
821 8898
>>8897
можно и не сокращать
822 8899
>>8896
>>8897
>>8898
Ничего не пойму.
Алгоритм бабушкина:
Алгоритм™ архивации таков: любой файл представляет собой HEX-последовательность символов, переводим этот HEX в DEC, получаем неебически-большое число, дописываем перед этим число 0, — получаем число в диапазоне от 0 до 1 с огромным числом знаков после запятой, а дальше всё просто — подбираем 2 таких целочисленных числа, частное которых даст нам искомое число в диапазоне от 0 до 1 с точностью совпадений до последнего знака. Беда в подборе чисел, которое может идти и 2 часа, а может идти и 2 недели. Есть опытные образцы и работающая программа, и всё это работает.

Пусть:
1. hex = 63D153
2. в dec = 6541651
3. 0 спереди = 0,6541651 (получилось число от 0 до 1).
4. умножить на 10000000 = 6541651 = dec = hex.
5. Получается, делимое 6541651, делитель 1, ну или кратные им числа.

В чём тогда суть архивации?
Гидроцефал.png59 Кб, 800x600
823 8900
Каких только долбоёбов не заносит сюда. То ли ещё будет весной.
824 8901
>>8900

>отрицать что учебники советские - говно.


Ты болен?
825 8902
>>8899
Делимое = 6541651, делитель = 10000000. И это блядь просто десятичная дробь. Бабушкин поехавший.
826 8903
>>8892

>кто-нибудь из людей, понимающих, например, общую теорию относительности, утверждал то же что и ты, напротив, они пишут статьи и книги без таких вот картинок.


Значит они пишут книги не с целю научить, а для таких самых ученых как и они.

>>8893

>По такому учебнику я бы учился раз в 10 дольше, чем по нормальному.


Значит ты тупой, эту книгу как раз писали для детей. Так как логически и доходчиво все объяснено, читать интересно и увлекательно.

Смотри скрин с книги зорича, там нихуя непонятно, про что написано где логическая цепочка и примеры, голова болит чтобы разобрать про что написано и сопоставить для себя логику, уйдет много драгоценного времени, хуета одна, такую книгу может и обезьяна нацарапать на дереве.

>>8894

>А где на первой картинке объяснено, что такое число пи?


>Это по-твоему удовлетворительное объяснение?


Скачай книгу и прочитай.
Усложнять материал нужно маленькими шагами и только постепенно увеличивая сложность, для начала и такого объяснения хватит, не перегружая мозг лишней информацией.
827 8904
>>8903

>Скачай книгу и прочитай


Я уже скачивал две книги, в которых как мне утверждали отсутствует теория множеств, а она там есть. Так что давай скрин определения числа пи из этого волшебного учебника.
828 8905
>>8903

>Значит они пишут книги не с целю научить, а для таких самых ученых как и они.


Учебники по математике, применяющейся в теории относительно, написаны для студентов.
829 8906
>>8901
А ты не иначе как дежуришь тут. Нормальный человек за то время, пока ты тут срёшь, уже осилил бы треть "Теоремы Абеля в задачах и решениях".
830 8907
>>8903

>Значит они пишут книги не с целю научить, а для таких самых ученых как и они.


А как они сами изучили теорию относительности?
831 8908
Как вообще люди читают книжки без картинок?
832 8909
>>8903

>Значит ты тупой, эту книгу как раз писали для детей.


Верно, книги для детей должны читать дети. Книги для взрослых взрослые.
Вот ты — интеллектуальный ребёнок и тебе нужны картинки, сам их ты нарисовать не способен.
833 8910
>>8903
Как человек с коэффициентом интеллекта 97 сможет написать хороший искусственный интеллект?
834 8912
>>8910
Так пусть сделает плохой.
835 8913
>>8910

>сможет написать хороший искусственный интеллект?


А где твой искусственны интеллект? И толку с того что у тебя IQ чуть больше 97, если ты не можешь его применить.
836 8914
>>8912
Ага, главное чтоб тут перестал срать.
6ec[1].jpg23 Кб, 533x361
837 8915
>>8902

>делитель = 10000000


Да, он такой.
838 8916
>>8900
Так ведь правда говно.
839 8917
>>8915
Такой ага, что-то не так?
840 8918
Почему программы не могут быть как дроби? Сократил дробь, а программа тоже самое исполняет.
841 8919
>>8907

>А как они сами изучили теорию относительности?


На это влияет много факторов, может им родители нанимали репетиторов которые доходчиво и с картинками им объясняли, или родители тоже были учеными, может когда учились и были студентами собирались группами и расшифровывали учебники.
842 8920
>>8919
Да, либо это, либо это ты глупый. Ну, если не ученые единомышленники, то хотя бы просто сторонники твоих идей относительно учебников имеются? Кто эти люди?
843 8921
>>8919
А как эти репетиторы и родители-ученые изучили теорию относительности?
844 8922
>>8909

>Верно, книги для детей должны читать дети. Книги для взрослых взрослые.


Учебники пишутся с целю НАУЧИТЬ. Книгу которую я скидывал поймет и ребенок и взрослый, книги которые мне рекомендовали, не каждый взрослый поймет.
845 8923
>>8917

>Такой ага, что-то не так?


Ну, у меня единичка там.
846 8924
>>8922
Ты уже нашел в этом учебнике определение числа пи?
847 8925
>>8924

>ПОПУЛЯРНАЯ КНИГА ДЛЯ ДЕТЕЙ


>ОПРЕДЕЛЕНИЕ ЧИСЛА ПИ


>ПОПУЛЯРНАЯ КНИГА


>ОПРЕДЕЛЕНИЕ

math.jpg381 Кб, 1522x915
848 8926
849 8927
Простите, господа. А может кто-нибудь открыто сказать, является ли он не только участником срачей в /math/, а также тралем в треде для начинающих, а настоящим математиком? Вообще тут есть настоящие математики? А то такие по античному мужественные дела творятся в треде, решаются судьбы, ставятся метки по типу "ты тупой", "ты дебил", "учебники пишут дураки", хочется знать эти все люди вообще понимают что пишут и являются ли они представителями математического сообщества.
850 8928
>>8922
Взрослому детские книги плохо читать. Много воды, рассказывают не всё и не точно. Скучно, очень долго и некрасиво. Взрослому хочется от такого блевать и он начинает ненавидеть предмет. Это как смотреть телепузиков, когда тебе 25. Очень хочется блевать.
851 8929
>>8927

>"ты тупой", "ты дебил", "учебники пишут дураки", хочется знать эти все люди вообще понимают что пишут и являются ли они представителями математического сообщества.


Конкретно картинкоблядь — не математик. Он веб-макака, которая хочет в гейм-дев (как и все ментальные подростки)
852 8930
>>8927
Есть задокументированные свидетельства присутствия тут настоящих ученых, но вряд ли они участвуют в этих срачах. Я для развлечения тут. Вкидываю совершенно абсурдные и оскорбительно нелепые сообщения и наслаждаюсь.
853 8931
>>8926
Смотри.

Окружность -- множество точек на плоскости, равноудаленных от данной, называемой центром. Расстояние от центра до любой точки окружности называется радиусом. Диаметр -- максимальное расстояние между точками окружности. Пи -- это отношение длины окружности к диаметру.

На усвоение этого уходит одна условная единица времени. Сколько тебе потребуется таких условных единиц, чтобы усвоить приведенный тобой оборот?
854 8932
>>8929
Ну тогда многие из веб-дева и геймдев индустрии ментальные подростки, которые, мягко говоря, не дружат с рациональным "само я" и очень засоряют свою технологическую сферу крайне не оптимизированными, дурными решениями. Потому что так оно в сфере IT и происходит. Конкретно тот, о котором ты говоришь, соглашусь, парень немного странноватый и назойливый, но все его посты можно объединить одним вопросом, я думаю, "где есть математика в в нашем, человеческом мире?" В этих вот немного неумелых стараниях в треде он пытается увидеть математическую сущность того, что читает, что пытается изучить, и как-то связать её со своим разумом. Может все эти вопросы от того, что как-то не выработана какая-то научная интуиция, чувство поиска прекрасного и поразительного в явлениях, структурах, системах?

>>8930
А, спасибо, понял. Энтузиаст и шутник?
855 8933
>>8906
Ты не ответил на вопрос, ты болен, раз считаешь совковые учебники годными по сравнению с пиндосскими?
856 8934
>>8933
Посыл, характеризующийся этой фразой про учебники был впервые упомянут в этом треде двежелезяки-дебилом. Если учитывать контекст, то ее суть в том, что переведенный советским издательством на русский язык анализ Шварца или тот же Зорич хуже ежегодного переиздания пиндосского Томаса или Стеорта. Я так не считаю. Но в совке выходило картафанное говно, да, с этим никто спорить тут не собирается.
857 8937
>>8931

>На усвоение этого уходит одна условная единица времени.


Тому кто первый раз это читает нужно раз десять прочитать и время чтобы понять, как минимум 100 единиц времени.

А взгляд на картинку ты сразу воспринимаешь информацию без чтения мгновенно. Займет это 0.01 единиц времени.
858 8938
>>8937
То есть получается, ты оцениваешь две строчки в 100 единиц времени, а эти две страницы -- в 0.01 единицы времени. То есть в десять тысяч раз легче? Ты считаешь себя глупым человеком?
859 8939
>>8928

>Взрослому детские книги плохо читать. Много воды, рассказывают не всё и не точно.


Именно ту книгу которою я скидывал, там мало воды и все конкретно написано что помогает усвоить и осознать материал.

>Взрослому хочется от такого блевать и он начинает ненавидеть предмет.


Мне 25 лет и мне хочется блевать от книг зорича, шварца и иже сними, и ненавидеть предмет.
860 8940
>>8938
Те две строчки не дают ту полноту информации чем те две страницы.

>Ты считаешь себя глупым человеком?


Нет не считаю.
861 8941
>>8939

>Мне 25 лет и мне хочется блевать от книг зорича, шварца и иже сними, и ненавидеть предмет.


Это и не удивительно, ты в 25 лет дошел до уровня седьмого класса.
862 8942
>>8940

>Нет не считаю.


В плане математики ты глупый человек?
863 8943
>>8939

>Мне 25 лет и мне хочется блевать от книг зорича, шварца и иже сними, и ненавидеть предмет.


Всё верно, потому что ты интеллектуальный ребёнок. 5-классник в теле 25-ти летнего жлоба.
864 8944
>>8940

>Те две строчки не дают ту полноту информации чем те две страницы.


То есть эти две строчки сложнее более чем в десять тысяч раз? Ведь из них нужно выводить и додумывать всякие вещи, которые понятны только гениальным детям? Так?
865 8946
>>8900
Вас (местных дол бой обов) просто жирно троллят. То ли еще веснгой будет - обос трение, все дела.
866 8947
>>8946
А при чем тут весна?
867 8948
>>8947
Ну, мы идем, понимаешь?
868 8950
>>8948
не понимаю
869 8951
>>8950
Ну, ты в тишине посиди, поразмысли над этим.
870 8953
>>8951
Ты можешь мне объяснить, где в физике применяются теории множеств?
871 8954
>>8953
Голова у меня слишком седая, чтобы объяснять такие вещи.
872 8955
>>8852

>путаюсь при делению в столбик


Это значит что у тебя с памятью проблемы.
873 8956
>>8954
а что такое гогоморфзм?
874 8957
>>8956
Это когда два мужика друг друга в жопу ебут.
875 8963
>>8942

>В плане математики ты глупый человек?


Математика понятия растяжимое, смотря какая математика, есть области в которых я глупый и незнающий, а есть в которых что то знаю. Зависит от качества пояснений и от сложности самого предмета.

>>8944

>То есть эти две строчки сложнее более чем в десять тысяч раз?


Да, так как взгляд на картинку сразу воспринимаю информацию, а эти строчки еще нужно прочитать, обдумать представить нарисовать на бумаге.
876 8964
>>8963
В каких областях математики, выходящих за пределы школьного курса, ты что-то знаешь?
143124.png11 Кб, 619x86
877 8965
Помогите понять, как брать такие большие производные. Это как-то связано с рядом Тейлора.
878 8966
>>8963
Такой существенный разрыв в более чем десять тысяч раз может свидетельствовать о проявлении некоторой умственной болезни?
879 8967
>>8965
Ну вот и подставь x^812+x^602 в ряд тейлора синуса.
880 8968
>>8967
Пиздец, как же я туп. Спасибо.
881 8969
>>8965
тебе надо производную в нуле найти, лол?
882 8970
>>8969
Ага, а че? Там же просто если эту 2016-производную в ручную считать без ряда Тейлора получится какая-то комбинаторная мишура. Но 812+602+602=2016 и в разложении синуса в ряд тейлора почти все зануляется.
883 8971
>>8964
Основы множеств.
Немного знаю, отношения между множествами и операции множеств.
О декартовом произведения имею смутное представления.
Далее идет уже неразбериха.
884 8972
>>8971
Как же я проорал!
Это всё?
885 8973
>>8972
да
886 8974
>>8966
Не может, так как разная подача информации.
887 8975
>>8973
Ты ведь понимаешь, что по сути, ты почти ничего из теории множеств и не знаешь. Даже разбиения на классы эквивалентности не написал.
888 8977
Как вы помните все термины, как минимум?
889 8978
>>8975
Яж говорю учебники идут зашифрованные, там хер поймешь о чем они дальше, то что мог расшифровать я уже сказал.
890 8979
>>8977
Нужно перечитывать учебники, хотя бы бегло, через долгое время, примерно несколько недель. Прочитал, повтори через неделю две, а потом ещё раз по такой же схеме. Можешь ещё в перерывах между чтением решать задачи для закрепления знаний.
891 8980
>>8978
С чего ты взял, что они зашифрованны? Подразумевая такое, получается, что изначально математические знания были в простой форме, понятной семикласснику, а потом злые авторы учебников зашифровали их, чтобы никто кроме них ничего не понял!
892 8981
>>8980
А ведь здравая мысль
893 8982
>>8971
Объясни мне эти темы максимально заумно по-твоему мнению, чтобы было сложнее в десятки тысяч раз, чем взглянуть на картинку. Вот как в том примере с числом пи, но только максимально заумно. Но при этом хочу напомнить, что в том же Шварце, например, имеются определения всех терминов, в которых формулируются теоремы, они могут казаться тебе непонятными, но они есть. Так что определи максимально непонятно основы множеств, можно начать с определения множества и так далее.
894 8983
>>8980
Им лень понятно писать книги.
895 8984
>>8983
Смотри, если предположить, что изначально математические знания понятны семикласснику, то для шифрования в сложный учебник авторы затратят гораздо больше времени и сил, чем просто печатая, как есть. Поэтому аргумент с ленью отпадает.
896 8985
>>8984
Эти знания по умолчанию зашифрованы, это как скульптор высекает из камня скульптуру, только авторы учебников ленятся высекать суть из общего.
897 8987
>>8984
Ну на самом деле тут есть что-то логичное, вот скажем я, изучив подробное доказательство какой-нибудь теоремы, пересказываю ее в свой конспект максимально кратко, т.к. писать лень. Получается мало понятно посторонним людям.
898 8988
>>8985
Предположим, что это так. Тогда, если бы они были однажды расшифрованны, то не надо было бы каждый раз пасать учебник с зашифрованными знаниями, был бы учебник с расшифрованными, т.к. это принесло бы больше профита. И учебники с зашифрованными знаниями перестали бы существовать.
Опять противоречие с твоими словами.
899 8989
>>8988
На расшифровывания более сложных знаний нужно затратить больше усилий, чтобы простые люди поняли, нужно иметь способность к простому объяснению материала и не лениться. Можно подать материал в котором человек ничего не поймет, а можно подать так что все станет понятно с первого раза.
Например, английски учебник по математик и алгебре, там расшифровка знаний 10 из 10, учебник зорича это дно дна, 0 из 10.
900 8991
>>8989
Всё сходится. Тебе нравится детский учебник и не нравится взрослый. Следовательно ты ребёнок. Это норма.
901 8992
>>8991
Причем тут детский?
Мне нравится учебник где понятно объясняется материал.
902 8993
>>8992
Тебе нравятся учебники, где понятно для твоего уровня объясняется материал. Если тебе нравятся детские учебники, значит ты ребёнок.
903 8994
>>8989
Простые люди и от логарифмов ноют, им это все не нужно. Им бы лишь сдачу в магазине считать.
904 8995
>>8989

>На расшифровывания более сложных знаний нужно затратить больше усилий,


Если знание в учебнике, то значит его кто-то понял и расшифровал, тогда зачем ему писать зашифрованное знание?
Значит, ты неправ и знания никто не шифрует.

>чтобы простые люди поняли, нужно иметь способность к простому объяснению материала и не лениться.


В математику нет царских дорог. Простые люди ленивые, как ты, и вместо чтобы сидеть и разбирать смысл понятий, ты ленишся и ищешь учебник, где всё разжеванно, как для детей. Ты ленивый.

>Можно подать материал в котором человек ничего не поймет, а можно подать так что все станет понятно с первого раза.


Если ты не можешь понять с первого раза вводную часть со множествами, то у тебя не университетский уровень, а школьный, тебе надо пройти школьный материал, чтобы встать на ступень выше. Ну или ты ленивый ребёнок.
>>8991
Гомалогчую.
905 8997
>>8993
Что делать дабы нравились взрослые?
906 8998
>>8997
Записаться в маткружок 15 лет назад, например.
907 8999
>>8997
Много читать, думать над прочитанным и решать упражнения. Если что-то непонятно можно погуглить, открыть другой учебник или спросить на каком-нибудь форуме смысл какого-то определения или доказательства.

Только трудом можно достигнуть просветления.
908 9000
А перекаты прикрепленного треда после 1500 или 1000 в этом разделе?
# OP 909 9003
>>9000
После 1000
910 9006
Наша математичка ебанулась и почти сошла с ума, что делать?
911 9007
>>9006
Помочь ей завершить процесс. Очевидно же.
912 9011
>>8820
Ну лол, то есть мне каждый раз уточнять как оно называется, с чем его едят и подобные вопросы? Я же вроде не роботами разговариваю.
913 9017
Школьник 18 лвл
В будущем хочу заняться бизнесом и наукой
Бизнес планирую развивать консалтинговый в области финансов и IT
Научную деятельность хотелось бы связать с компьютерными науками/математикой и их непосредственным применениям в бизнесе.
Встал вопрос с выбором специальности
"прикладная математика" или "финансовая математика"
Что можете посоветовать неопределившейся школоте?
1484982418170-1026835315.jpg297 Кб, 2560x1440
914 9020
>>6879 (OP)
спаси меня, двач
915 9021
>>9020
Врачём будешь, инфа 70%
916 9022
>>9020
Континум-гипотиза верна?
917 9023
>>9020
aleph_0 x aleph_k = aleph_k верно для любого кардинала. В том числе и для кардинала <2^aleph0. Что полностью решает.
918 9024
>>9022
нет
919 9025
>>9023

>aleph_0 x aleph_k = aleph_k


а как можно это доказать ?
920 9026
При разбиении континуального множества на конечное или счётное число частей хотя бы одна из частей будет иметь мощность континуум.
Докажите это, молю.
921 9028
>>9025
Это общий факт. Леммой цорна 4 раза.
922 9030
>>9026
Не более чем счётное объединение не более чем счётных множеств не более, чем счётно. Следовательно, если в твоём разбиении не будет множества мощности континуум, его будет можно пересчитать.
923 9031
>>9030

> не более чем счётных множеств


Там не более, чем континуальных.
924 9032
>>9030
Т.е менее, чем континуальных.
>>9031
925 9033
Какой у вас IQ?
926 9035
>>9017
идти в ВШЭ на любую специальность.
927 9036
>>9032
Same shit. Если, конечно, между счётным и несчётным множеством других нет.
928 9037
929 9043
>>9037
Ну вот и я о том же.
930 9044
Как решить задачку?
Есть шестиугольник со стороной "а", с центром диагоналей находящейся в начале координат, при чем одна из диагоналей находится на оси иксов. Найти координаты вершин шестиугольника?
931 9045
>>9044
Ты вот этот >>8930 ?
932 9046
>>9045
Нет, я не могу решить задачку.
933 9047
>>9046
На каком шаге возникает проблема?
934 9051
>>9044
корни из единицы
935 9052
>>9051
Это понятно, так можно получить координаты вершин шестиугольника с единичной стороной. А что дальше?
936 9053
>>9052
Знаешь, как найти гипотенузу, зная катеты?
937 9054
>>9052
ну и поверни его.
938 9055
Лучшие умы человечества собрались на научную конференцию. Обсуждается вопрос: "сколько будет дважды два".
Инженер колдует с рулеткой и логарифмической линейкой, после чего уверенно объявляет результат: "3,99".
Программист обратился в службу технической поддержки, поставил численный эксперимент на компьютере и доложил: "между 3,98 и 4,02".
Математик посмотрел в потолок, подумал и сказал, что точного ответа он не знает, но зато может доказать, что этот ответ существует.
Логик попросил более точно определить, что такое "дважды два".
Философ полчаса рассуждал о том, что "дважды два" можно понимать совершенно по-разному.
Хакер предложил взломать защиту секретной сети Пентагона и заставить все компьютеры решать эту проблему.
Наконец, бухгалтер сказал: "Закройте все двери и окна, а теперь ответьте - а сколько вы хотите получить?"
939 9056
Поймал Мефистофель философа, математика, физика, и сказал: прыгайте с десятиметровой вышки в бассейн диаметром 1 метр.
Философ порассуждал, примерился, помедитировал, потом махнул рукой, авось повезет и прыгнул. Не повезло.
Физик поднял палец, померил скорость ветра, просчитал несколько вариантов, прыгнул и попал точно в середину бассейна.
Математик построил модель, написал программу, вычислил траекторию полета, построил график разбега. Разбежался, прыгнул и... Стрелой унесся вверх!!! Ошибка в вычислениях, противоположный знак результата!
940 9057
Летят двое на воздушном шаре... Унесло их, и не знают, где они сейчас... Пролетают мимо холма, на котором сидит человек. Храбрые воздухоплаватели спрашивают его:
- Скажите, пожалуйста, где мы сейчас находимся?
Человек на холме долго думает, после чего отвечает:
- На воздушном шаре.
Более пожилой и, следовательно более умудренный опытом воздухоплаватель говорит другому:
- Этот человек на холме - математик.
- Почему же?
- Он долго раздумывал над простым вопросом, после чего дал абсолютно точный и совершенно бесполезный ответ...
941 9058
Не смешно.
942 9059
>>9055
>>9056
>>9057
Полный хохотач.
943 9062
Встретились как-то два математика в баре. Первый говорит: "А не хочешь ли ты ПОДЕЙСТВОВАТЬ СВОИМ ФУНКТОРОМ на мой ТЕРМИНАЛЬНЫЙ ОБЪЕКТ?", а второй ему: "Я бы с радостью, но у меня ПУЧОК ВЯЛЫЙ".
Этим математиком был Альберт Галуа-Гротендик.
944 9066
>>9062

смешно
945 9068
946 9069
>>8932

>Может все эти вопросы от того, что как-то не выработана какая-то научная интуиция, чувство поиска прекрасного и поразительного в явлениях, структурах, системах?



Возможно мой мозг не настолько приспособленный чтобы воспринимать высокий уровень математической абстракции.

>>8982

>Объясни мне эти темы максимально заумно по-твоему мнению



Открой книгу зорича и шварца и увидешь заумное объяснения, без примеров, это все равно что я попытаюсь тебе словами описать как выглядит динозавра, так ты будешь очень долго в голове представлять как он выглядит и то неправильно себе представишь, а взглянувши на картинку ты сразу составишь для себя целостную картину.

>>8993
Охуеная логика, если тебе нравятся игры значит ты ребенок а взрослые дяди в игры не играют они зарабатывают деньги и ебут тян.
screen.png322 Кб, 1362x745
947 9070
>>9069
Это в Зориче-то всё заумно и без примеров? Там к каждому определению по 10 штук примеров с детальным разжёвыванием каждого. Ты ещё литературы без примеров не читал, я погляжу.
948 9071
>>9047
Я не учил в школе шестиугольники и не знаю их свойств.
Предыдущая задачка была на нахождение углов квадрата обе диагонали которого совпадают с осями и проходят через начало координат, и равностороннего треугольника, одна сторона которого на оси иксов с одним углом в начале координат.
Их я решил, для квадрата ответ +/-корень(а^2/2) для треугольника координаты второго угла (а,0) третьего (а/2, корень(а^2-a^2/4))
949 9072
>>9057
Летят шерлок холмс и доктор ватсон на воздушном шаре...Унесло их, и не знают, где они сейчас... Пролетают мимо холма, на котором сидит человек и пасет овец. Храбрые воздухоплаватели спрашивают его:
- Скажите, пожалуйста, где мы сейчас находимся?
Человек на холме долго думает, после чего отвечает:
- На воздушном шаре.
Шерлок холмс говорит ватсону
- Этот человек на холме - математик.
- Почему же?
- Он долго раздумывал над простым вопросом, после чего дал абсолютно точный и совершенно бесполезный ответ.
Еще несколько подумав он говорит
- Я точно уверен что мы в России
- Но почему?
- Где еще вы сможете увидеть математика пасущего овец?


Вот теперь правильно.
950 9076
>>9069

>Охуеная логика, если тебе нравятся игры значит ты ребенок а взрослые дяди в игры не играют они зарабатывают деньги и ебут тян.


Совсем не так. Если тебя нравятся детские игры, то ты ментальный ребенок. Вот скажем игры про Дашу путешественницу могут вызвать искреннее удовольствие только у человека, который прибывает в состоянии детского мышления или является ребенком. Так же и с учебниками. Если ребенку нравится изучать дифференциальные формы на гладких многообразиях, то он ментальный взрослый.

>Открой книгу зорича и шварца и увидешь заумное объяснения


Я там не увижу твоих заумных объяснений. Поставь себя на место авторов учебников, с чувством собственной важности выплескивающих заумные определения. Начни с множества, объединения и пересечения, например. Представь, что тебе лень разжевывать и выдай все максимально зашифровано.
951 9078
>>9071
ну я тебе подсказал гарантированное: через корни из единицы и поворот плоскости. А так вполне вероятно, что надо просто какие-то треугольники другие посчитать.
952 9079
>>9072
Веселее будет "скорее всего мы в Дагестане".
953 9080
>>9078
А зачем поворачивать, если вершина лежит на абсциссе? И как от единицы перейти к произвольной стороне?
954 9081
>>9080
Я не читал далее "шестиугольник", может и не надо никуда поворачивать.

>как от единицы перейти к произвольной стороне


Что будет если корень из единицы умножить на a?
955 9085
https://interneturok.ru/algebra/7-klass/matematicheskij-yazyk-matematicheskaya-model/matematicheskaya-model-i-tekstovye-zadachi?seconds=0

Посмотрел видео этой тематики [Математическая модель и текстовые задачи] и просто прозрел. Что еще можно посмотреть и почитать [кроме этого сайта естественно] дабы так же прозреть? Но учтите, что я сейчас нахожусь в 7 классе по своим знаниям[, а может и ниже, так как сейчас пытаюсь вспомнить пропорции].
956 9086
>>9085
Как ты с таким уровнем знаний сайты делаешь? Ты как работу нашел?
957 9087
>>9085
Анон, это ты выше про шифрованного Зорича бугуртил?
958 9088
>>9086
Это другой анон, почини свою логику, я раньше скидывал эту ссылку.
Для делания сайтов математика ненужна, неук ты.
959 9089
>>9087
Я другой.
>>9086
Я не делаю сайты. И не работаю. Думаю получить справку по психической инвалидности и получать выплату по оной.
960 9090
>>9085
Еще есть это https://stepik.org/explore/courses но там похеровей, мало объяснений и сложные задания.

Еще вот это недавно нашел https://allbesta.net/torrent/15022-matematika-7-11-klass.html
961 9091
>>9088
Как-то подозрительно куда-то пропал две-железяки-дебил и появился картинко-дебил делающий сайты с iq 97 и уровнем знаний семиклассника. Это один и тот же человек.
962 9092
>>9089
Можешь прочитать учебники Алимова 7,8,9, 10-11 класс, там даже множества есть с картинками.
963 9093
>>9090
Спасибо. Попробую взять и это, когда дойду до 9 класса понимания.
>>9092
Иллюстраций не так уж и много, но спасибо. Буду решать задачи.
964 9094
>>9088
Для создания ии математика тоже не нужна, но ты почему-то тут упорно тусуешься.
965 9095
>>9094
В ии применяется теория графов, а это из математики.
math.jpg386 Кб, 1375x923
966 9096
>>9070

>Это в Зориче-то всё заумно и без примеров?


да
967 9097
>>9095
И поэтому ты втыкаешь в учебники по математическому анализу?
968 9098
>>9096
Очень толсто. Ты обычную школу закончил? Тебя оставляли на второй год?
969 9099
>>9096
И ты после этого говоришь, что ты не тупой. Боже.
970 9100
>>9095
А просто программировать ты уже научился? Алгоритмы какие-нибудь знаешь?
971 9101
>>9096
Во-первых - это не учебник по теории множеств и математической грамотности, когда ты учишь анализ то предполагается, с исчислением кванторов первого порядка и с базовой теорией множеств. Зорич лишь сделал главу для беглого повторения твоих знаний.
Во-вторых половина ответов на твои вопросы на этой же странице и написана, другая половина была написана раннее. Не очень понятно как ты читал. Я по Зоричу на первом курсе в математику вкатывался, мне было (после некоторых усилий) всё понятно.
В-третьих с таким подходом ты точно вообще ничего не изучишь. Не только в математике. Когда что-то непонятно - то нужно пытаться выяснить - искать доп. материалы, спрашивать, пытаться самому строить примеры; а не сидеть ровно и жаловаться на то, что учебники не заточены специально под тебя.
Ты, к слову, вообще хоть представляешь что значит написать учебник? Понимаешь, что учебник по анализу в себе должен содержать довольно большое количество информации и при этом не должен быть на 3к страниц? Знаешь, что иногда нужно выбирать, на чём нужно акцентировать внимание, а на чём нет (не только из-за объёма, но ещё из-за того, что если акцентировать внимание на всём, читатель просто устанет и не составит для себя никакой big picture)?
972 9102
>>9098
Нет не оставляли, да закончил. По математике всегда была оценка 2, от 6 класса и выше.

>>9100
Синтаксис хорошо знаю и умею программировать, но алгоритмы не учил.
Я хотел узнать по какой логике работают графы чтобы не из книг брать готовый программный код, а самому написать, я предположил что лучше изучить теорию графов, чтобы в этом теме без проблем ориентироваться и обрести полное понимания.
973 9103
>>9102
А почему тебя не оставляли на второй год с двойкой или не перевели в спецшколу?
974 9104
>>9102

>По математике всегда была оценка 2, от 6 класса и выше.


>Синтаксис хорошо знаю и умею программировать, но алгоритмы не учил.


> но алгоритмы не учил.


>умею программировать


О, быдломакака, а вот и ты.
975 9105
>>9103
Не везде и не всегда так строго, я живу на Украине и смутно помню только один случай где оставляли на второй год. Хотя были люди которые учились похуже меня, такое предписания нигде не выполняется про которое ты говоришь, и я мог спокойно забивать болт на уроки и уходит не дождавшись окончания всех уроков.
976 9106
>>9102
Никаких предпосылок к самостоятельной разработке ии и даже понимания готовых программных кодов я пока у тебя не увидел. Почему бы тебе не копипастить все решения и алгоритмы из хабра и обучающих видео на ютюбе? Ориентироваться ни в чем математическом ты к 25 годам не в состоянии, даже выдавить из себя пару строчек о теории множеств не можешь. Ты же собираешься заниматься геймдевом в свободное от работы время? Или ты мечтаешь о работе в рокстар? Или индиигры для телефончиков?
977 9107
>>9105
А после школы чем занимался? 9 классов, пту, офис? Так примерно выглядела твоя жизнь?
978 9108
Перекат точно после 1к будет? Срачи все из нового треда пидорнут в другой, да? Я буду репортить вас нахуй развк ли тут /пр и /сок бля
979 9109
>>9108
Что ты тут надеешься увидеть кроме споров и невежества?
980 9110
>>9106

>Почему бы тебе не копипастить все решения и алгоритмы из хабра и обучающих видео на ютюбе?


Могу и скопипастить, но как я добьюсь гибкости в разработки без понимания сути происходящего в этих алгоритмов, и я подумал что дискретная математика мне поможет понять.

>Ты же собираешься заниматься геймдевом в свободное от работы время?


Если добюсь понимания этих алгоритмов то может и перейду в геймдев.

>Или ты мечтаешь о работе в рокстар? Или индиигры для телефончиков?


Я не особо фанатичен к деньгам чтобы так сильно напрягаться, чтобы устроится в крупную компанию, мне нужно чтобы хватало на жилье и на хавку, меня интересует удовольствие от процесса разработки, возможно геймдев это то что мне нужно.

>>9107

>А после школы чем занимался? 9 классов, пту, офис? Так примерно выглядела твоя жизнь?


Было пту, но там еще больше напрягали по учебе чем в школе. После учился на программиста, а после немного работал на дому.
981 9111
>>9110
То есть тебе отчисли из пту, потом ты пошел на курсы программиста и теперь верстаешь в конструкторе сайтики?
982 9112
>>9110

>меня интересует удовольствие от процесса разработки, возможно геймдев это то что мне нужно.


Ты думаешь это такое весёлое дело от которого ты будешь получать удовольствие? Я вот занимался подобным и скажу, что это довольно нудно, особенно вылавливание багов. Если у тебя не хватает сообразительности для того, чтобы осилить определения из Зорича, то как ты будешь выискивать где ошибка? Ты же будешь так сильно тупить и проклинать создателей движка, библиотек и т.д. из-за того, что они не объяснили с примерами, что и как делается. Ты ленивый и не хочешь разбираться с определениями, то можешь даже и не мечтать о создании ИИ, сам же потом забросишь, не трать время.
983 9113
>>9109
Ну что-нибудь про матешу тип))
984 9114
>>9113
Я серьезно, организовал бы лучше семинар в своем университете, пересказывали бы там друг другу любимые теоремы. Тут мало чему можно научится.
985 9115
>>9111

>То есть тебе отчисли из пту, потом ты пошел на курсы программиста и теперь верстаешь в конструкторе сайтики?



Неоткуда меня не вычисляли, да и сайты я особо не делал, делал парсеры на С# под заказ, а жил на свою инвалидность проблемы со спиною были, хорошо знаю С#, сейчас учу js и node.js, чтобы вкатится в бекенд.

>>9112

>Ты думаешь это такое весёлое дело от которого ты будешь получать удовольствие?


Может будет интересно не знаю.

>Если у тебя не хватает сообразительности для того, чтобы осилить определения из Зорича, то как ты будешь выискивать где ошибка?


Человек существо ситуативное, некоторые области я плохо знаю, некоторые знаю хорошо.

>то можешь даже и не мечтать о создании ИИ


То есть написать ии для ботов, чтобы боты знали как обходить игрока и атаковать, искали оптимальные укрытия и не лезли на рожон. Ты считаешь что это неподъемная задача?
986 9116
>>9115

>чтобы боты знали как обходить игрока и атаковать, искали оптимальные укрытия и не лезли на рожон. Ты считаешь что это неподъемная задача?



Это элементарная задача и для этого не нужно даже знать графы и множества, особенно читать все эти ублюдские учебники для задротов. Скачай юнити, братан. Там куча видеоуроков, подробная документация с примерами. Код можно писать на нескольких языках, в том числе и на си шарпе.
987 9117
>>9116
Спасибо за совет, попробую unity.
988 9118
>>9078
Откуда ты взял корень из единицы?
Там же если считать треугольниками получится что угол на оси иксов находится на расстоянии а и -а соответственно.
989 9119
>>9101

> не должен быть на 3к страниц


Зачем?
990 9120
>>9118
два корня шестой степени ты нашел, осталось еще 4.
991 9122
>>9119
Нихуя не понял. Ты из тех, кто слова "зачем" и "почему" путает или это мем какой-то?
992 9123
По геометрии и подразделам, планиметрии, стереометрии, книжек насоветуйте.
993 9124
>>9118

>считать треугольниками


>угол находится на расстоянии


иди на хуй
994 9125
>>9120
Если считать треугольниками, то получается что ответ из предыдущего задания, про треугольник с вершиной в начале координат и стороной на оси, нужно просто взять четыре раза с разными знаками.
995 9126
>>9124
На ровном месте порвался.
С чего вдруг?
996 9128
>>9095
Смотри как все должно быть.
Стоит бот. Тут из за угла ГЕЙмер.
Бот должен сперва определить друг перед ним или враг.

Если друг, стоять дальше.

Если враг узнать какой враг.
Узнать какое у него шмотье, увидел ли он бота или нет, как он себя ведет, куда смотрит.
Потом решить стоит атаковать сейчас или отступить, если атаковать то как и откуда, если отступить то куда и как.

А это все решается по опыту. Как бы ты йобко не сделал ии, опыт ты ему не запрограмируешь.
997 9130
>>9099

>начинайко тред


>не тупой


Сам то с рождения коши был?
998 9132
>>9115
Он тупой даже по меркам треда для начинающих. Сколько ещё человек гнали на авторов учебников, называя их тупыми дегенератами, которые сами ничего не понимают из того, что пишут?
999 9133
1000 9134
>>9125
далее находим первообразные корни: (1+-sqrt(3)i)/2, выделяем мнимую и реальную часть, ответ готов.
1001 9135
>>9126
Этих терминов не существует в природе, что значит считать треугольниками? Что такое расстояние до угла?
1002 9136
>>9125
и иди посчитай своими треугольниками, например, 17-угольник, сторона которого лежит на одной оси, а одним из углов он касается другой оси.
1003 9138
Оп, ты не умер?
1004 9141
>>9033
Зашкаливающий. Серьёзно. Максимальный балл во всех тестах.
1005 9143
>>9141
Докажи. Скрин из теста покажи.
1006 9144
>>9143
Смысл? Скрин можно нарисовать в фотошопе.
1007 9156
В тред врывается лауреат премии долбоеб-2016.

Объясните по-простому, пожалуйста, почему булеан является множеством всем подмножеств. И какой смысл заложен в его записи типа 2^X. А если X - множество отображений плоскости в себя, то как 2 в его степень возводить?
1008 9161
>>9156
Слово "булеан" значит "множество всех подмножеств" по определению.

Символом YX обозначается множество всех существующих функций из X в Y. В арифметике кардиналов это действительно соответствует возведению в степень, но это неважно для понимания дальнейшего.

Символом 2 обозначается множество {0,1}, где 0 и 1 - два каких-нибудь неравных объекта. Символом 2X обозначается множество всех существующих функций из X в 2.

Функция из множества X во множество 2 называется характеристической. Между подмножествами X и характеристическими функциями на X есть биекция. Она такова.

Пусть f - какая-нибудь характеристическая функция. Определим множество M как множество всех таких x из X, что f(x) = 1. То есть если f(x) = 1, то мы считаем, что x входит в M. Если f(x)=0, то мы считаем, что x не входит в M. Таким образом, каждой характеристической функции сопоставлено подмножество X. Причем инъективно.

Обратно, пусть M - какое-нибудь подмножество в X. Определим характеристическую функцию f, положив f(x)=1, если x входит в M, и f(x)=0 в противном случае. Таким образом, каждому подмножеству X сопоставлена характеристическая функция. Причем инъективно.

В силу теоремы Кантора-Бернштейна, между множеством всех подмножеств X и множеством всех функций из X в 2 есть биекция. Поэтому-то булеан и обозначается 2X.
1009 9162
>>9134
При чем тут комплексные числа, если задание дано в декартовых координатах?

>>9135
Считать треугольниками значит упростить шестиугольник до треугольников.
Расстояния от начала координат до угла.

>>9136
Давай мне ты почитаешь координаты углов семнадцатиугольника, покажи что ты не какой то лох-начинающий.
1010 9164
>>9161
Спасибо за ответ. Суть понял. Но если не трудно, разъясни, пожалуйста, пару моментов.

>Символом 2 обозначается множество {0,1}, где 0 и 1 - два каких-нибудь неравных объекта.


В данном случае что это за объекты? Принадлежность x к M, и не принадлежность?

>Между подмножествами X и характеристическими функциями на X есть биекция.


Биекция - это же 1-1 соответствие, нет? Характеристических функций ведь много, а подмножеств - 2: M и его дополнение в X.
1011 9165
>>9164
В данном случае это ординалы. 0 - пустое множество, 1 - множество {0}.

Каждой характеристической функции f соответствует одно конкретное подмножество X, а именно {x∈X | f(x)=1}.
Это соответствие биективно.
1012 9168
>>9165
Спасибо.
1013 9170
>>8977
spaced repetition, например
1014 9172
>>9085
Неожиданно неплохой сайт
1015 9173
>>9033
Минимальный
1016 9175
Существуют ли дешифрованные учебники по топологии?
1017 9176
>>9175
Это какие? Типа топология для самых маленьких?
1018 9177
>>9176
Ну чтоб автор был не тупой.
1019 9178
>>9177
Возможно стоит попробовать каналы на ютубе.
1020 9180
>>9177
Автор не тупой, он просто преподавать не умеет.
Попробуй сам рассказать кому то как на велосипеде кататься.
1021 9181
>>9180
А на каком велосипеде? Цирковой с квадратными колесами? Эт я умею.
1022 9182
>>9180
Умение ездить на велосипеде - неявное знание по Полани. Рассказать об этом в принципе невозможно.

Мимо-проходил.
1023 9183
>>9182
Но научить можно.
1024 9184
>>9183
Нет. В этом и суть. Неявное знание можно получить только самостоятельно. Окружающие могут только дать совет и надеяться, что он поможет.
1025 9185
>>9182
Почему невозможно?
1026 9186
>>9180
Если он умеет держать равновесие, то легко
1027 9187
>>9185
Потому, что неявное знание неформализуемо и может быть получено только при непосредственном контакте с ним. Так, например, словами нельзя нельзя рассказать как правильно плавать, кататься на велосипеде, невозможно объяснить что такое красный цвет, если человек его никогда не видел. Почитай статьи Майкла Полани. Так-то это одна из основополагающих статей современной теории познания.
1028 9188
>>9187
основополагающих концепций имел ввиду.

самофикс
1029 9189
>>9187

>невозможно объяснить что такое красный цвет, если человек его никогда не видел


возможно
1030 9190
>>9189
Ты шизофреник?
1031 9191
>>9187
Но ведь посмотрев на картинку можно понять как плавать, кататься на велосипеде. Или там видео + картинки + схемы, еще что. Ты можешь стать великим теоретиком по этим делам даже не пытаясь. Или вот, люди могут вызывать эйдетические образы и запускать механизмы прямо перед собой. Еще почему-то мне хочеться приплести к этому синестезию, но это, наверное, не совсем верно.
1032 9192
>>9190
Невозможно объяснить как он выглядит, но возможно его физическую природу.
1033 9193
>>9190
Глухие люди учаться распознавать красный цвет по звуку, например. Тебе могут сказать, что красный цвет обозначает опасность, кровь и так далее. Ты можешь построить с этим ассоциацию. И прибор будет видеть красный цвет, а передавать тебе звуком.

мимо
1034 9195
>>9191
Это все не не верно. Зная что нужно делать, кататься на велосипеде ты не научишься. Теория может лишь помочь двигаться в верном направлении, но пока не сформируется мышечная память ничего не выйдет.
1035 9196
>>9192
Физическая природа - абстракция, неинформативная сама по себе. Никто глазами волновую функцию не видел, поэтому без представления, опосредованного органами чувств, понятия о красном цвете не будет.

>>9193
Как глухие люди могут учиться по звуку? Не понял.
1036 9197
>>9196
Думаю "слепые", а пишу "глухие". Извиняй.
1037 9199
>>9197
Я так и подумал, но для порядку надо было уточнить.

О таком я не слышал, но судя по описанию это - простая замена визуального образа слуховым. Понятия о красном цвете таким образом получить невозможно, слепой человек может только узнать присутствует этот цвет в окружении или нет.
1038 9200
>>9199
А теперь докажи, что тот гештальт, который формируется у слепого и ассоциируется со звуком, отличается от того, что мы называем "красным цветом".
1039 9201
>>9200
Не ударяйся в платонизм. Рациональное мышление построено на материалистических началах. В рамках теории научного познания подобные рассуждения неправомочны. Человек рождается без представлений об окружающем мире, соответственно без представлений о красном цвете, и сформироваться эти представления могут только напрямую под влиянием этого цвета и никак иначе. Дальнейшие мудрствования уводят в область идеализма и солипсизма.
1040 9202
>>9162

>Расстояния от начала координат до угла


до вершины
1041 9207
Что такое нетривиальные нули дзета функции? Может кто объяснить простым языком?
1042 9208
Почему мне математика теперь кажеться реальным миром? А ведь я только начал. Я словно обмазываюсь всеми этими аксиомами, плаваю в мире ощущая полет фантазии и состояние эйфории. Все ощущения удивительным образом переплелись, я могу слышать увиденное, чувствую запах звука, математика становиться как переплетения цвета у импрессионистов. Это может привести к плохим последствиям? Связанно ли это с тем стрессом, в которым у меня прекратился внутренний диалог? Все плавает вокруг.
1043 9210
Переведите на русский, плз.

Рассмотрим множество A
всех множеств X таких, что X не является элементом X. Будет ли A
принадлежать A? Если A не принадлежит A, то A должно быть своим
элементом. То есть формальным следствием A /∈ A является A ∈ A.
1044 9211
>>9207
Ноль, который не является чётным целым отрицательным числом.
>>9210
Брадобрей бреет всех, кто не бреет самого себя. Должен ли брадобрей брить сам себя?
1045 9212
>>9211
Должен, так как в любом случае он или бреется сам, или не бреется сам и бреется как брадобрей.
1046 9214
>>9212
Бреет себя -> не должен брить себя (так как брадобрей бреет всех, кто не бреет себя)
Не бреет себя -> должен брить себя (так как браадобрей бреет всех, кто не бреет себя)

Твоё рассуждения не понял. И не очень хочу.
1047 9216
>>9214
Брадобрей бреет себя либо как тот кто бреет себя сам, либо не бреется сам и бреет себя как брадобрей, который бреет тех кто не бреет себя сам, что тут не понятного?
1048 9217
>>9211
Как ноль может быть четный или нечетным, это же просто ноль.
1049 9218
>>9217
Ноль функции f, это такая точка x из области определения f, что f(x) = 0.
1050 9219
>>9208

>Я словно обмазываюсь всеми этими аксиомами, плаваю в мире ощущая полет фантазии и состояние эйфории.


По этому поводу я бы не беспокоился - конечно из твоего описания не до конца ясно, что ты имеешь ввиду, но в целом люди которые любят математику часто испытывают положительные эмоции по её поводу.

>Все ощущения удивительным образом переплелись, я могу слышать увиденное, чувствую запах звука, математика становиться как переплетения цвета у импрессионистов


Я вовсе не психолог, но насколько я могу судить это ненормально. В норме математика воспринимается в визуальных и вербальных терминах. Хотя, насколько я помню из чтения чего-то популярного по психологии, незначительная ассоциация одного вида сенсорных ощущений с другим находится в пределах нормы, но когда этот эффект становится существенным это уже ненормально. Впрочем, ничего не могу сказать, касательно того, может ли это являться симптомом более существенной проблемы или нет.
1051 9220
>>9216
Тебе же уже сказали идти ебаться со своим юнити, тебе нечего тут делать с iq 97
1052 9221
>>9175
Попробуй из них
1) Васильев В. А. - Топология для младекурсников
2) Сидней А. Моррис - Топология без слёз
3) В. Г. Болтянский, В. А. Ефремович - Наглядная топология
4) В. В. Прасолов - Наглядная топология
5) Стинрод Н., Чинн У. - Первые понятия топологии
1053 9223
>>9216
Брадобрей либо бреет себя, либо не бреет себя. Никаких модальных связок вида "бреет себя как тот, кто себя сам", "бреет себя как брадобрей", "бреет как Николай II" в этой задаче не может быть. Когда ты их вводишь - ты решаешь другую задачу.
1055 9225
>>9224
Зачем ты мне это скидываешь?
1056 9226
>>9220
Не угадал мудак, это другой анон, я тот.
Пиздеть насчет iq каждый может, пройди тест сняв этот процесс на видео, и сбрось сюда результат, хуеплет.
1057 9227
>>9211

>Брадобрей бреет всех, кто не бреет самого себя. Должен ли брадобрей брить сам себя?


Да-да, видел такую формулировку этого парадокса. Мне не понятна связь между ними. В терминах множеств для меня это не имеет смысла. Начиная хотя бы с того, как могут быть "все" множества X. X - либо одно множество, либо множеств много и тогда они X1, ..., Xn, а не просто X. И так далее. Вообще для меня такая формулировка - просто набор не связанных семантически слов.
1058 9229
>>9201

> могут только напрямую под влиянием


Редукционист, ты психическую деятельность мозга к восприятию длин волн сетчаткой глаза не приравнивай, угумс? Начнем с того, что "красный цвет" - понятие настолько абстрактное и размытое, что нет, наверное, даже двух человек, у которых оно сформировалось бы под действием одних и тех же раздражителей. Я даже не заикаюсь про лингвистическую сторону вопроса, я о более простых вещах говорю.
1059 9230
>>9208

> математика становиться


Не мог не обосраться, напыщенный школьничек.
1060 9233
>>9229

>Редукционист


Лол, значение знаешь?

>нет, наверное, даже двух человек, у которых оно сформировалось бы под действием одних и тех же раздражителей


Почему ты приравниваешь единичную сущность к классу сущностей? Красный цвет в человеческой культуре воспринимается исключительно как класс раздражителей во всем многообразии их подтипов. Ты бы хоть учебник по эпистемологии открыл прежде чем позориться.
1061 9234
>>9233
А можно ссылку?
1062 9236
>>9226
Ну у меня в том тесте был результат 140, и это по-моему максимальный результат этого теста. Давай я запарюсь и сниму видео с прохождением для тебя, если ты воспроизведешь действия заумных авторов и распишешь максимально непонятно все, что знаешь про множества, уже который пост тебя прошу. Ты же говорил, что это легче, чем объяснять с примерами и картинками, чего же ты этого не сделаешь?
1063 9237
>>9201

>рациональное мышление построено на материалистических началах


Рационалист выискался, поглядите. До Полани добрался, но до Фейерабенда ещё нет?
Назвать какую-то процедуру или точку зрения объективной (объективно истинной) — значит утверждать, что она верна независимо от человеческих ожиданий, идей, позиций, желаний. Однако сама идея объективности гораздо старше, чем наука, и не связана с ней. Она появляется тогда, когда какой-то народ, племя или цивилизация отождествляет свой образ жизни с законами мироздания (физическими и нравственными).
Второй идеей, играющей важную роль в защите западной цивилизации, является идея Разума (с большой буквы),
или рациональности. Как и понятие объективности, эта идея имеет материальный и формальный варианты. Быть рациональным в материальном смысле — значит избегать одних идей и признавать другие идеи.
1064 9238
Как доказать, что при заданном угле и противолежащей ему стороне сумма двух других сторон наибольшая тогда, когда эти две стороны равны (треугольник равнобедренный)?
1065 9239
>>9233

> значение знаешь


То же самое хотел спросить по поводу не к месту вспомненного тобой платонизма.

> Почему ты приравниваешь


Тебе показалось. А еще ты упустил контекст и совершенно не уловил мой поинт. Я даже не знаю, что тут можно сделать; если тебе хочется специальной олимпиады вместо обсуждения - увольте, давай как-нибудь сам.
1066 9240
Ребята-математики, помогите решить задачу по СПУ, у меня уже нет сил, завтра экзамен и мне ещё кучу всего надо сделать, это единственное, что вызывает у меня проблемы.
Условие задачи помечено жёлтым, остальное не надо, данные предоставил и примерное решение аналогичной задачи, очень надеюсь на вашу помощь, если кто-то добрый вдруг возьмётся, прошу, отпишите.
1067 9241
>>9090

> https://allbesta.net/torrent/15022-matematika-7-11-klass.html


Как это скачать? Скачивается экзешник с амигой.
NoName 1068 9242
>>9240

Это ни разу не матан. Но попытаюсь разобраться (как у вас там назывется каждое из чисел в кружках я не знаю буду называть по позиции).
Значит так.
Обозначения: В левой верхней графе кружечка находится его номер в остальных 3 графах другие данные. Как они у вас называются спросить у согрупников.

I) Составление схемы.
1) Определяем номер который не встречается как последний аргумент в таблице - это начало схемы в данном случае - это номер 1.
2) Слева вверху схемы рисуем первый кружокк. Кружок делим на 4 графы. В левой верхней графе пишем номер 1.
3) В правом нижнем углу схемы рисуем кружок с тем номером который не встречается в таблице в качестве первого аргумента. В данном случае это кружок номер 10. Опять в левой углу кружка пишем его номер т.е. 10.
4) Аналогично рисуем кружки с номерами от 2 до 9. Анон расставляй их широко я не знаю правил расстановки, но думаю что их нет. В левой верхней графе каждого кружка пишем номер кружка.
5) Теперь нужно соединить кружки стрелками. Смотрим таблицу и русуем стрелки:
Т(1,2) означает что нам нужно на схеме провести стрелку от кружка 1 к кружку 2 и написать число 13 над стрелкой. Когда все стрелки нарисованы у нас есть схема для анализа.

II) Анализ схемы "вперед"
1) В кружке 1 пишем три нуля в отставшиеся 3 графы.
2) Идем "волной" с первого кружка по направлению стрелок.
Складываем число на стрелке с числом записанным справа вверху в том кружке откуда стрелка выходит. Получаем числа-кандидаты. Если в кружок заходит одна стрелка - сразу пишем число в графу справа вверху в том кружке куда стрелка зашла. Если в кружок зашло более одной стрелки определям максиамльное из чисел кандидатов и пишем туда же.
3) Повторяем так до тех пор пока не заполним графу во всех кружках.
III) Анализ схемы "назад"
1) В кружке 10 переписываем число из правой верхней графы в правую нижнюю
2) Идем волной против направления стрелок от последнего кружка - кружка 10. Вычитаем из числа записанного справа внизу в том кружке куда стрелка заходит число записанное на стрелке. Это число-кандидат. Если из кружка выходит одна стрелка - сразу пишем число справа внизу. Если из кружка вышло более одной стрелки определям минимальное из чисел и пишем туда же.
3) Повторяем так до тех пор пока не заполним графу во всех кружках.
IV) Определние критического пути и прочей хуеты
1) В каждой кружке вычитаем из нижней правой графы верхнюю правую. И пишем результат в нижнюю левую графу. Кружки с нулевыми значениями этой графы образуют цепочку - критический путь. Ненулевые значения указывают запас.
NoName 1068 9242
>>9240

Это ни разу не матан. Но попытаюсь разобраться (как у вас там назывется каждое из чисел в кружках я не знаю буду называть по позиции).
Значит так.
Обозначения: В левой верхней графе кружечка находится его номер в остальных 3 графах другие данные. Как они у вас называются спросить у согрупников.

I) Составление схемы.
1) Определяем номер который не встречается как последний аргумент в таблице - это начало схемы в данном случае - это номер 1.
2) Слева вверху схемы рисуем первый кружокк. Кружок делим на 4 графы. В левой верхней графе пишем номер 1.
3) В правом нижнем углу схемы рисуем кружок с тем номером который не встречается в таблице в качестве первого аргумента. В данном случае это кружок номер 10. Опять в левой углу кружка пишем его номер т.е. 10.
4) Аналогично рисуем кружки с номерами от 2 до 9. Анон расставляй их широко я не знаю правил расстановки, но думаю что их нет. В левой верхней графе каждого кружка пишем номер кружка.
5) Теперь нужно соединить кружки стрелками. Смотрим таблицу и русуем стрелки:
Т(1,2) означает что нам нужно на схеме провести стрелку от кружка 1 к кружку 2 и написать число 13 над стрелкой. Когда все стрелки нарисованы у нас есть схема для анализа.

II) Анализ схемы "вперед"
1) В кружке 1 пишем три нуля в отставшиеся 3 графы.
2) Идем "волной" с первого кружка по направлению стрелок.
Складываем число на стрелке с числом записанным справа вверху в том кружке откуда стрелка выходит. Получаем числа-кандидаты. Если в кружок заходит одна стрелка - сразу пишем число в графу справа вверху в том кружке куда стрелка зашла. Если в кружок зашло более одной стрелки определям максиамльное из чисел кандидатов и пишем туда же.
3) Повторяем так до тех пор пока не заполним графу во всех кружках.
III) Анализ схемы "назад"
1) В кружке 10 переписываем число из правой верхней графы в правую нижнюю
2) Идем волной против направления стрелок от последнего кружка - кружка 10. Вычитаем из числа записанного справа внизу в том кружке куда стрелка заходит число записанное на стрелке. Это число-кандидат. Если из кружка выходит одна стрелка - сразу пишем число справа внизу. Если из кружка вышло более одной стрелки определям минимальное из чисел и пишем туда же.
3) Повторяем так до тех пор пока не заполним графу во всех кружках.
IV) Определние критического пути и прочей хуеты
1) В каждой кружке вычитаем из нижней правой графы верхнюю правую. И пишем результат в нижнюю левую графу. Кружки с нулевыми значениями этой графы образуют цепочку - критический путь. Ненулевые значения указывают запас.
1069 9243
>>9242
Дружище, спасибо тебе большое, к сожалению я уже решил, но ты всё равно хорош!
1070 9244
>>9241
Cкачал, эта хуйня работает вообще на вин10?
1071 9245
>>9244
У меня на 7 ссылки не открывает.
1072 9247
>>9227
Определим формулу P(X) как P(X) ⇔ X ∉ X.
Рассмотрим класс K, определенный как K = {x | P(x)}.
Предположим, что K - множество.
Тогда либо K∈K, либо K∉K.
Предположим, что K∈K. Тогда P(K). Тогда K∉K.
Предположим, что K∉K. Тогда P(K). Тогда K∈K.
В обоих случаях имеем противоречие.
Значит, K не является множеством.
1073 9252
Пока все эксперты-топологи в школе, помогите мне решить еще одну задачку.
Нужно найти обратную функцию к функции y=х^4+x^2.
В предыдущем задании была линейна функция и она легко прошла, делом то, плюс минус и корень из степени сделать.
А тут я не знаю как быть.
1074 9253
ОП, добавь в шапку
https://www.youtube.com/watch?v=ysGdt9ZLiQQ
1075 9254
>>9238
Бамп
1076 9255
>>9252
Начни с нахождения обратной функции для f(x) = x^2
1077 9256
>>9255
Корень игрека.
Получается корень игрека плюс корень четвертой степени игрека?
1078 9257
Как называется способность прикинуть взглядом сколько есть предметов? То есть, прикинул взглядом и уже понимаешь, что шесть, например. Как это называется и как гуглить?
47c2cf8605ee9ddd07411bb1ef89a4edbf0e34.pngsrz39339385220.50[...].png103 Кб, 393x393
1079 9258
>>9257
Нагуглил какую-то "ментальную арифметику" и эйдетизм.

Суть в том, что сначала берут устройство типа пикрелейтед под названием абакус. Что-то похожее на счеты. Учат с ними работать.
Потом как-то постепенно перемещают процесс такого счета с реального предмета в ум.

Но я таки не нашел материалов для самостоятельного развития. Да и говорят, мол это лишь детям до 12 лет.
1080 9259
>>9258
На минуту славы хочешь попасть?
1081 9260
>>9256
Определи строго понятие функции и понятие обратной функции.
1082 9261
>>9258
Ну, вот. Заблокировали в гугле.
1083 9262
>>9259
Какую минуту славы?
103726160largePR20121203140916[1].JPG63 Кб, 640x360
1084 9263
1085 9264
>>9263
Ты хочешь сказать, что я фиолетовая стрелка?
hm.png226 Кб, 407x472
1086 9265
>>6879 (OP)
Математики изобретают или открывают?
1087 9266
>>9265
Доказывают.
1088 9267
>>9263
Нет, просто видел там детей, которые складывали большие числа, перебирая пальцами по воображаемым счётам. Выглядело это глупо.
1089 9268
>>9260
Набор действий чтобы однозначно получить из одних чисел другие.
Обратная когда из других получают одни.
1090 9269
>>9268
Это скорее понятие алгоритма.
1091 9270
>>6879 (OP)
Тривиальный вопрос:
как определяется кольцо целых чисел Z[\sqrt(-1)]?
Ну через эти, всякие категорные конструкции.
Это двумерный модуль над целыми числами - ок.
Но там ещё определено умножение... Вот что-то меня беспокоит... Как определяете?
1092 9271
ну ок, это ещё кольцо полиномов, факторизованное по уравнению (x^2+1=0). Осталось определить написанное.

Кольцо остатков от деления на x^2+1. Всегда ли это кольцо?

Факторкольцо...
Факторизовать можно на идеал, вроде. Значит идеал у нас - это многочлены, делящиеся на x^2+1. Главный идеал, так как умножение многочленов коммутативно. (Я чего-то очень торможу.)
1093 9272
>>9268
>>9268
Функция это такое подмножество декартового произведения двух множеств AxB, что все a из A встречаются один и только один раз в нём.

Т.е. функция это множество пар (x, f(x)), где f(x) значение функции, которое мы выбрали для данного х. И в этом множестве каждое конкретное х встречается только один раз.

Пусть дана функция f и предположим существование обратной функции g для этой f. Тогда должно выполнятся тождество g(f(x)) = x по определению обратной функции.

Очевидный способ получить обратную функцию из множества пар (x, f(x)) это построить множество пар (f(x), x) (взять кажду пару из определения f и поменять местами компоненты пары). Покажи теперь, что для функции f(x) = x^2 определённой на всех (в т.ч. отрицательных) действительных числах это не сработает. В частности, т.к. нарушится условие уникальности первой компоненты в множестве пар (f(x), x).

Если всё ещё непонятно: заметь, что f(-2) = f(2), чем должно быть ровно g(f(-2)) и g(f(2))? -2 и 2? Но ведь f(-2) = f(2), а значит по определению функции g(f(-2)) = g(f(2)), -2 = 2?!??!??!
1094 9273
Есть две окружности s и S радиусом соответственно r и R (при чем r<R).
Требуется найти параметрическое уравнение кривой получающейся при качения точки на окружности s по S в случае
если s внутри S
если s снаружи S
Поясните как тут нужно действовать. Я могу понять что будет какая то кривая где то между R и R+r|R-r, но я не могу понять как взять синус с косинусом чтобы получить то что требуется.
1095 9274
>>9272

>g(f(x)) = x


разве равно не единице?
1096 9275
>>9274
Что? х? Нет

возьми f(x) = 2 * x и обратную ей g(x) = x/2. g(f(100500)) = 100500
1097 9276
>>9274
Если говорить в терминах композиции функций, то g ∘ f = Id, где Id(x) = x. Id в алгебраическом смысле "единица".
1098 9278
Анон, я 26-летний долбоёб без образования и будущего надо бы хотя бы заочку какую-нибудь окончить, а знаний со школы для написания пресловутого егэ уже и нет нихуя.
С чего стоит начать? C 7 класса общеобразовательной?
1099 9279
>>6879 (OP)

>Список от ОП-а, бывшего тут до меня. Был составлен на протяжении 13 тредов, к ознакомлению обязателен.


>http://pastebin.com/4iMjfWAf



>>9278
1100 9280
Чики, анон. Я тут понемногу угораю по алгтопу, но жестко туплю местами. Например, не могу посчитать гомологии CP^2xS^1. Вот по отдельности и у того, и другого найти могу, а как у произведения не знаю. Помоги, пожалуйста.
12957132613621.png85 Кб, 1451x799
1101 9282
Парни, поясните, вот у нас график. Если мало точек, то квадрат превращается в странную хренотень. Так вот.
Есть ли в природе обратные интерполяторые? Как они называются? Называются ли они как-то?
1102 9284
>>9270

>Ну через эти, всякие категорные конструкции.


Честно-говоря не очень понимаю, как с разумными категорными конструкциями может быть ясно, что это двумерный модуль, но не ясно, что это кольцо (скорее вопрос будет о том, как доказать двумерность). Если не вдаваться в излишнюю общность, то я бы определял примерно так. Рассмотрим категорию колец и категорию колец с константой i в которых имеет место тождество i^2=-1. Естественным образом есть забывающий функтор из второй категории в первую P. У него есть левый сопряженный F. Искомое Z[\sqrt(-1)] - это просто P(F(Z)). Притом, кольцом оно будет просто в силу своего определения. Единственное остается вопрос, почему у P есть левый сопряженный, но здесь либо нужно сослаться на общие факты, либо доказать в конкретном случае, например в духе >>9271.
1103 9285
>>9278
Если цель написать егэ, то можешь смотреть видео\читать пособия по подготовке.
1104 9292
>>9285
С тем остаточным набором знаний, которые я еще не проебал, я могу надеяться только на какую-нибудь специальность, где придется что-то считать первые пару курсов, знания лишними не будут.
1105 9295
Вкотился. Вы отпочковались от sci? Когда это произошло?
1106 9300
>>9282
ты хочешь интерполировать разрывную функцию
Безымянный.png19 Кб, 645x230
1107 9301
FFFUUU как же у меня бомбит от этих двух пределов. Препод говорит, что в первом необходимо на самом начальном этапе перейти к эквивалентности бм, но, в этом случае, всё равно какая-то хуйня получается. Если следовать решению из Китайского Антидемидовича, то препод говорит, что НЕВЕРНО, ибо выделять второй замечательный в сумме - страшная ошибка.

Во втором всё вроде идёт ровно, пока не остаётся дробь с кошинусами в числителе и знаменателе, в которых хуй знает, как доказать, что бесконечности одного порядка.
1108 9304
>>9280
В тематический тред ступай, брат.
1109 9305
>>9295
Давным давно уже. Тут такие драмы были, все пропустил, братан.
1110 9306
>>9295
От нас уже успел форум отпочковаться.
1111 9313
>>9305
Расскажи про драмы.
1112 9314
>>9301
В первом же вроде просто правило Лопиталя применяешь и всё, нет?
1113 9317
>>9314
Лопиталить нельзя.
1114 9318
>>9301
В числителе вынести за скобку e^x, в знаменателе e^2x. получится нечто вроде
[1+ln(1+x^2 exp(-x))]/[2 + ln(1 + x^4 exp(-2x)]
Теперь вспомнить, что ln(1+x) ~ x. такие же примеры всегда по одному и тому же шаблону делаются, ну
1115 9319
>>9317
Потому что учитель сказал решать арифметикой, евпочя?
1116 9320
>>9318
Вооот. Препод сказал, что это неправильно, ибо там сумма.

>>9319
ПОтому что на момент, когда нам задавали эти задания, до правила Лопиталя мы ещё не дошли, а, следовательно, не имеем права пользоваться.
1117 9321
Вообще правильно, потому что ln(1+x) это буквально x+o(x) при x->0, но это не суть даже. Можно и не переходить к эквивалентностям а сразу высилить предел в виде
[1+ln(1+x^2 exp(-x))]/[2 + ln(1 + x^4 exp(-2x)]
простой подстановкой x=0.
1118 9323
>>9320
То есть нужно не получить правильный ответ, а проебать своё время строго определенным способом. Ничем не отличается от покраски травы, в принципе.
1119 9324
>>9323
Разве плохо уметь решать проблему несколькими способами?
1120 9325
>>9323
Кафедра матанализа в нашем ВУЗике достаточно консервативная.
1121 9329
>>9324

>решать проблему несколькими способами


решать проблему нужна самым рациональным способом.
1122 9330
>>9329
И как ты узнаешь, что он самый рациональный, если других не пробовал?
1123 9331
>>9329

>нужна


нужно, епт
1124 9332
>>9330
Конкретно тот пример в лоб решается Лопиталем.
1125 9334
>>9313
Погугли по архивачу. Если коротко: был хорен, потом хорена пидорнули.
1126 9335
>>9332
в лоб решается производной сложной функции, вместо того, чтобы просто вынести экспоненту и получить ответ. Весьма рационально.
1127 9336
Оп, ты где? Обещал после тыщи постов - и проебался. У меня уже тред тормозить начинает, если не появишься - я сам нелегитимно перекачу.
1128 9337
>>9301
во втором скорее всего надо переходить к экспонентам:
chx = (e^x+e^-x)/2
1129 9339
Добро пожаловать в dobromath: https://2ch.hk/math/res/9338.html (М)
1130 9340
>>9301
во втором разность синухов можно в произведение преобразовать и дальше по тексту
1131 9343
Почему интеграл по объему, от двух переменных, суммируется а не умножается?
Ведь по идее мы находим объем тела ограниченного осями, пределами интегрирования и криволинейной поверхностью.
1132 9346
>>9343
можно конкретный пример?
1133 9347
>>9346
Я сейчас прям конкретно не вспомню, но вот в качестве примера функция хе^y.
Интегрируем по икс, получаем x^2e^y/2, интегрируем по игрек, получаем xe^y.
Вот я и спрашиваю, для нахождения полного интеграла их надо умножить? Или полный интеграл интеграл по икс и по игрек одновременно я выдумал пока читал книжку?
СУКА БЛЯДЬ! 1134 9348
Проверьте, что рациональные числа поддаются всем аксиомам действительных чисел окромя аксиомы непрерывности.

ЧТО ОТ МЕНЯ ТРЕБУЮТ?!!
1135 9352
>>9347

>Или полный интеграл я выдумал пока читал книжку?


Ты перепутал с полными дифференциалами. Объёмы то как ты описал в >>9343
вычисляются через кратные интегралы.
1136 9355
>>9348
Доказать, что рациональные числа - упорядоченное поле, в котором аксиома Дедекинда не выполняется.
blob18 Кб, 578x270
Dependency grammars / Constituency Grammars 1137 9356
Господа, Natural Lang Processing.
Не уверен, что с этим прям сюда.
Но точно не к лингвистам.

По ссылке : http://taweb.aichi-u.ac.jp/tmgross/DG.html , коротко о Dependency grammars и две схемы для наглядности. ,

Я не могу уловить принцип по которому слова в схемах объединены именно так, а не иначе.
Объясните пожалуйста.
1138 9358
>>9300
Что? Я интересовался анти интерполяцией, которая позволила бы получить обратно этот квадрат который потрачен при обработке. Но не учёл тот факт что в звуке не бывает отдельных квадратов, там слишком много частот, а что делать если они рядом - наверное никто толком и не знает. тут по крайней мере
1139 9361
>>9356
Так ведь там написано, в чем принцип. Или тебе нужен полный алгоритм синтаксического разбора?
1140 9367
Привет народ
В будущем хочу посвятить себя научной карьере в области математики и информатики.
Но, учиться на очном не могу, нужно как-то зарабатывать на жизнь. Скажите, если сперва отучиться ЗАОЧНО на бакалавра, а уже потом перейти в магистратуру на очное, можно ли таким образом построить профессиональную карьеру в науке? Или обязательно нужно учиться на очном?
1141 9368
>>9352
Ну я провел аналогию, раз у дифффуров есть такое, то по закону бабкина-няньского и у интегралов должно быть что то такое.
Хорошо, где почитать про кратные интегралы чтобы я понял или ты объясни.
1142 9369
>>9358
Это невозможно. Через 8 точек можно провести бесконечно много графиков, как и через 10/300/100(500) и никто не дает гарантию что график получившийся даже после 100(500) точек будет в точности повторять исходный.
Поэтому интерполяция Интерполяция(от лат. inter–polis — «разглаженный, подновлённый, обновлённый; преобразованный») так и называется, что может примерно сказать какой график будет.
1143 9370
Как найти для этого 1°47'24" угла тангенс? Распишите, пожалуйста. Для целых углов понятно. Но как это делать для таких?
1144 9374
Помогите, пожалуйста. Нужно сократить. На второй пытался решать.
1145 9375
>>9367
Сомнительно, что ты сможешь получить хорошее образование, посвящая весь день работе. Но очно учится, конечно, необязательно - по идее, заочно даже лучше, можешь двигаться быстрее.
1146 9376
>>9370
Што? В вольфрам введи.
1147 9380
>>9374
Всё, разобрался. Формулу половинного аргумента не проходили ещё, вот и не мог решить.
1148 9382
>>9370
гельфанд тригонометрия
1149 9383
>>9236
Сложнее чем у зорича я не смогу объяснить.

>Ты же говорил, что это легче


Я такого не говорил.
7092e6123680.jpg69 Кб, 638x741
1150 9385
>>9358
Ну вот косяки интерполяции и возникли от того, что интерполировали не непрерывную функцию.
Если тебе нужно убрать эти косяки лишь для того, чтобы получить обратно этот сигнал по типу меандр, то могу предположить, что тебе надо добавлять отсутствующие нечетные гармоники высших порядков
1151 9390
Посоветуйте литературу по дискретной математике.
n5H3ShMXBF0[1].jpg18 Кб, 947x130
1153 9392
Аноны, посоветуйте как это решать, в деталях не надо, нужно идею/в каком направлении двигаться.
1154 9396
>>9383
Объясни настолько сложно, насколько можешь. Меня интересует твое объяснение, сделанное тобой. Меня интересует не объяснение, а твое объяснение твоими словами, сделанное тобой. Ну чтобы ты объяснил. И автором объяснений был ты. И объяснения были сделаны тобой. И то, что ты объяснял, было бы сделанным тобой твое объяснение. Что бы ты объяснял. Твое собственное определение множества, сделанное тобой. Ну там и связанные определения данные тобой в твоем объяснении.
14134965544672.png47 Кб, 259x291
1155 9397
Как за месяц выучить математику с 7 по 9 класс?
1156 9398
>>9397
Изи, школьная программа вообще мизерная.
1157 9399
Аноны, че посоветуете почитать на русском что бы понимать о чем идет речь когда говорят о производящих функторах типа TOR и EXT
1158 9400
>>9399
что-то по гомологической алгебре, ротман, вайбель там
1159 9401
Сап, аноны, можете, такой вопрос-как оценивается асимптотическая сложность алгоритма. и типы сложности алгоритмов. примеры алгоритмов различной сложности данного дерьма.
можете не расписывать, и можете дать соус из соответствующей литературы
1160 9405
>>9401
Кормен "Алгоритмы: построение и анализ". Сам изучал по этой книге. Многие курсы и статьи также по этой книге пилятся, это что-то типа стандарта
1161 9406
>>9405
>>9401
С такими вопросами в /pr лучше
1162 9407
>>9368

>раз у дифффуров есть такое


Снова-здорова. Диффуры это уравнения, содержащие производные.

>Хорошо, где почитать про кратные интегралы чтобы я понял


Сначала хорошо всосать, что такое первообразная и определённый интеграл особенно второе.
Книг, таких, чтобы "кратные интегральны for dummies" я не встречал. Единственное хочу сказать, что кратные интегралы охуенно раскрываются с своих приложениях по нахождению квадратур площадей и кубатур объёмов фигур. Попробуй Фихтенгольца 2ой том X главу.
На пальцах, кратные интегралы - это такое дерьмо, когда подынтегральная функция последовательно интегрируется по разным переменным.
1163 9408
>>9401
Дасгупта, "Алгоритмы". Вначале совсем базовые вещи типа O-нотации, в конце про P, NP и около того.
>>9406
Там наверно джява-макаки сидят, которые и про двоичный поиск не слыхали.
1164 9410
>>9408
Кстати, может тредж для программинг лангуадж энтузиастов и прочих комплюктер сциенсов запилить? Ну типа тоже математика же, хули.
1165 9412
>>9410
А вроде был в сцы, но можно свой запилить.
1166 9413
Не знаю, как грамотно объяснить, но где можно почитать про запись точек, прямых, отрезков, векторов и т.п. в плоскости или пространстве и их взаимное расположение? Как они там ложатся и трутся друг об друга косинусами (а кто-то и синусами), расстояниями меж друг дружкой.
1167 9416
Есть в математике какой-то особенный шарм, когда ты расписываешь что-то сложное мелом на доске, с головой уходя в эти цифры и формулы, отрываясь от реального мира. А потом училка ставит 5 и мамке показываешь дневник.
1168 9419
>>9413
Я бы потёрся своим синусом об твою точку.
1169 9427
Посоны, я тут закономерность нашёл: если мы возьмём натуральные числа и будем возводить их в квадрат, то n^2 = (n-1)^2+m, где n>1, а m – нечётное число, а (n+1)^2 = n^2+(m+2), где m+2 – следующее нечётное число.

То есть,
1^2 = 1
2^2 = 1+3
3^2 = 1+3+5
4^2 = 1+3+5+7
и т.д.

У этого есть какое-нибудь название?
1170 9429
>>6879 (OP)
Это всё можно математизировать?
1171 9430
Зачем нужна теория типов и почему ее используют в Haskell?
1172 9431
>>9430
Теория типов одна из альтернатив мат. логики в основаниях математики.

>>9430

>почему ее используют в Haskell


Для спецификации и автоматической проверки компилятором свойств программы.
1173 9432
>>9427
сумма последовательных нечетных чисел. Очень просто выводится, если знаешь просто сумму последовательных чисел (1+2+3...). Также нетрудно заметить это геометрически: пирамидку из кубиков можно всегда перестроить в квадрат.
1174 9433
>>9431

>Теория типов одна из альтернатив мат. логики в основаниях математики.


Яйцо как альтернатива курице: звучит как бред.
c7f0099c-19b8-4c03-a14d-c35eb4b6c59d.gif9 Кб, 356x236
1175 9434
Нужно посчитать площадь проекции трёхосного эллипсоида размерами a,b,c на произвольную плоскость Ax+By+Cz=0.
Пытался влоб — не осилил. Ответ вроде интуитивно прост, но не всплывает. Я так понял надо аффинные преобразования курить, чего стоить читануть по этому поводу?
1176 9437
Вектор a = 2, b = 4, угол между ними 60 (pi/3) градусов.
1)Найти угол между a и а-b: 90 (pi/2) градусов. Тут проблем нет.
2)Найти угол между b и a+b, здесь сложности, что это за HEX a+b, как это представить, в чем различие a-b и a+b в данном треугольнике. (0.3333 радиана). Спасибо.
1177 9438
>>9419
Я дам тебе её координаты, если ты мне поможешь
1178 9439
>>9437
Забыл добавить, это начало изучения векторов, и решение опирается на тригонометрию и простую алгебру.
1179 9440
>>9407
Ну производная и интеграл взаимно обратимы, же.

>подынтегральная функция последовательно интегрируется по разным переменным


Это я и так понял по самому тексту.
Короче, такой интеграл можно самому вывести из обычного или нет?
1180 9441
http://mathprofi.ru/ryady_dlya_chajnikov.html
Что вы думайте о статье и о сайте.
Годно ли начать изучение рядов с этого сайта?
1181 9442
>>9441
Статья говно, сайт говно. Годно начать изучение рядов со Шварца.
1182 9443
>>9434
надо сделать так (поворотом), чтобы векторы плоскости стали новыми осями. Останется одна неизвестная z.
1183 9444
Масанята, кто учился\учится на прикладной математике, скажите, там физика есть на первых курсах? Я просто 0 в ней, вот думаю нужно ли начать учить ее сейчас, чтобы потом было больше времени.
1184 9445
>>9437
неясен вопрос. угол между векторами это скалярное произведение векторов делить на произведение модулей. Берешь и считаешь по формулам.
если геометрически, то полезно знать
a-b=a+(-b)
и правило параллелограмма.
1185 9446
>>9442
Какой шварц?
1186 9447
>>9444
В ИТМО когда учился - физика с первого семестра, вместе с лабораторными. В другом ВУЗе, но тоже в примате, был только термех со второго курса. А вообще, такие вопросы скорее в /un.
1187 9448
>>9446
лоран шварц
1188 9451
>>9447
Спасибо за ответ. Чем после учебы занимаешься?
1189 9452
>>9451
Завтра свободен. Жду тебя на Петровско-разумовской, я буду в центре зала с красной розой.
математика саша 1190 9454
Всегда считал, что математика - это ЯЗЫК науки.
http://dedbotan.com/
1191 9455
>>9447
>>9451
Посоны, го я создал: https://2ch.hk/math/res/9453.html (М)
1192 9456
>>9454
Ты охуел тут спамить? Зарепортил
1193 9457
Ищу околоолимпиадного математика для того, чтобы помог мне подготовится к поступлению этим летом в ШАД. Сам 3 курс ВШЭ, минимальный математические знания есть, но без наставника и четкого плана действий, чувствую, что не успею подготовится. Цену обсудим. ДС.
aleqTr4uarANUSyay$andexPUNCTUMr5tWu
1194 9458
Анон, подкинь сайтиков с задачами по Методам Гаусса и Крамера
CodeCogsEqn.gif26x39
1195 9459
Салют, матемач. Вот уже два дня ищу инфу по разложению неправильной дроби на целую и дробную часть в общем виде. Если вносить больше конкретики, то нужно разложить в общем виде следующую дробь (пикрелейтед). Хотя может я просто дауненок и как-то неправильно ищу.
1196 9460
>>9458
каких нахуй сайтиков? вот у тебя цифры какие-то:
270117 001851 #1195 N9457
берешь просто на три столбца
2 7 11 7
18 5 11 19
5 8 4 57
и решаешь эту систему любым методом. Потом на вольфраальфе проверяешь.

>>9459
факториал же только для целых определен, не?
1197 9461
>>9460
Квадратные скобки - это целая часть от деления.
1198 9462
>>9461
т.е. значения для 123456 это 1 0.5 0,(3) 0.5 0.4 1 ?
1199 9463
меня просят показать что производная от корня икс есть минус один делить на д
1200 9464
>>9462
Да, так и есть. Вопрос в том, что делать, если у меня X больше 1,000,000. Вообще, мне нужно взять синус от этой дроби, умноженной на пи, соответственно целая часть мне не нужна, поэтому я пытаюсь найти формулу в общем виде для дробной части.
1,032.jpg12 Кб, 394x157
1201 9476
Вот это все в скобочках, в -2 степени как решать, поясните пожалуйста.
1202 9478
>>9476
Даже не буду спрашивать в каком ты классе. Держи.
1203 9480
Короче у меня тред тормозит, так что я запилил

ПЕРЕКАТ
https://2ch.hk/math/res/9479.html (М)
https://2ch.hk/math/res/9479.html (М)
https://2ch.hk/math/res/9479.html (М)
Тред утонул или удален.
Это копия, сохраненная 25 декабря 2022 года.

Скачать тред: только с превью, с превью и прикрепленными файлами.
Второй вариант может долго скачиваться. Файлы будут только в живых или недавно утонувших тредах. Подробнее

Если вам полезен архив М.Двача, пожертвуйте на оплату сервера.
« /math/В начало тредаВеб-версияНастройки
/a//b//mu//s//vg/Все доски